Você está na página 1de 327

Mestrado Profissional

em Matemtica em Rede Nacional

Iniciao Matemtica

Autores:

Krerley Oliveira Adn J. Corcho

Unidade I:

Captulos I e II
.

Dedicamos este livro as nossas esposas e lhos, que compreenderam

os sbados sacricados em funo de escrev-lo e a nossos pais, por

tudo o que eles representam.

Tente! E no diga que a vitria est perdida. Se de batalhas que se

vive a vida. Tente outra vez! (Raul Seixas)


vi
Sumrio

Prefcio xi
1 Primeiros Passos 1
1.1 Organizando as Ideias . . . . . . . . . . . . . . . . . . 1

1.2 Verdadeiro ou Falso? . . . . . . . . . . . . . . . . . . . 5

1.3 Teoremas e Demonstraes . . . . . . . . . . . . . . . . 9

1.3.1 Mtodos de Demonstrao . . . . . . . . . . . . 10

1.4 Algumas Dicas para Resolver Problemas . . . . . . . . 15

1.5 Solues dos Problemas da Seo 1.4 . . . . . . . . . . 18

1.6 Exerccios . . . . . . . . . . . . . . . . . . . . . . . . . 26

2 Equaes e Inequaes 31
2.1 Equaes do Primeiro Grau . . . . . . . . . . . . . . . 33

2.1.1 Problemas Resolvidos . . . . . . . . . . . . . . . 37

2.2 Sistemas de Equaes do Primeiro Grau . . . . . . . . 42

2.2.1 Problemas Resolvidos . . . . . . . . . . . . . . . 46

2.3 Equao do Segundo Grau . . . . . . . . . . . . . . . . 49

2.3.1 Completando Quadrados . . . . . . . . . . . . . 50

2.3.2 Relao entre Coecientes e Razes . . . . . . . 55

2.3.3 Equaes Biquadradas . . . . . . . . . . . . . . 59

2.3.4 O Mtodo de Viti . . . . . . . . . . . . . . . . 60

vii
viii SUMRIO

2.4 Inequaes . . . . . . . . . . . . . . . . . . . . . . . . . 62

2.5 Inequao do Primeiro Grau . . . . . . . . . . . . . . . 63

2.6 Inequao do Segundo Grau . . . . . . . . . . . . . . . 69

2.6.1 Mximos e Mnimos das Funes Quadrticas . 75

2.7 Miscelnea . . . . . . . . . . . . . . . . . . . . . . . . . 77

2.7.1 Equaes Modulares . . . . . . . . . . . . . . . 77

2.7.2 Um Sistema de Equaes No lineares . . . . . 80

2.8 Exerccios . . . . . . . . . . . . . . . . . . . . . . . . . 81

3 Divisibilidade 89
3.1 Conceitos Fundamentais e Diviso Euclidiana . . . . . 90

3.2 Bases Numricas . . . . . . . . . . . . . . . . . . . . . 99

3.3 Mximo Divisor Comum e Mnimo Mltiplo Comum . 106

3.3.1 Mximo Divisor Comum . . . . . . . . . . . . . 106

3.3.2 Algoritmo de Euclides . . . . . . . . . . . . . . 111

3.3.3 Mnimo Mltiplo Comum . . . . . . . . . . . . 115

3.3.4 Equaes Diofantinas Lineares . . . . . . . . . . 120

3.4 Nmeros Primos e Compostos . . . . . . . . . . . . . . 123

3.5 Procurando Primos . . . . . . . . . . . . . . . . . . . . 127

3.5.1 O Crivo de Eratstenes . . . . . . . . . . . . . . 127

3.5.2 Primos de Mersenne . . . . . . . . . . . . . . . 129

3.5.3 O Teorema Fundamental da Aritmtica . . . . . 133

3.6 Exerccios . . . . . . . . . . . . . . . . . . . . . . . . . 139

4 O Princpio da Casa dos Pombos 143


4.1 Primeiros Exemplos . . . . . . . . . . . . . . . . . . . . 145

4.2 Uma Verso mais Geral . . . . . . . . . . . . . . . . . . 146

4.3 Aplicaes na Teoria dos Nmeros . . . . . . . . . . . . 149

4.4 Aplicaes Geomtricas . . . . . . . . . . . . . . . . . . 151


SUMRIO ix

4.5 Miscelnea . . . . . . . . . . . . . . . . . . . . . . . . . 153

4.6 Exerccios . . . . . . . . . . . . . . . . . . . . . . . . . 157

5 Contagem 161
5.1 Princpio Aditivo da Contagem . . . . . . . . . . . . . 162

5.2 Princpio Multiplicativo de Contagem . . . . . . . . . . 170

5.3 Uso Simultneo dos Princpios Aditivo e Multiplicativo 178

5.4 Permutaes Simples . . . . . . . . . . . . . . . . . . . 181

5.5 Arranjos Simples . . . . . . . . . . . . . . . . . . . . . 184

5.6 Combinaes Simples . . . . . . . . . . . . . . . . . . . 188

5.7 O Binmio de Newton . . . . . . . . . . . . . . . . . . 193

5.8 Contagem e Probabilidades . . . . . . . . . . . . . . . 195

5.9 Exerccios Propostos . . . . . . . . . . . . . . . . . . . 197

6 Induo Matemtica 203


6.1 Formulao Matemtica . . . . . . . . . . . . . . . . . 204

6.2 Aplicaes . . . . . . . . . . . . . . . . . . . . . . . . . 206

6.2.1 Demonstrando Identidades . . . . . . . . . . . . 206

6.2.2 Demonstrando Desigualdades . . . . . . . . . . 210

6.2.3 Induo e Problemas de Divisibilidade . . . . . 212

6.3 Induo na Geometria . . . . . . . . . . . . . . . . . . 215

6.4 Miscelnea . . . . . . . . . . . . . . . . . . . . . . . . . 220

6.4.1 Cuidados ao Usar o Princpio da Induo . . . . 222

6.5 Induo e Recorrncias . . . . . . . . . . . . . . . . . . 222

6.6 Exerccios . . . . . . . . . . . . . . . . . . . . . . . . . 229

7 Desigualdades 233
7.1 Desigualdade Triangular . . . . . . . . . . . . . . . . . 234

7.2 Desigualdade das Mdias . . . . . . . . . . . . . . . . . 238


x SUMRIO

7.3 Desigualdade de Cauchy-Schwarz . . . . . . . . . . . . 245

7.4 Desigualdade de Jensen . . . . . . . . . . . . . . . . . . 246

7.5 Exerccios . . . . . . . . . . . . . . . . . . . . . . . . . 250

8 Polinmios 255
8.1 Operaes com Polinmios . . . . . . . . . . . . . . . . 255

8.2 Algoritmo de Euclides . . . . . . . . . . . . . . . . . . 263

8.3 Sempre Existem Razes de um Polinmio? . . . . . . . . 268

8.3.1 Nmeros Complexos e Razes de Polinmios . . 269

8.4 Exerccios . . . . . . . . . . . . . . . . . . . . . . . . . 272

A Apndice: Funes 279


Referncias 285
Prefcio
Imaginao mais importante que onhe imento.

Albert Einstein

Leo, vo tem uma religio? Assim, uma religio, omo judasmo,

ou ristianismo, ou Matemti a...?

Alon Peres, 6 anos, lho do Matemti o Yuval Peres

Neste livro pretendemos oferecer ao leitor uma introduo Mate-

mtica Elementar. Juntando as experincias didticas vividas pelos

autores individualmente no Brasil e em Cuba, e mais alguns anos

juntos como treinadores de projetos de introduo Matemtica no

estado de Alagoas, esperamos tornar para o leitor a Matemtica mais

interessante, mostrando um pouco do imenso brilho e beleza que ela

esconde.

O livro foi escrito em captulos, cada um deles detalhando um

tema central e trazendo alguns teoremas fundamentais. Com muitos

exemplos e aplicaes dos conceitos introduzidos, pretendemos mos-

trar ao leitor a importncia do assunto abordado. A organizao dos

exemplos tenta seguir uma linha em ordem crescente de diculdade e,

para o melhor aproveitamento do livro, o trabalho com os exerccios

parte fundamental. Ler o enunciado e resolver o maior nmero pos-

xi
xii Prefcio

svel de exerccios imperativo. Como j disse o Prof. Elon Lima,

Matemtica no se aprende passivamente.

Os exemplos e aplicaes dos conceitos, bem como os teoremas,

devem ser lidos com cuidado e muita ateno. Para os estudantes

que desejem treinar para olimpadas de Matemtica, sugerimos que

formem grupos de estudo para trabalhar os temas individualmente,

sob a orientao de um professor. Acreditamos que o texto pode ser

utilizado em uma disciplina elementar num curso de licenciatura ou

bacharelado em Matemtica.

O primeiro captulo para introduzir o leitor no esprito do livro

e dar uma amostra do tipo de problemas e material que seguir nos

demais captulos. So propostos alguns problemas, muitos deles com

solues, e discutimos alguns mtodos importantes para uso no dia a

dia dos estudantes. Nesta discusso inclumos o estudo de proposies

matemticas, provas por contraposio, o mtodo de reduo ao absur-

do e algumas outras regras bsicas e cuidados que devemos ter ao

resolver problemas em Matemtica.

Em seguida, estudamos as equaes do primeiro e do segundo grau.

Estudamos os mtodos de resoluo dessas equaes, sistemas de equa-

es, relaes entre razes e coecientes, bem como alguns problemas

interessantes que podem ser solucionados via essas equaes. Em se-

guida, estudamos inequaes do primeiro e do segundo grau.

O captulo seguinte trata do conceito de divisibilidade . Tentamos

introduzir o leitor nos principais aspectos bsicos, incluindo-se a divisi-

bilidade com resto, mximo divisor comum e mnimo mltiplo comum,

nmeros primos e compostos, e um pouco de equaes diofantinas li-

neares.

Um captulo til para o estudante que deseja participar de Olim-


Prefcio xiii

padas de Matemtica o que trata do princpio da casa dos pombos .

Este captulo um belo exemplo de como algo aparentemente ingnuo

pode gerar consequncias interessantes. Alguns dos exemplos esto

conectados com os captulos anteriores e aparentemente aplicam o

princpio de modo inusitado, em problemas de geometria, teoria dos

nmeros e em reas diversas.

No captulo de contagem, comeamos com noes teis sobre con-

juntos e princpios bsicos para contar os elementos de um conjunto.

Nesse captulo, estamos mais preocupados com as aplicaes imediatas

do assunto, sugerindo alguns problemas para o estudante iniciante.

Seguimos discutindo os tipos de agrupamento de elementos e suas

consequncias. Obtemos o binmio de Newton e introduzimos a no-

o de probabilidade de um conjunto, resolvendo alguns problemas

relacionados.

Em seguida, estudante se depara com uma arma poderosa do mate-

mtico. O mtodo da induo nita estudado procurando conectar

esta noo com os captulos anteriores, reobtendo com o auxlio do

mtodo da induo algumas coisas que j foram deduzidas por outros

mtodos. Vrios exemplos e problemas so resolvidos, alguns deles de

modo surpreendente e inesperado.

No prximo captulo, introduzimos algumas desigualdades popula-

res para o uso do estudante. Algumas dessas desigualdades so muito

importantes no estudo mais profundo da Matemtica e no apare-

cem em cursos introdutrios, apesar de suas provas e aplicaes serem

elementares. Todas as desigualdades aparecem com demonstraes,

em muito dos casos utilizando-se lgebra elementar e o mtodo de

induo nita. So apresentados vrios exemplos que mostram a uti-

lidade dessas desigualdades em alguns problemas prticos. Para xar


xiv Prefcio

o conhecimento, propomos vrios exerccios complementares. Alguns

deles, cuja soluo mais elaborada, so sugeridos. No ltimo ca-

ptulo, estudamos um pouco as propriedades gerais dos polinmios.

Para complementar a formao do leitor menos experiente, inclumos

um apndice sobre funes.

Somos gratos a muitas pessoas que colaboraram com a elaborao

deste livro com sugestes e correes em verses iniciais. Entre eles,

citamos: Carlos Gustavo Moreira, Ali Tahzibi, Feliciano Vitrio, Edu-

ardo Teixeira, Chico Potiguar e vrios de nossos alunos de Iniciao

Cientca e mestrado, que por vrias ocasies deram sugestes para

a melhoria do texto. Um agradecimento especial vai para Fernando

Echaiz, que nos ajudou ativamente nas notas do Captulo 5 que origi-

naram este texto. Finalmente, agradecemos aos revisores pela leitura

cuidadosa e ao comit editorial da SBM, na pessoa da profa. Helena

Lopes, pelo excelente trabalho de editorao.

Macei, Abril de 2010

Krerley Oliveira

Adn J. Corcho
1
Primeiros Passos
Redu tio ad absurdum, que Eu lides gostava tanto, uma das mais

nas armas do matemti o. muito mais no que um movimento

de xadrez: o jogador de xadrez pode ofere er o sa rif io de uma

pea, mas o matemti o ofere e o jogo inteiro.

G. H. Hardy

Neste captulo, discutiremos algumas ideias gerais e convenes

que serviro como base para os diferentes mtodos de resoluo de

problemas que trataremos nos captulos seguintes. Alguns dos exem-

plos que abordamos sero teis para orientar quanto ao cuidado que

devemos ter quando discutimos problemas em Matemtica.

1.1 Organizando as Ideias

Para resolver problemas matemticos precisamos ter bem claro o que

devemos provar e o que estamos assumindo como verdade. sobre

isso que falaremos agora. Comearemos observando as seguintes ar-

maes:

1
2 1 Primeiros Passos

(a) A soma de dois nmeros pares sempre um nmero par.

(b) Todo brasileiro carioca.

(c) A terra um planeta.

(d) Se c o comprimento da diagonal de um retngulo de lados a e

b, ento c 2 = a2 + b 2 .

(e) Se a < 1, ento a2 > a.

Todas as armaes acima se encaixam no conceito de proposio,

que damos a seguir.

Uma proposio ou sentena uma frase armativa em forma de

orao, com sujeito, verbo e predicado, que ou falsa ou verdadeira,

sem dar lugar a uma terceira alternativa.

Por exemplo, as proposies (a) e (c) so claramente verdadeiras;

mais adiante nos convenceremos da veracidade da proposio (d). Por

outro lado, as proposies (b) e (e) so falsas. Com efeito, para cons-

tatar a veracidade da sentena (b) teramos que checar o registro de

nascimento de cada brasileiro e vericar se nasceu no Rio de Janeiro,

mas isto falso pois o conhecido escritor Graciliano Ramos um

brasileiro nascido em Alagoas. Analogamente, para convencer-nos de

que a proposio (e) falsa basta tomar a = 1/2 e checar que (1/2)2 =
1/4 no maior do que 1/2 como a sentena arma. Em ambos os

casos temos vericado que as proposies (b) e (e) so falsas apre-

sentando casos particulares onde as mesmas deixam de valer. Estes

casos particulares so chamados de contraexemplos e so muito teis

para vericar a falsidade de algumas proposies.

Notemos que as proposies (d) e (e) so do tipo:


1.1 Organizando as Ideias 3

Se P, ento Q,

onde P e Q tambm so sentenas. Por exemplo, na proposio (e)

temos que:

P: c o comprimento da diagonal de um retngulo de lados a e b,

Q: c2 = a2 + b2 ,

ou seja, estamos assumindo que P verdade e usando este fato deve-

mos vericar se P verdade ou no.

Uma proposio condicional ou implicativa uma nova proposio

formada a partir de duas proposies P e Q, que escrita na forma:

Se P, ento Q ou P implica Q,

onde para o ltimo caso usamos a notao: P = Q. Chamaremos


a proposio P de hiptese e a proposio Q de tese. A hiptese
tambm chamada de proposio antecedente e a tese, de proposio

consequente.

Por exemplo, na proposio condicional (f ) a hiptese : a<1 e a


2
tese : a > a.
A partir de uma de uma proposio condicional podem-se gerar

novas proposies que so de especial interesse para os matemticos.

Vamos chamar o modo em que apresentamos uma proposio de forma

positiva. Por exemplo, quando enunciamos a proposio

Se como laranja, ento gosto de frutas,

assumimos esta armao como sua forma positiva. Vamos descrever

agora como podemos obter novas proposies a partir desta.


4 1 Primeiros Passos

Forma recproca de uma proposio condicional: para cons-

truirmos a forma recproca, temos que trocar na forma positiva a hi-

ptese pela proposio consequente e vice-versa. Vejamos em nosso

exemplo:

Forma da proposio Hiptese Tese


Positiva como laranja gosto de frutas

Recproca gosto de frutas como laranja

Assim, a recproca de proposio de nosso exemplo ento:

Se gosto de frutas, ento como laranja

Forma contrapositiva de uma proposio condicional: Para

obtermos a forma contrapositiva a partir da forma positiva de uma

proposio condicional podemos fazer primeiro sua forma recproca e

em seguida negamos as sentenas antecedente e consequente da rec-

proca ou, tambm, podemos primeiro negar as sentenas antecedente

e consequente da forma positiva e imediatamente fazer a forma rec-

proca desta ltima. A forma contrapositiva tambm conhecida como

forma contrarrecproca . Usando novamente nosso exemplo temos que:

Forma da Proposio Hiptese Tese


Positiva como laranja gosto de frutas

Recproca gosto de frutas como laranja

Contrapositiva no gosto de frutas no como laranja

Portanto, a forma contrapositiva escreve-se assim:


1.2 Verdadeiro ou Falso? 5

Se no gosto de fruta, ento no como laranja

Em particular, a forma contrapositiva de uma proposio poder ser,

eventualmente, uma forma indireta muito ecaz de vericar resultados

em Matemtica.

1.2 Verdadeiro ou Falso?

Uma das coisas que distingue a Matemtica das demais cincias natu-

rais o fato de que um tema de Matemtica discutido utilizando-se

a lgica pura e, por conta disso, uma proposio em Matemtica, uma

vez comprovada sua veracidade, aceita como verdade irrefutvel e

permanecer assim atravs dos sculos. Por exemplo, at hoje usamos

o teorema de Tales do mesmo modo que foi usado antes de Cristo e

este fato continuar valendo eternamente.

Vamos ilustrar melhor essa diferena com um exemplo em Geo-

graa. Hoje, todos ns sabemos que a Terra tem aproximadamente

o formato de uma laranja, um pouco achatada nos polos. Porm,

na poca de Pitgoras, um dos grandes temores dos navegadores era

encontrar o m do mundo. No pensamento de alguns destes aventu-

reiros, a Terra tinha o formato de um cubo, e uma vez chegando em

um dos seus extremos, o navio despencaria no vazio. Esse um dos

muitos exemplos de como a concepo da natureza mudou ao longo

do tempo, transformando uma concepo verdadeira num perodo da

humanidade em algo completamente falso em outra poca. Porm,

para nossa felicidade, isso no acontece na Matemtica. Uma propo-

sio matemtica ou verdadeira ou falsa e permanecer assim para

sempre.
6 1 Primeiros Passos

Mas como saber se uma proposio verdadeira ou falsa? A pri-

meira coisa que devemos fazer tomar muito cuidado. As aparncias

enganam ou, como diziam nossos avs, nem tudo que reluz ouro.

O leitor, avisado disso, pense agora na seguinte pergunta:

Pergunta 1: Qual a chance de que pelo menos duas pessoas num

nibus com 44 passageiros faam aniversrio no mesmo dia do ano?

Como j avisamos, o leitor deve ter cuidado ao responder per-

gunta acima, pois podemos nos enganar muito facilmente. Por exem-

plo, podemos formular o seguinte argumento errado: o ano tem 365

dias e, como estou escolhendo um grupo de 44 (nmero muito pequeno

com respeito a 365) pessoas ao acaso, claro que podemos responder

pergunta com a seguinte armao:

Resposta intuitiva: A chance de que num grupo de 44 pessoas pelo

menos duas delas faam aniversrio no mesmo dia do ano pequena.

primeira vista a resposta dada pode at parecer verdadeira, mas

com uma anlise mais cuidadosa veremos que completamente falsa.

Na verdade, a chance de que pelo menos duas pessoas do nibus faam

aniversrio no mesmo dia do ano de cerca de 93%!

Quem no acreditar nisto pode fazer duas coisas: primeiro, ir a

sua sala de aula ou no seu nibus escolar, que deve ter pelo menos

44 pessoas, e fazer o experimento ao vivo. Muito provavelmente voc

deve conseguir duas pessoas que fazem aniversrio no mesmo dia do

ano. Se voc verica que existem duas pessoas que fazem aniversrio

no mesmo dia do ano, no por acaso, pois a chance de isso acontecer

muito alta. Mas, cuidado! Isso no uma prova matemtica para

este fato. Para provar que este fato verdadeiro voc deve vericar

que se escolhermos ao acaso um grupo de 44 pessoas ento com aproxi-


1.2 Verdadeiro ou Falso? 7

madamente 93% de chance, pelo menos duas delas fazem aniversrio

no mesmo dia do ano!

Porm, se voc faz o experimento e no encontra duas pessoas que

fazem aniversrio no mesmo dia do ano (voc seria muito azarado!),

no se desespere. Lembre-se de que se trata de algo que acontece com

chance de 93% e que pode no acontecer quando fazemos um teste.

Em qualquer um dos casos, para ter a certeza de que a proposio

verdadeira o leitor deve demonstr-la. Faremos isso no nal do

Captulo

Vamos analisar agora outro fato aparentemente bvio.

Pergunta 2: Num campeonato de futebol onde cada time joga a

mesma quantidade de jogos, cada vitria vale trs pontos, o empate

vale um ponto e a derrota nenhum ponto. Em caso de empate, o

critrio de desempate entre as equipes era o seguinte:

A melhor equipe aquela que tem mais vitrias.

Os organizadores decidiram passar a adotar o critrio a seguir:

A melhor equipe aquela que tem mais derrotas.

Voc acha que este ltimo critrio adotado justo?

Com respeito a esta pergunta, o leitor deve ter respondido do se-

guinte modo:

Resposta: Um time que perdeu mais pior que um que perdeu me-

nos; portanto, a mudana de critrio totalmente injusta. Acertamos

a sua resposta?

Na verdade, no houve mudana nenhuma de critrio, ou seja,

ambos os critrios nos conduzem ao mesmo ganhador.


8 1 Primeiros Passos

Para ver isso rapidamente, lembre-se de que se a equipe A perdeu

mais que a equipe B e ainda assim empataram, ento ela deve ter

ganho mais, para que no m do campeonato a equipe A ainda assim

conseguisse empatar com a equipe B. Vamos mostrar isso precisa-

mente. Sejam d1 , e1 , v1 o nmero de derrotas, empates e vitrias,

respectivamente, da equipe A. Do mesmo modo, sejam d2 , e2 , v2 o

nmero de derrotas, empates e vitrias, respectivamente, da equipe

B. Suponhamos que a equipe A obteve mais vitrias do que a equipe

B, ou seja, que v1 > v2 . Como cada equipe jogou o mesmo nmero de

jogos, temos que

d1 + e1 + v1 = d2 + e2 + v2 . (1.1)

Por outro lado, note que o nmero de pontos obtidos pela equipe A

e1 + 3v1 . Do mesmo modo, o nmero de pontos obtidos pela equipe

B igual a e2 + 3v2 . Como as duas empataram, temos que:

e1 + 3v1 = e2 + 3v2 .

Ou ainda,

e2 e1
3(v1 v2 ) = e2 e1 ou v2 v1 = .
3
Como v1 v2 > 0, temos que e2 e1 > 0. Reescrevendo a equao (1.1),

temos que:

e2 e1 2
d1 d2 = e2 e1 + (v2 v1 ) = e2 e1 = (e2 e1 ).
3 3
Logo, temos que d1 d2 > 0, pois e2 e1 > 0. Isso signica que
A teve mais derrotas que B ; logo, qualquer um dos dois critrios de

desempate usado nos leva equipe vencedora.


1.3 Teoremas e Demonstraes 9

Assim, como estes dois exemplos mostram, ao depararmos com um

problema em Matemtica, devemos ter cuidado ao tirar concluses

apressadas para evitar que cometamos algum engano. Pode acontecer

que uma situao que claramente falsa para um observador menos

atento, se mostre verdadeira quando fazemos uma anlise mais crite-

riosa.

1.3 Teoremas e Demonstraes

Agora denimos o que entendemos por demonstrao matemtica de

uma proposio.

Uma demonstrao em Matemtica o processo de raciocnio l-

gico e dedutivo para checar a veracidade de uma proposio condici-

onal. Nesse processo so usados argumentos vlidos, ou seja, aqueles

que concluam armaes verdadeiras a partir de fatos que tambm

so verdadeiros.

Como exemplo de demonstrao citamos a argumentao usada

para mostrar na segunda pergunta da seo anterior que os critrios

de desempate eram similares.

Sempre que, via uma demonstrao, comprovemos a veracidade de

uma proposio passamos ento a chamar esta de teorema. Assim, um

teorema qualquer armao que possa ser vericada mediante uma

demonstrao.

Alguns teoremas se apresentam na forma de uma proposio con-

dicional, isto , uma sentena do tipo Se P , ento Q ou implicativa


da forma  P = Q. Nesse caso, a sentena P chamada de hiptese

e a sentena Q denominada de tese. Ou seja, a validade da hiptese

nos implica a veracidade da tese.


10 1 Primeiros Passos

Um exemplo de teorema o famoso teorema de Pitgoras , cujo

enunciado diz o seguinte:

Teorema 1.1 (Teorema de Pitgoras) . Num tringulo retngulo a

soma dos quadrados dos catetos igual ao quadrado da hipotenusa.

Notemos que o teorema de Pitgoras no est enunciado na forma

condicional, mas pode ser reescrito nessa forma como:

Teorema 1.2 (Teorema de Pitgoras) . Se T um tringulo retngulo

de catetos a e b e hipotenusa c, ento c = a + b2 .


2 2

Observao 1.3. Em geral, mais comum usar a palavra teorema

apenas para certas proposies que so de grande importncia mate-

mtica, chamando-se simplesmente de proposio ao resto das propo-

sies verdadeiras que admitem uma demonstrao. Para uma discus-

so mais detalhada, recomendamos [8].

1.3.1 Mtodos de Demonstrao


Quando realizamos uma demonstrao no existe um caminho nico.

Dependendo do problema em questo podemos usar mtodos dife-

rentes. A seguir ilustramos os seguintes trs mtodos:

Demonstrao direta.

Demonstrao por contraposio.

Demonstrao por reduo ao absurdo.


1.3 Teoremas e Demonstraes 11

Demonstrao Direta
A demonstrao direta aquela em que assumimos a hiptese como

verdadeira e atravs de uma srie de argumentos verdadeiros e dedu-

es lgicas conclumos a veracidade da tese.

a b
a
b
Q
c b
a

Figura 1.1: Figura auxiliar para a demonstrao do teorema de Pitgoras

Um exemplo de demonstrao direta a que daremos a seguir,

para o teorema de Pitgoras enunciado anteriormente no Teorema

1.1. Com efeito, usando a gura acima temos que a rea do quadrado

de lado a+b a soma das quatro reas dos tringulos retngulos

congruentes pelo critrio lado-ngulo-lado (de catetos a e b) mais a

rea do quadriltero Q, o qual um quadrado visto que cada um dos

seus lados coincide com a hipotenusa c dos tringulos retngulos de

catetos a
b e, alm disso, cada um dos seus ngulos internos
e mede
o
= 180 ( + ) = 180 90 = 90 (veja a Figura 1.1).
Portanto,
ab
(a + b)2 = 4 + c2 ,
2
de onde

a2 + 2ab + b2 = 2ab + c2 ,
e consequentemente

a2 + b 2 = c 2 ,
12 1 Primeiros Passos

como queramos.

Demonstrao por Contraposio


Este mtodo baseado no fato de que a veracidade de forma positiva

de uma proposio equivalente veracidade de sua forma contraposi-

tiva, podendo ser esta ltima, eventualmente, mais fcil de se provar.

Por exemplo, a armao

Se sou alagoano, ento sou brasileiro

equivalente armao

Se no sou brasileiro, ento no sou alagoano

Por exemplo, provemos a seguinte proposio:

Proposio 1.4. Se N2 par, ento N par.

Hiptese: N2 par.

Tese: N par.

Desaamos o leitor a tentar mostrar esta proposio partindo da hip-

tese e tentando concluir a tese. Note que podemos vericar que nossa

proposio verdadeira para vrios valores de N2 como na tabela a

seguir, mas isso no uma prova matemtica da nossa proposio.

N2 4 16 36 64 100 144

N 2 4 6 8 10 12
1.3 Teoremas e Demonstraes 13

Mesmo vericando para um bilho de valores de N 2, sempre nos

restariam nmeros para serem vericados. Como nossas tentativas

de provar a forma positiva dessa proposio esto sendo frustradas,

apelaremos para mostrar a forma contrapositiva da mesma, isto :

Proposio 1.5. Se N no par, ento N2 no par.

Neste caso, temos:

Hiptese: N no par.

Tese: N2 no par.

Demonstrao. Como estamos assumindo que N N


no par, logo

tem que ser mpar, ou seja, existe p, nmero inteiro, tal que N = 2p+1.

Logo,

N 2 = (2p + 1)(2p + 1)
= 4p2 + 2p + 2p + 1
= 4p2 + 4p + 1
= 2(2p2 + 2p) + 1
= 2q + 1,
onde q = 2p2 + 2p. Logo, N 2 = 2q + 1 mpar e conclumos assim

nossa prova.

Demonstrao por Reduo ao Absurdo


Este mtodo uma das ferramentas mais poderosas da Matemtica.

O nome provm do latim reductio ad absurdum e tambm conhecido

como mtodo do terceiro excludo devido ao mesmo estar baseado na


14 1 Primeiros Passos

lei do terceiro excludo que diz o seguinte: uma armao que no

pode ser falsa, dever ser consequentemente verdadeira.

De um modo geral, o roteiro que segue uma demonstrao por

reduo ao absurdo o seguinte:

Assumimos a validade da hiptese.

Supomos que nossa tese falsa.

Usando as duas informaes anteriores conclumos, atravs de

argumentos verdadeiros, uma armao falsa; como tal fato no

poder ocorrer, ento nossa tese dever ser verdadeira.

Vamos mostrar como o mtodo funciona na prtica provando a

seguinte proposio:

Proposio 1.6. Seja x um nmero positivo, ento x + 1/x 2.

Destaquemos primeiramente a nossa hiptese e a nossa tese.

Hiptese: x um nmero positivo.

Tese: x + 1/x 2.

Demonstrao. Seja x um nmero positivo e suponhamos que a tese


1
falsa, isto , x+ x
< 2. Usando que x > 0 e multiplicando por este

a desigualdade anterior, obtemos que

x2 + 1 < 2x.

Da segue-se quex2 2x + 1 < 0 equivalente a (x 1)2 < 0, j que


x2 2x + 1 = (x 1)2 , o que impossvel. Portanto, x + 1/x 2,
como desejvamos.
1.4 Algumas Dicas para Resolver Problemas 15

1.4 Algumas Dicas para Resolver Proble-

mas

Nesta seo, damos algumas regras gerais que consideramos impor-

tante ter em mente na hora de resolver um problema de Matemtica.

Aplicaremos estas regras a alguns problemas interessantes para ilus-

trar a sua importncia. Elas so:

R1) Ler bem o enunciado do problema e utilizar todas as informaes

disponveis.

R2) Fazer casos particulares ou casos mais simples de problemas si-

milares, para adquirir familiaridade com o problema.

R3) Mudar a representao do problema, transformando-o em um

problema equivalente.

R4) Usar a imaginao pesquisando caminhos alternativos. Extra-

polar os limites!

A seguir propomos vrios problemas onde as regras anteriores so

muito teis. O leitor deve tentar resolv-los; mas se no conseguir

achar soluo depois de muito tentar poder ento passar para a pr-

xima seo onde os solucionamos.

Problema 1.7. Ao encontrar uma velha amiga (A), durante uma

viagem de trem, um matemtico (M) tem a seguinte conversa:

(M)  Como vo os trs lhos da senhora?

(A)  Vo bem, obrigada!


16 1 Primeiros Passos

(M)  Qual a idade deles mesmo?

(A)  Vou lhe dar uma dica. O produto das idades deles 36.

(M)  S com essa dica impossvel!

(A)  A soma das idades deles igual ao nmero de janelas deste

vago.

(M)  Ainda no sei!

(A)  O mais velho toca piano!

(M) Agora eu sei!

Voc capaz de descobrir as idades dos trs lhos da senhora?

Problema 1.8. Numa cesta encontram-se 9 moedas idnticas, sendo

que 8 delas tm o mesmo peso e uma moeda mais leve que as demais.

Usando duas vezes uma balana de dois pratos, encontrar a moeda

mais leve.

Problema 1.9. Numa pequena ilha existem 5 pessoas de olhos azuis

e 5 pessoas de olhos verdes. Existe um grande tabu nesta ilha que o

seguinte: se uma pessoa descobre que possui olhos azuis ela se suicida

meia-noite do dia em que descobriu, pulando do alto da prefeitura.

Por conta disso, ningum conversa sobre o assunto, olha para espelhos

ou v seu reexo na gua. Todos se cruzam diariamente e conhecem

os olhos de seus amigos. Numa manh, um estrangeiro chegou ilha

e reuniu as 10 pessoas para o seguinte pronunciamento:

Nesta ilha, existe uma pessoa de olhos azuis.

Pergunta-se:
1.4 Algumas Dicas para Resolver Problemas 17

(a) O que aconteceu com os habitantes da ilha?

(b) Que informao nova o estrangeiro trouxe?

Problema 1.10. Um viajante deseja se hospedar durante 31 dias num

hotel. Entretanto, percebe que est sem dinheiro e que a nica coisa

que possui uma corrente com 31 elos de ouro. Para pagar sua conta,

ele acertou com o gerente pagar um elo por dia, sem atrasar ou a-

diantar o pagamento, durante os 31 dias. O gerente pode dar troco em

elos. Depois ele deseja recuperar a corrente e por isso ele quer pagar

a conta cortando a corrente no menor nmero de pedaos. Quantos

cortes voc conseguiria dar e pagar a conta?

Problema 1.11. Sabendo que em cada jogada o movimento do cavalo

consiste em se deslocar duas casas na horizontal e uma na vertical

ou duas na vertical e uma na horizontal, decidir se possvel sair

da congurao apresentada no tabuleiro (a) e chegar congurao

apresentada no tabuleiro (b) da Figura 1.2 sem que em algum momento

existam dois cavalos na mesma casa.

(a) (b)

Figura 1.2: Cavalos de xadrez


18 1 Primeiros Passos

Problema 1.12. Mostre que podemos cobrir os 9 pontos no reticulado


da Figura 1.3 traando 4 segmentos de reta sem tirar o lpis do papel.

Figura 1.3: Reticulado de 9 pontos

Sugerimos seguir as dicas abaixo para obter sucesso na soluo dos

problemas:

Para os problemas 1.7 e 1.8 use a primeira regra.

Para os problemas 1.9 e 1.10 use a segunda regra. Por exemplo,

no problema 1.9 fazer primeiro o caso: uma pessoa com olhos

azuis e uma com olhos verdes e depois fazer o caso: duas pessoas

de olhos azuis e duas de olhos verdes; generalize.

Para os problema 1.11 use a terceira regra.

Para o problema 1.12 use a quarta regra.

1.5 Solues dos Problemas da Seo 1.4

A seguir apresentamos solues para os problemas enunciados na seo

anterior.

Soluo do Problema 1.7. muito importante neste problema tirar

o mximo de informao das dicas da senhora. Vamos primeira dica:

o produto das idades 36.


1.5 Solues dos Problemas da Seo 1.4 19

Suponhamos que as idades dos lhos sejam 0 6 x 6 y 6 z 6 36.


Como xyz = 36, temos as seguintes possibilidades para os nmeros x,

y e z:
x y z xyz
1 1 36 36
1 2 18 36
1 3 12 36
1 4 9 36
1 6 6 36
2 2 9 36
2 3 6 36
3 3 4 36
A segunda dica dada pela senhora a soma das idades. Assim,

vamos agora calcular todas as possveis somas de acordo com as fato-

raes de 36 dadas na tabela anterior:

x y z x+y+z
1 1 36 38
1 2 18 21
1 3 12 16
1 4 9 14
13
1 6 6
13
2 2 9
2 3 6 11
3 3 4 10
Sabemos que aps a segunda dica, o matemtico ainda no conse-

guiu deduzir as idades das crianas.


20 1 Primeiros Passos

Por que ele no conseguiu? Imagine que o nmero da casa fosse

14. Ora, de acordo com nossa tabela, s existe um terno de nmeros

cujo produto 36 e a soma 14, que o terno (1,4,9). Assim, se o

nmero da casa fosse 14 o matemtico teria dado a resposta aps a

segunda dica. Como ele cou em dvida, olhando a tabela 2, chegamos

concluso de que o nmero da casa s pode ser igual a 13.

Lembremos a ltima dica: o mais velho toca piano. No incio essa

dica parecia intil, mas agora ela fundamental para resolvermos o

problema. De fato, como o mais velho toca piano, isso signica que

existe um mais velho, o que descarta o caso (1,6,6). Assim, as idades

so 2, 2, e 9.

Soluo do Problema 1.8. Este o tipo de problema que a primeira

vista pode parecer difcil, mas que quando usamos todas as informa-

es do seu enunciado se torna fcil. A ideia dividir as moedas em

trs grupos de trs moedas cada, que chamaremos grupos A, B e C.


Colocaremos na balana os grupos A e B e deixaremos o grupo C fora.

Podem acontecer duas coisas:

(a) Os pratos cam equilibrados.

(b) Os pratos cam desequilibrados.

A e B tm o mesmo peso. Logo,


No caso (a), temos que os grupos

a moeda mais leve deve estar no grupo C . No caso (b), um dos grupos

cou mais leve, o que signica que a moeda mais leve est neste grupo.

Assim, utilizando a balana apenas uma vez conseguiremos descobrir

qual o grupo em que a moeda mais leve est. Digamos que este grupo

seja o grupo A. Para achar a moeda mais leve, procedemos de modo

semelhante ao que zemos anteriormente: separamos as trs moedas


1.5 Solues dos Problemas da Seo 1.4 21

do grupo A colocando uma em cada prato e deixando a terceira de

fora. Podem acontecer duas coisas:

(a) Os pratos cam desequilibrados e assim a moeda mais leve est

no prato mais leve.

(b) Os pratos cam equilibrados, logo a moeda mais leve foi a que

cou fora.

No nal, usamos a balana exatamente duas vezes.

Soluo do Problema 1.9. Como em muitos problemas de Mate-

mtica, abordar casos mais simples do problema pode ajudar bastante

na soluo. Assim, vamos imaginar o seguinte caso mais simples:

na ilha existe somente uma pessoa de olhos azuis e a outra de olhos

verdes. Pensando neste caso, a pessoa que tinha olhos azuis s via as

que tinham olhos verdes. Quando o estrangeiro armou que existia

uma pessoa de olhos azuis, ela descobriu que tinha olhos azuis, pois as

outras pessoas tinham olhos verdes. Assim, meia-noite ela subiu na

prefeitura e pulou. Com isso, a pessoa que tinha olhos verdes descobriu

que tinha olhos verdes, pois se ela tivesse olhos azuis sua companheira

no se suicidaria no dia anterior.

Vamos agora dar um passo crucial na soluo do nosso problema

original, considerando o caso onde existem duas pessoas de olhos azuis

e duas pessoas de olhos verdes na ilha. Vamos chamar as pessoas de

olhos azuis de A e B e as pessoas de olhos verdes de C e D. No dia

em que o estrangeiro fez o seu pronunciamento, nada aconteceu, pois

as pessoas C e D viam as pessoas A e B com olhos azuis e a pessoa

A via a pessoa B com olhos azuis e vice-versa. J no segundo dia, a

pessoa A teve o seguinte pensamento:


22 1 Primeiros Passos

Se eu tivesse olhos verdes, a pessoa B teria descoberto que

tinha olhos azuis ontem, pois ela veria trs pessoas de olhos

verdes. Como ela no se suicidou ontem, eu tenho olhos

azuis.

Pensando da mesma forma, a pessoa B descobriu que tambm tinha

olhos azuis. Por isso, meia-noite do segundo dia, as pessoas A e B


se suicidaram.

O que aconteceu depois? As pessoas C e D ainda tinham a dvida

da cor de seus olhos. Para chegar concluso de que seus olhos so

verdes, no terceiro dia, a pessoa C pensou assim:

Bem, se eu tivesse olhos azuis, as pessoas A e B veriam

cada uma duas pessoas com olho azul. Logo, elas no te-

riam se suicidado no segundo dia, pois no conseguiriam

deduzir a cor de seus olhos. Logo, tenho olhos verdes.

Ufa!

Do mesmo modo, a pessoa D conseguiu descobrir a cor de seus olhos.

Analisando de modo semelhante, conseguiremos deduzir que no

problema original as cinco pessoas de olhos azuis descobriro que pos-

suem olhos azuis e juntas se suicidaro no quinto dia aps o pronun-

ciamento do estrangeiro.

Agora vamos descobrir a resposta da segunda pergunta do enun-

ciado: que informao nova o estrangeiro trouxe? Aparentemente

nada de novo foi acrescentado pela frase do estrangeiro, pois cada

pessoa estava vendo alguma pessoa com olhos azuis. Mas isso no

verdade.

Para ver isso e descobrir qual a nova informao que o estrangeiro

trouxe, vamos voltar ao caso de somente duas pessoas na ilha, uma


1.5 Solues dos Problemas da Seo 1.4 23

de olhos azuis e outra de olhos verdes. Neste caso, a pessoa de olhos

azuis somente v uma pessoa de olhos verdes. Com a informao de

que existe uma pessoa de olhos azuis ela pode descobrir a cor de seus

olhos. Note que a pessoa de olhos verdes j sabia que existia pelo

menos uma pessoa de olhos azuis. Mas ela no sabia que a pessoa

de olhos azuis tinha conhecimento de que na ilha existia algum com

olhos azuis. Essa a nova informao que o estrangeiro trouxe.

Soluo do Problema 1.10. Uma primeira soluo cortar a cor-

rente 30 vezes, separando todos os elos. Porm, essa no a melhor so-

luo, como veremos a seguir. Vamos iniciar nossa anlise observando

que para pagar o primeiro dia precisamos dar um corte na corrente.

Assim, o gerente receber um elo. O pulo do gato do problema vem



agora: para pagar o 2 dia, vamos cortar a corrente de modo a separar

dois elos de uma vez. Assim, daremos dois elos ao gerente e ele de-

volver um elo de troco. Com este elo pagaremos o terceiro dia. Note

que pagamos trs dias fazendo dois cortes na corrente, como mostra a

tabela:

Gerente Viajante

Elos 1, 2 28

Note que o nmero 2 denota o pedao que contm 2 elos. Para



pagar o 4 dia, cortaremos a corrente de modo a obter um pedao

com quatro elos. Entregamos ao gerente este pedao e recebemos

de troco um elo solto e um pedao com dois elos. Com o elo solto,

pagamos o 5 dia. Assim, no 5 dia teremos os seguintes grupos de

elos:

Gerente Viajante

Elos 1, 4 2, 24
24 1 Primeiros Passos


Assim, pagamos o 6 dia com o pedao que contm dois elos e

receberemos o elo solto de troco. Finalmente pagaremos o 7 dia com

o elo solto. Note que foi possvel pagar 7 dias com apenas trs cortes na

corrente. A continuao do procedimento est quase revelada. Para



pagar o 8 dia, cortaremos um pedao com oito elos. Daremos este

pedao e receberemos de troco 7 elos, sendo um elo solto, um pedao

com 4 e um pedao com dois elos. Repetindo o procedimento anterior,



pagaremos os 7 dias seguintes, pagando at o 15 dia sem precisar de

cortes adicionais. Ou seja, para pagar os 15 primeiros dias, precisamos

de 4 cortes na corrente. Neste momento, a corrente est distribuda

do seguinte modo:

Gerente Viajante

Elos 1, 2, 4, 8 16


Para pagar o 16 dia, entregaremos ao gerente o pedao com os 16

elos restantes, recebendo 15 elos divididos em pedaos de 1, 2, 4 e 8



elos. Se repetirmos o processo, pagaremos o hotel at o 31 dia sem

precisar de novos cortes. Assim, o mnimo nmero de cortes 4.

Soluo do Problema 1.11. Para resolver este problema vamos usar

a estratgia de mudar a representao . O que signica isso? Vamos

reescrever o problema com outros ingredientes, porm sem alterar em

nada sua essncia. Primeiramente, enumere as casas do tabuleiro com

os nmeros 1, 2, . . . , 9, como na Figura 1.4.

Vamos agora associar ao tabuleiro, um conjunto de nove pontos

tambm enumerados com os nmeros 1, 2, . . . , 9. Se for possvel sair

de uma casa i e chegar casa j com apenas uma jogada do cavalo,

colocaremos um segmento ligando os pontos i e j. Por exemplo,


1.5 Solues dos Problemas da Seo 1.4 25

1 2 3

4 5 6

7 8 9

Figura 1.4: Tabuleiro de 9 casas

possvel, saindo da casa 1 chegar casa 6 e a casa 8. Desse modo,

o ponto com nmero 1 est ligado com o ponto com nmero 8. Se

analisarmos todas as possveis ligaes entre os pontos obteremos um

esquema com o mostrado na Figura 1.5

9 9
2
2 4 4
5 5
7 3 7 3


6 8 6 8
1 1

Figura 1.5: Conexes das casas Figura 1.6: Tabuleiro (a)

Assim, se colocarmos os cavalos como no tabuleiro (a), teremos

a situao descrita na Figura 1.6. Deste modo, ca evidente que no

podemos trocar a posio dos cavalos branco e preto sem que em algum

momento eles ocupem a mesma casa.


26 1 Primeiros Passos

1.6 Exerccios

1. Uma sacola contm meias cujas cores so branca, preta, amarela

e azul. Sem olhar para a sacola, qual a quantidade mnima de

meias que precisamos retirar da mesma para garantir pelo menos

um par de meias da mesma cor?

2. O pai do padre lho nico de meu pai. O que eu sou do padre?

3. Numa mesa h 5 cartas:

Q T 3 4 6

Cada carta tem de um lado um nmero natural e do outro lado

uma letra. Joo arma: Qualquer carta que tenha uma vogal

tem um nmero par do outro lado. Pedro provou que Joo

mente virando somente uma das cartas. Qual das 5 cartas foi a

que Pedro virou?

4. A polcia prende 4 homens, um dos quais culpado de um furto.

Eles fazem as seguintes declaraes:

Arnaldo: Bernaldo o culpvel.

Bernaldo: Cernaldo o culpvel.

Dernaldo: eu no sou culpvel.

Cernaldo: Bernaldo mente ao dizer que eu sou culpvel.

Se se sabe que s uma destas declaraes a verdadeira, quem

culpvel pelo furto?


1.6 Exerccios 27

5. Numa cidade existe uma pessoa X que sempre mente teras,

quintas e sbados e completamente sincera o resto dos dias

da semana. Felipe chega um certo dia na cidade e mantm o

seguinte dilogo com a pessoa X:


 Felipe: Que dia hoje?

 X: Sbado.
 Felipe: Que dia ser amanh?

 X: Quarta-feira.
Em qual dia da semana foi mantido este dilogo?

6. Divida o relgio de parede abaixo em 6 partes iguais de forma tal

que a soma das horas que cam em cada parte seja a mesma.n

12
11 1
10 2
9 3
8 4
7 5
6

7. Joo adora Gabriela, que uma aluna excelente em Matemtica.

Joo armou um plano para dar um beijo nela, e descobriu que

poder fazer isso apenas dizendo uma frase. Que frase essa?

8. No plano se colocam 187 rodas dentadas do mesmo dimetro,

enumeradas de 1 at 187. A roda 1 acoplada com a roda 2, a 2

com a 3, ..., a 186 com a 187 e esta ltima com a roda 1. Pode

tal sistema girar?


28 1 Primeiros Passos

9. Um canal, em forma quadrada, de 4 metros de largura rodeia um

castelo. A ponte do castelo est fechada e um intruso quer entrar

no castelo usando duas pranchas de 3,5 metros de comprimento.

Ser que o intruso consegue?

10. Os nmeros 1, 2, 3, . . . , 99 so escritos no quadro-negro e permi-


tido realizar a seguinte operao: apagar dois deles e substitu-

los pela diferena do maior com o menor. Fazemos esta operao

sucessivamente at restar apenas um ltimo nmero no quadro.

Pode o ltimo nmero que restou ser o zero?

11. Algum elege dois nmeros, no necessariamente distintos, no

conjunto de nmeros naturais 2, . . . , 20. O valor da soma destes

A) e o valor do produto dos


nmeros dado somente a Adriano (

nmeros dado unicamente a Karla ( K).

 Pelo telefone A diz a K: No possvel que descubras minha


soma.

 Uma hora mais tarde, K lhe diz a A: Ah! sabendo disso, j

sei quanto vale tua soma!

 Mais tarde A chama outra vez a K e lhe informa: Poxa,

agora eu tambm conheo teu produto!

Quais nmeros foram eleitos?

12. possvel cobrir um tabuleiro de xadrez com 31 domins onde

removemos as casas dos vrtices superior esquerdo e inferior di-

reito?

13. Num saco encontram-se 64 moedas leves e 64 moedas pesadas.


1.6 Exerccios 29

possvel separar duas moedas de pesos diferentes com 7 pesa-

gens?

14. Quantas vezes precisamos dobrar um papel de 1mm de espessura

para que a altura da pilha chegue da Terra Lua?

15. Descubra o erro da prova da armao abaixo:

Armao: Trs igual a dois.

Seja x um nmero diferente de zero. Temos que:

3x 3x = 2x 2x.

Colocando xx em evidncia, temos que:

3(x x) = 2(x x).

Cancelando xx em ambos os lados, obtemos que 3 = 2.


30 1 Primeiros Passos
2
Equaes e Inequaes

lgebra generosa; ela geralmente nos d mais do que lhe pedimos.

D'Alembert

Na antiguidade, todo conhecimento matemtico era passado de

gerao para gerao atravs de receitas. A falta de smbolos e notao

adequada complicava substancialmente a vida de quem precisava usar

a Matemtica e de quem apreciava sua beleza. Por exemplo, o uso de

letras (x, y, z etc.) para representar nmeros desconhecidos no tinha

sido inventado ainda. Isso s veio ocorrer por volta dos meados do

sculo XVI, ou seja, a menos de 500 anos atrs.

Apesar disso, o conhecimento matemtico das antigas civilizaes

era surpreendente. Os egpcios, babilnios, gregos e vrios outros po-

vos tinham o domnio de mtodos e tcnicas que so empregados hoje,

como solues de equaes do primeiro e segundo graus, inteiros que

so soma de quadrados e vrios outros conhecimentos. Especialmente

os gregos, cuja cultura matemtica resistiu aos tempos com a preser-

vao de Os Elementos de Euclides , tinham desenvolvido e catalisado

31
32 2 Equaes e Inequaes

muitos dos avanos da poca.

Entretanto, todos os resultados tinham uma linguagem atravs dos

elementos de geometria, mesmo aqueles que s envolviam proprieda-

des sobre os nmeros. Essa diculdade deve-se em parte aos sistemas

de numerao que eram utilizados pelos gregos e, posteriormente, pe-

los romanos, que eram muito pouco prticos para realizar operaes

matemticas.

Por volta de 1.100, viveu na ndia Bhaskara, um dos mais impor-

tantes matemticos de sua poca. Apesar de suas contribuies terem

sido muito profundas na Matemtica, incluindo-se a resultados sobre

equaes diofantinas, tudo indica que Bhaskara no foi o primeiro a

descobrir a frmula, que no Brasil chamamos de frmula de Bhaskara,

assim como Pitgoras no deve ter sido o primeiro a descobrir o te-

orema que leva o seu nome, j que 3.000 a.c. os babilnios tinham

conhecimento de ternas pitagricas de nmeros inteiros bem grandes.

Apesar de ter conhecimento de como solucionar uma equao do

segundo grau, a frmula que Bhaskara usava no era exatamente igual

a que usamos hoje em dia, sendo mais uma receita de como encontrar

as razes de uma equao. Para encontrar essas razes, os indianos

usavam a seguinte regra:

Multiplique ambos os membros da equao pelo nmero que vale

quatro vezes o coeciente do quadrado e some a eles um nmero igual

ao quadrado do coeciente original da incgnita. A soluo desejada

a raiz quadrada disso.

O uso de letras para representar as quantidades desconhecidas s

veio a se tornar mais popular com os rabes, que tambm desenvol-

veram um outro sistema de numerao, conhecido como indo-arbico .

Destaca-se tambm a participao do matemtico francs Franois


2.1 Equaes do Primeiro Grau 33

Viti, que aprimorou esse uso dos smbolos algbricos em sua obra In

artem analyticam isagoge e desenvolveu um outro mtodo para resol-

ver a equao do segundo grau.

Na seo seguinte estudaremos com detalhe a equao do primeiro

grau, e como podemos utiliz-la para resolver alguns problemas em

Matemtica.

2.1 Equaes do Primeiro Grau

Iniciamos nossa discusso resolvendo o seguinte problema:

Exemplo 2.1. Qual o nmero cujo dobro somado com sua quinta

parte igual a 121?

Soluo: Vamos utilizar uma letra qualquer, digamos a letra x, para


designar esse nmero desconhecido. Assim, o dobro de x 2x e sua
quinta parte x/5. Logo, usando as informaes do enunciado, obte-

mos que:
x
2x + = 121,
5
ou ainda,

10x + x = 605,
onde 11x = 605. Resolvendo, temos que x = 605/11 = 55.

Se voc j teve contato com o procedimento de resoluo do exem-

plo acima, notou que o principal ingrediente a equao do primeiro

grau em uma varivel.

Denio 2.2. Uma equao do primeiro grau na varivel x uma

expresso da forma

ax + b = 0,
34 2 Equaes e Inequaes

onde a 6= 0, b R e x um nmero real a ser encontrado.

Por exemplo, as seguintes equaes so do primeiro grau:

(a) 2x 3 = 0.

(b) 4x + 1 = 0.
3
(b) x = 0.
2
Para trabalhar com equaes e resolv-las, vamos pensar no mo-

delo da balana de dois pratos. Quando colocamos dois objetos com

o mesmo peso em cada prato da balana, os pratos se equilibram.

Quando os pratos esto equilibrados, podemos adicionar ou retirar a

mesma quantidade de ambos os pratos, que ainda assim eles perma-

necero equilibrados. Essa uma das principais propriedades quando

estamos trabalhando com uma equao. Em geral, para resolver uma

equao, utilizamos as seguintes propriedades da igualdade entre dois

nmeros:

Propriedade 1. Se dois nmeros so iguais, ao adicionarmos a

mesma quantidade a cada um destes nmeros, eles ainda permane-

cem iguais. Em outras palavras, escrevendo em termos de letras, se a


e b so dois nmeros iguais, ento a+c igual a b + c, ou seja,

a=b = a + c = b + c.

Note que podemos tomar c um nmero negativo, o que signica

que estamos subtraindo a mesma quantidade dos dois nmeros. Por

exemplo, se x um nmero qualquer que satisfaz

5x 3 = 6,
2.1 Equaes do Primeiro Grau 35

somando-se 3 a ambos os lados da equao acima, obtemos que x deve


satisfazer:

(5x 3) + 3 = 6 + 3, ou seja, 5x = 9.

Propriedade 2. Se dois nmeros so iguais, ao multiplicarmos a

mesma quantidade por cada um destes nmeros, eles ainda permane-

cem iguais. Em outras palavras, escrevendo em termos de letras, se a


e b so dois nmeros iguais, ento ac igual a b c, ou seja,

a=b = ac = bc.

Por exemplo, se 5x = 9 podemos multiplicar ambos os lados da

igualdade por 1/5 para obter

5x 9
x= = ,
5 5
encontrando o nmero que satisfaz a equao 5x 3 = 6.
Para nos familiarizarmos um pouco mais com a linguagem das

equaes, vamos pensar no seguinte problema:

Exemplo 2.3. Para impressionar Pedro, Lucas props a seguinte

brincadeira:

- Escolha um nmero qualquer.

- J escolhi, disse Pedro.

- Multiplique este nmero por 6. A seguir, some 12. Divida o que

voc obteve por 3. Subtraia o dobro do nmero que voc escolheu. O

que sobrou igual a 4!

Pedro realmente cou impressionado com a habilidade de Lucas.

Mas no h nada de mgico nisso. Voc consegue explicar o que Lucas

fez?
36 2 Equaes e Inequaes

Soluo: Na verdade, Lucas tinha conhecimento de como operar com

equaes. Vamos ver o que Lucas fez de perto, passo a passo, utili-

zando a linguagem das equaes. Para isso, vamos chamar a quanti-

dade que Pedro escolheu de x:

Escolha um nmero: x.

Multiplique este nmero por 6: 6x.

A seguir, some 12: 6x + 12.


6x + 12
Divida o que voc obteve por 3: = 2x + 4.
3
Subtraia o dobro do nmero que voc escolheu: 2x + 4 2x = 4.

O que sobrou igual a 4!

Observao 2.4. Devemos ter cuidado na hora de efetuar divises em

ambos os lados de uma equao, para no cometer o erro de dividir

os lados de uma igualdade por zero. Por exemplo, podemos dar uma

prova (obviamente) falsa de que 1 = 2, utilizando o seguinte tipo de

argumento: sempre verdade que

x + 2x = 2x + x.

Logo,

x x = 2x 2x
Colocando (x x) em evidncia:

1(x x) = 2(x x)

Dividindo por (x x) os dois lados da igualdade acima, temos que

1 = 2. Qual o erro?
2.1 Equaes do Primeiro Grau 37

Para encontrar a soluo da equao ax + b = 0, procedemos do

seguinte modo:

Somamos b a ambos os lados da equao, obtendo

ax + b + (b) = 0 + (b) ax = b.

Note que como somamos a mesma quantidade aos dois lados da

equao, ela no se alterou.

Dividimos os dois lados da equao por a 6= 0. Isso tambm no


altera a igualdade e nos d que o valor de x :

ax b b
= x = .
a a a

Assim, a soluo da equao ax + b = 0

b
x= .
a

2.1.1 Problemas Resolvidos


Vamos ver agora alguns problemas que podem ser solucionados utili-

zando as equaes do primeiro grau.

Problema 2.5. Se x representa um dgito na base 10 e a soma

x11 + 11x + 1x1 = 777,

quem x?
38 2 Equaes e Inequaes

Soluo: Para resolver este problema, precisamos nos recordar que se

abc a escrita de um nmero qualquer na base 10, ento esse nmero

igual a 102 a + 10b + c. Assim, temos que

x11 = 100x + 11
11x = 110 + x
1x1 = 101 + 10x
Logo, temos a seguinte equao do primeiro grau:

100x + 11 + 110 + x + 101 + 10x = 777 ou 111x + 222 = 777

Logo,
777 222
x= = 5.
111

Problema 2.6. Determine se possvel completar o preenchimento

do tabuleiro abaixo com os nmeros naturais de 1 a 9, sem repetio,

de modo que a soma de qualquer linha seja igual a de qualquer coluna

ou diagonal.

1 6

9
Soluo: Primeiro, observe que a soma de todos os nmeros naturais

de 1 a 9 45. Assim, se denotamos por s o valor comum da soma dos

elementos de uma linha, somando as trs linhas do tabuleiro, temos

que:

45 = 1 + 2 + + 9 = 3s,
Onde s deve ser igual a 15. Assim, chamando de x o elemento da

primeira linha que falta ser preenchido,


2.1 Equaes do Primeiro Grau 39

1 x 6

temos que 1 + x + 6 = 15. Logo, x = 8. Assim, observando a coluna


que contm 8 e 9, temos que sua soma maior que 15. Logo, no

possvel preencher o tabuleiro de modo que todas as linhas e colunas

tenham a mesma soma.

Os quadrados de nmeros com essas propriedades se chamam qua-

drados mgicos . Tente fazer um quadrado mgico. Voc j deve ter

percebido que no centro do quadrado no podemos colocar o nmero

9. De fato, vamos descobrir no exemplo abaixo qual o nmero que

deve ser colocado no centro de um quadrado mgico.

Problema 2.7. Descubra os valores de x de modo que seja possvel

completar o preenchimento do quadrado mgico abaixo:

Soluo: Para descobrir x, vamos utilizar o fato de que a soma de

qualquer linha, coluna ou diagonal igual a 15, j obtido no exemplo

anterior. Se somarmos todas as linhas, colunas e diagonais que contm

x, teremos que a soma ser 4 15 = 60, pois existem exatamente uma

linha, uma coluna e duas diagonais que contm x. Note tambm que

cada elemento do quadrado mgico ser somado exatamente uma vez,

exceto x que ser somado quatro vezes. Assim:

1 + 2 + 3 + 4 + + 9 + 3x = 60,

onde temos que 45 + 3x = 60 e consequentemente x = 5.


40 2 Equaes e Inequaes

O problema a seguir um fato curioso que desperta nossa ateno

para como a nossa intuio s vezes falha.

Problema 2.8. Imagine que voc possui um o de cobre extrema-

mente longo, mas to longo que voc consegue dar a volta na Terra

com ele. Para simplicar a nossa vida e nossas contas, vamos supor

que a Terra uma bola redonda (o que no exatamente verdade)

sem nenhuma montanha ou depresso e que seu raio de exatamente

6.378.000 metros.

O o com seus milhes de metros est ajustado Terra, cando

bem colado ao cho ao longo do equador. Digamos agora que voc

acrescente 1 metro ao o e o molde de modo que ele forme um crculo

enorme, cujo raio um pouco maior que o raio da Terra e tenha o

mesmo centro. Voc acha que essa folga ser de que tamanho?

Nossa intuio nos leva a acreditar que como aumentamos to

pouco o o, a folga que ele vai ter ser tambm muito pequena, di-

gamos alguns poucos milmetros. Mas veremos que isso est comple-

tamente errado!

Soluo. Utilizaremos para isso a frmula que diz que o comprimento

C de um crculo de raio r

C = 2r,

onde (l-se pi ) um nmero irracional que vale aproximadamente


3, 1416 (veja a observao a seguir).
De fato, o comprimento da Terra CT calculado com essa frmula

aproximadamente:

CT = 2rT
= 2 3, 1415 6.378.000 = 40.072.974 metros,
2.1 Equaes do Primeiro Grau 41

onde rT o raio da Terra.

Se chamamos de x o tamanho da folga obtida em metros e rf o raio


do o, temos que a folga ser igual a x = rf rT . Logo, basta calcular

rf . Por um lado, o comprimento do o igual a CT + 1 = 40.072.975.


Logo,
40.072.975
40.072.975 = 2rf onde rf = .
2
Fazendo o clculo acima, temos que rf aproximadamente igual

a 6.378.000, 16 metros. Assim, x aproximadamente igual a x =


rf rT = 0, 16 metros, ou seja, 16 centmetros!

Observao 2.9. Vale observar que a folga obtida aumentando o o

no depende do raio em considerao. Por exemplo, se repetssemos

esse processo envolvendo a Lua em vez da Terra, obteramos que ao

aumentar o o em um metro, a folga obtida seria dos mesmos 16

centmetros. Verique isso!

Observao 2.10. De fato, podemos denir (e calcular!) o nmero

de vrias maneiras prticas. Vamos considerar dois experimentos

(que se voc no conhece deve fazer):

Experimento 1: Pegar um cinto e fazer um crculo com ele. Calcule

o comprimento do cinturo e divida pelo dimetro do crculo obtido.

Experimento 2: Pegar uma tampa de uma lata e medir o compri-

mento do crculo da tampa e dividir pelo dimetro da tampa.

Se voc efetuou os clculos acima com capricho, deve ter notado

que o nmero obtido aproximadamente o mesmo. Se nossos crculos

fossem perfeitos (eles nunca so: sempre tm algumas imperfeies)

obteramos o nmero . Uma aproximao para


= 3, 1415926535897932384626433832795.
42 2 Equaes e Inequaes

2.2 Sistemas de Equaes do Primeiro Grau

Nesta seo iremos discutir situaes onde queremos descobrir mais de

uma quantidade, que se relacionam de modo linear, ou seja, atravs

de equaes do primeiro grau. Por exemplo, considere o seguinte pro-

blema:

Exemplo 2.11. Joo possui 14 reais e deseja gastar esse dinheiro em

chocolates e sanduches para distribuir com seus 6 amigos, de modo

que cada um que exatamente com um chocolate ou um sanduche.

Sabendo que cada chocolate custa 2 reais e cada sanduche custa 3

reais, quantos chocolates e sanduches Joo deve comprar?

Para resolver esse problema, vamos chamar de x a quantidade de

chocolates que Joo deve comprar e y o nmero de sanduches. Assim,

como Joo deseja gastar 14 reais, temos que

2x + 3y = 14. (2.1)

Como Joo comprar exatamente 6 guloseimas, uma para cada amigo,

temos que

x + y = 6. (2.2)

Note que no encontramos uma equao do primeiro grau em uma

varivel e sim duas equaes do primeiro grau em duas variveis. Esse

um caso particular de um sistema de equaes do primeiro grau em

vrias variveis.

Denio 2.12. Uma equao do primeiro grau nas variveis x1 , x2 ,


. . . , xn uma expresso da forma

a1 x1 + a2 x2 + + an xn + b = 0,
2.2 Sistemas de Equaes do Primeiro Grau 43

onde os nmeros a1 , a2 , . . . , an so diferentes de zero e b um nmero

real.

Por exemplo,

2x 3y = 0
uma equao do primeiro grau nas variveis x e y. Assim como,
c
2a b + =5
3
uma equao do primeiro grau nas variveis a, b e c.
Dizemos que os nmeros (r1 , r2 , . . . , rn ) formam uma soluo da
equao, se substituindo x1 por r1 , x2 por r2 , . . . , xn por rn , temos

que a equao acima satisfeita, isto , a1 r1 +a2 r2 + +an rn +b = 0.

Por exemplo, (3, 2) uma soluo da equao 2x 3y = 0 acima,

pois

2 3 3 2 = 0.
Note que a ordem que apresentamos os nmeros importa, pois (2, 3)
no soluo da equao 2x 3y = 0, j que 2 2 3 3 = 5 6= 0.
c
Do mesmo modo, (2, 0, 3) soluo da equao 2a b + = 5, pois
3
3
2 2 0 + = 5.
3
Denio 2.13. Um sistema de equaes do primeiro grau em n
variveis x1 , x2 , . . ., xn um conjunto de k equaes do primeiro

grau em algumas das variveis x1 , x2 , . . . , xn , isto , tem-se o seguinte

conjunto de equaes



a11 x1 + a12 x2 + + a1n xn + b1 = 0,

a x + a x + + a x + b = 0,

21 1 22 2 2n n 2
(2.3)





ak1 x1 + ak2 x2 + + akn xn + bk = 0,

44 2 Equaes e Inequaes

onde alguns dos elementos aij (1 i k, 1 j n) podem ser zero.

Porm, em cada uma das equaes do sistema algum coeciente aij

diferente de zero e, alm disso, cada varivel xj aparece em alguma

equao com coeciente distinto de zero.

Dizemos que os nmeros (r1 , r2 , . . . , rn ) formam uma soluo do


sistema de equaes (2.3) se (r1 , r2 , . . . , rn ) soluo para todas as

equaes simultaneamente.

Quando resolvemos um sistema de equaes do primeiro grau, po-

dem acontecer trs situaes:

(a) o sistema tem uma nica soluo;

(b) o sistema tem uma innidade de solues;

(c) o sistema no possui soluo.

A seguir ilustramos com exemplos cada uma das situaes acima.

Situao (a): Retomamos o sistema proposto no Exemplo 2.11, o

qual se encaixa neste caso.


2x + 3y = 14,
x + y = 6.

Isolamos o valor de uma das variveis numa das equaes. Por conveni-

ncia nos clculos isolamos o valor de x na segunda equao, obtendo:

x = 6 y.

A seguir, substitumos esse valor na outra equao, obtendo uma equa-


2.2 Sistemas de Equaes do Primeiro Grau 45

o do primeiro grau. Resolvendo temos:

2(6 y) + 3y = 14,
12 2y + 3y = 14,
y = 2.

Assim, y = 2. Imediatamente, encontramos o valor de x = 6 2 = 4.


Vamos agora resolver alguns problemas semelhantes.

Situao (b): Consideremos os sistema de primeiro grau nas variveis

x, y e z dado por
x + y z 1 = 0,
(2.4)
x y 1 = 0.

Da segunda equao segue-se que

x = y + 1. (2.5)

Substituindo esta expresso na primeira equao obtemos

(y + 1) + y z 1 = 0,
2y z = 0,
z = 2y. (2.6)

Notemos que as variveis xez so resolvidas em funo da varivel y,


a qual no possui nenhuma restrio, de modo que se y assumir um

valor real t ento x e z cam automaticamente determinadas por este

valor t. Isto , para todo t real, de (2.5) e (2.6) tem-se que

x = t + 1, y = t, z = 2t

soluo do sistema (2.4) e, portanto, temos innitas solues para

este.
46 2 Equaes e Inequaes

Situao (c): Consideremos agora o sistema de primeiro grau nas

variveis x, y e z dado por


x + y + 2z 1 = 0,



x + z 2 = 0, (2.7)


y + z 3 = 0.

Neste caso, da segunda e da terceira equao segue-se que

x=2z e y = 3 z.

Substituindo estas expresses na primeira equao obtm-se

(2 z) + (3 z) + 2z 1 = 0 4 = 0,

o que uma incompatibilidade. Logo, este sistema no tem soluo.

Observao 2.14. Os sistemas de equaes de primeiro grau so tam-


bm conhecidos como sistemas de equaes lineares. Quando um sis-

tema de equaes lineares envolve muitas variveis no to fcil

resolv-lo se no se organiza com cuidado seu processo de resoluo.

Existe uma teoria bem conhecida e amplamente divulgada sobre m-

todos de resoluo para esse tipo de sistemas. Um dos mtodos mais

usado e eciente para resolver sistemas lineares o mtodo de elimi-

nao gaussiana. O leitor interessado pode consultar [7].

2.2.1 Problemas Resolvidos


O problema a seguir foi proposto na primeira fase da Olimpada Bra-

sileira de Matemtica.
2.2 Sistemas de Equaes do Primeiro Grau 47

Problema 2.15. Passarinhos brincam em volta de uma velha rvore.

Se dois passarinhos pousam em cada galho, um passarinho ca voando.

Se todos os passarinhos pousam, com trs em cada galho, um galho ca

vazio. Quantos so os passarinhos?

Soluo: Vamos chamar de p o nmero de passarinhos e g o nmero

de galhos da rvore. Temos que se dois passarinhos pousam em cada

galho, um passarinho ca voando, ou seja,

2g = p 1.

Alm disso, se todos os passarinhos pousam, com trs em um mesmo

galho, um galho ca vazio:

3(g 1) = p.

Substituindo na equao anterior, temos que 2g = 3g 3 1, onde

segue-se que g=4 e p = 9.


Problema 2.16. Quanto medem as reas A1 e A2 na gura abaixo,

sabendo que o quadrado tem lado 1 e as curvas so arcos de crculos

com centros nos vrtices V1 e V2 do quadrado, respectivamente.

V2
A2

A1

Soluo: Aplicando relaes de reas na gura temos que


(
A1 + A2 = 4 ,
A1 + 2A2 = 1,
48 2 Equaes e Inequaes

ou seja, chegamos a um sistema de equaes do primeiro grau com

duas incgnitas A1 e A2 . Da primeira equao temos que


A1 = A2 ;
4
substituindo esta na segunda equao obtemos


A2 + 2A2 = 1,
4
de onde

+ A2 = 1.
4

A1 = 4 1 4 =

Logo, A2 = 1 4
e
2
1.

Problema 2.17. Carlos e Cludio so dois irmos temperamentais

que trabalham carregando e descarregando caminhes de cimento. Para

Carlos e Cludio tanto faz carregar ou descarregar o caminho, o tra-

balho realizado por eles o mesmo. Quando esto de bem, trabalham

juntos e conseguem carregar um caminho em 15 minutos. Cludio

mais forte e trabalha mais rpido conseguindo carregar sozinho um

caminho em 20 minutos.

(a) Um dia, Cludio adoeceu e Carlos teve que carregar os caminhes

sozinho. Quanto tempo ele leva para carregar cada um?

(b) Quando os dois brigam, Carlos costuma se vingar descarregando

o caminho, enquanto Cludio o carrega com sacos de cimento.

Quanto tempo Cludio levaria para carregar o caminho com

Carlos descarregando?

Soluo: Vamos chamar de x a quantidade de sacos que Cludio car-

rega por minuto e y a quantidade de sacos que Carlos carrega por


2.3 Equao do Segundo Grau 49

minuto. Como Cludio carrega mais que Carlos, sabemos que y < x.
Do enunciado, sabemos que os dois juntos carregam um caminho em

15 minutos. Se um caminho tem capacidade para c sacos, temos que:


15x + 15y = c.
Alm disso, sabemos que Cludio sozinho carrega o mesmo caminho

em 20 minutos. Logo,

20x = c.
Assim, igualando as duas equaes, temos que

15x + 15y = 20x, onde 15y = 20x 15x = 5x.


Logo, dividindo ambos os lados por 5, temos que 3y = x. Assim, Clu-

dio carrega trs vezes mais sacos que Carlos e a resposta do primeiro

item 20 3 minutos, j que 60y = 20 3y = 20x = c.


Para descobrir quanto tempo os dois levam para carregar o cami-

nho quando esto brigados, observamos que a cada minuto eles car-

regam xy sacos, ou seja, 3y y = 2y sacos. Logo, precisam de 30


minutos, j que 30 2y = 60y = c.

2.3 Equao do Segundo Grau

Como j mencionamos em nossa introduo, o conhecimento de m-

todos para solucionar as equaes do segundo grau remonta s civi-

lizaes da antiguidade, como os babilnios e egpcios. Apesar disso,

a frmula que conhecemos por frmula de Bhaskara , em homenagem

ao matemtico indiano de mesmo nome e que determina as solues

de uma equao do segundo grau, s veio a aparecer do modo que

usamos muito mais tarde, com o francs Viti. Nesta seo iremos

deduzir esta frmula e aplic-la a alguns problemas interessantes.


50 2 Equaes e Inequaes

2.3.1 Completando Quadrados


Um modo de resolver uma equao do segundo grau o mtodo de

completar quadrados. Ele consiste em escrever a equao numa forma

equivalente que nos permita concluir quais so as solues diretamente.

Vamos ilustrar isso com um exemplo, resolvendo a equao

x2 6x 8 = 0.

Podemos escrever essa equao como:

x2 6x = 8.

Somando 9 ao lado esquerdo, obtemos x2 6x + 9 que o mesmo que


2
(x 3) . Assim, somando 9 a ambos os lados da equao, obtemos:

(x 3)2 = 9 + 8 = 17.

Logo, x3= 17 x 3 = 17. Logo, as solues
ou so:


x1 = 3 + 17 e x2 = 3 17.

Denio 2.18. A equao do segundo grau com coecientes a, b e c


uma expresso da forma:

ax2 + bx + c = 0, (2.8)

onde a 6= 0, b, c R e x uma varivel real a ser determinada.

Para encontrar as solues desta equao, vamos proceder do se-

guinte modo: isolando o termo que no contm a varivel x do lado

direito da igualdade na equao (2.8)

ax2 + bx = c
2.3 Equao do Segundo Grau 51

e dividindo os dois lados por a, obtemos:

b c
x2 + x = .
a a
Agora vamos acrescentar um nmero em ambos os lados da equa-

o acima, de modo que o lado esquerdo da igualdade seja um qua-


b2
drado perfeito. Para isso, observe que necessrio adicionar aos
4a2
dois lados da igualdade. Assim, temos que:

2  2
b2 b2 4ac

b 2 b b c
x+ =x +2 x+ = 2 = .
2a 2a 2a 4a a 4a2

Em geral, chamamos a expresso b2 4ac de discriminante da equao


(2.8) e denotamos pela letra maiscula (l-se delta ) do alfabeto

grego. Assim, podemos escrever a igualdade anterior como:

2
b2 4ac

b
x+ = 2
= 2. (2.9)
2a 4a 4a

Por isso, para que exista algum nmero real satisfazendo a igual-

dade acima, devemos ter que 0, j que o termo da esquerda na

igualdade maior ou igual a zero. Extraindo a raiz quadrada quando

0, temos as solues:


b b2 4ac b b2 4ac
x+ = e x+ = .
2a 2a 2a 2a
Assim, obtemos as seguintes solues:


b b2 4ac b +
x1 = + =
2a 2a 2a

e
52 2 Equaes e Inequaes


b b2 4ac b
x2 = = .
2a 4a2 2a

Em resumo,

Se >0 existem duas solues reais.

Se =0 s existe uma soluo real ( x1 = x2 = b/2a).

Se <0 no existe soluo real.

A seguir apresentamos alguns exemplos.

Exemplo 2.19. Encontre as solues da equao 2x2 4x + 2 = 0.

Soluo: Observe que a = 2, b = 4 e c = 2. Logo,

= b2 4ac = (4)2 4 2 2 = 0.
b 4
Assim, a nica soluo x= = = 1.
2a 4
Exemplo 2.20. Encontre as razes da seguinte equao do segundo

grau:

x2 x 1 = 0.

Soluo: Basta aplicarmos diretamente a frmula que acabamos de

deduzir. Como a = 1, b = 1 e c = 1, calculando temos:

= b2 4ac = (1)2 4 1 (1) = 5.

Logo, as solues so

b + 1+ 5 b 1 5
x1 = = e x2 = = .
2a 2 2a 2
2.3 Equao do Segundo Grau 53

Exemplo 2.21. Sabendo que x um nmero real que satisfaz

1
x=1+ ,
1
1+
x
determine os valores possveis de x.

Soluo: A soluo desse problema consiste numa simples manipula-

o algbrica, que feita com cuidado nos levar a uma equao do

segundo grau. Com efeito,

1 x+1
1+ = .
x x
Logo,
1 x 1 + 2x
1+ =1+ = .
1 1+x 1+x
1+
x
1 + 2x
Ento devemos ter x= , de onde segue-se que
1+x
x2 + x = 1 + 2x x2 x 1 = 0.

Resolvendo a equao tem-se



1+ 5 1 5
x1 = e x2 = .
2 2


Observao 2.22. O nmero (1 + 5)/2 chamado de razo urea.

Este nmero recebe essa denominao pois, frequentemente, as pro-

pores mais belas e que a natureza nos proporciona esto prximas

da razo urea. Por exemplo, no arranjo das ptalas de uma rosa, nas

espirais que aparecem no abacaxi, na arquitetura do Parthenon, nos

quadros de da Vinci e nos ancestrais de um zango podemos encon-

trar a razo urea.


54 2 Equaes e Inequaes

O problema a seguir est relacionado com a seqncia de Fibonacci

e com a razo urea. Dizemos que uma seqncia de nmeros an


satisfaz a relao de Fibonacci se, para todo n 0, temos que

an+2 = an+1 + an . (2.10)

Exemplo 2.23. Encontre todas as sequncias an da forma an = x n


para algum x 6= 0 que satisfazem a relao de Fibonacci.

Soluo: Sabendo que an satisfaz a relao de Fibonacci e que an da


n
forma x , podemos concluir que para todo n0 tem-se

xn+2 xn+1 xn = 0.

Colocando xn em evidncia na equao acima, temos que

xn (x2 x 1) = 0

Logo, temos duas possibilidades: xn = 0 ou x2 x 1 = 0. Como


2
x 6= 0, temos que xn 6= 0 e, portanto, x x 1 = 0. Observando a

soluo do Exemplo 2.20 temos que as nicas sequncias so

!n !n
1+ 5 1 5
an = ou an = .
2 2

Observao 2.24. Se an e bn satisfazem a relao de Fibonacci (2.10),


ento dados nmeros reais e , qualquer sequncia da forma an +
bn satisfaz a relao. Pode-se provar que as sequncias dessa forma,

com an = xn1 e bn = xn2 calculados anteriormente, so as nicas

sequncias que satisfazem a relao. Veja, por exemplo, [4].


2.3 Equao do Segundo Grau 55

2.3.2 Relao entre Coecientes e Razes


Dada a equao ax2 + bx + c = 0, com a 6= 0, j calculamos explicita-
mente as suas razes x1 e x2 . Vamos estabelecer agora as relaes entre

a, b e c e as razes x1 e x2 . Vamos supor 0. Como j sabemos,


temos que

b + b
x1 = e x2 = .
2a 2a
Assim, somando x1 com x2 tem-se


b + b 2b b
x1 + x2 = + = = . (2.11)
2a 2a 2a a
Por outro lado, fazendo o produto x1 x2 obtemos

! !
b + b
x1 x2 =
2a 2a
 
b+ b b2 (2.12)
= =
4a2 4a2
4ac c
= 2 = .
4a a
Em particular, quando a = 1, temos o seguinte resultado.

Teorema 2.25. Os nmeros e so as razes da equao

x2 sx + p = 0 (2.13)

se, e somente se,

+ =s e = p. (2.14)
56 2 Equaes e Inequaes

Demonstrao. Com efeito, se e so as razes de (2.13) ento os

clculos feitos em (2.11) e (2.12) nos do (2.14). Reciprocamente, se

vale (2.14) ento da igualdade

(x )(x ) = x2 sx + p

segue-se que e so as razes de (2.13).

Observao 2.26. ax2 + bx + c = 0, com


Em geral, dada a equao

a 6= 0, podemos escrev-la como a(x2 sx + p) = 0, com s = b/a e


p = c/a. Supondo que a equao x2 sx + p = 0 tem razes e , a
igualdade

ax2 + bx + c = a(x2 sx + p) = a(x )(x ) (2.15)

nos permite concluir que e so as razes da equao de segundo


2
grau ax + bx + c = 0.
A equao (2.15) mostra que se raiz de um polinmio do se-

gundo grau, ento a diviso desse polinmio pelo polinmio (x )

uma diviso exata. Voltaremos a tratar desse assunto no Teorema 8.5.

Exemplo 2.27. Paulo cercou uma regio retangular de rea 28 m2


com 24 metros de corda. Encontre as dimenses dessa regio.

Soluo: Se chamamos de a e b os lados do retngulo construdo por

Paulo, as condies sobre o permetro e a rea desse retngulo nos

levam s seguintes equaes:


(
a + b = 12,
ab = 28.

Como j observamos, a e b so razes da equao x2 12x + 28 = 0.


Calculando o discriminante, obtemos = 122 428 = 32. Utilizando
2.3 Equao do Segundo Grau 57

a frmula, temos que as solues so



12 + 32
a= =6+2 2
2
e
12 32
b= = 6 2 2.
2

Exemplo 2.28. Mostre que a equao x2 + bx + 17 = 0 no possui

raiz inteira positiva, se b um inteiro no negativo.

Soluo: Suponhamos que a equao possui alguma raiz inteira n po-


sitiva e sejam a outra raiz (podendo ser m = n). Ento, n + m = b,
onde m = n b dever ser necessariamente um nmero inteiro. Por

outro lado, m e n so nmeros inteiros tais que m n = 17, o que s

possvel se m = 1 ou n = 1, o que nos daria em qualquer um dos

casos que 1 + b + 17 = 0 (b = 18), sendo isto uma contradio com

o fato de b ser inteiro no negativo.

Exemplo 2.29. Numa reunio havia pelo menos 12 pessoas e todos

os presentes apertaram as mos entre si. Descubra quantas pessoas

estavam presentes na festa, sabendo que houve menos que 75 apertos

de mo.

Soluo: Vamos denotar por a o nmero de apertos de mo e enumerar


as pessoas com os nmeros do conjunto P = {1, 2, . . . , n}. A cada

aperto de mo associaremos um par (i, j), signicando que a pessoa i

apertou a mo da pessoa j. Assim, os apertos de mo envolvendo a

pessoa 1 foram


A1 = (1, 2), (1, 3), . . . , (1, n) .
58 2 Equaes e Inequaes

Do mesmo modo, denimos os apertos de mo envolvendo a pessoa 2


que no envolvem a pessoa 1, como


A2 = (2, 3), (2, 4), . . . , (2, n) .

Note que o aperto (2, 1) o mesmo que o aperto (1, 2), j que se 1
aperta a mo de 2, ento 2 aperta a mo de 1. Analogamente,


Ai = (i, i + 1), (i, i + 2), . . . , (i, n) , para 1 i n 1.

Note que Ai Aj = para i 6= j . Observe tambm que todos os

apertos aparecem em um dos conjuntos Ai . Assim, A1 An1


contm todos os apertos de mo. Logo, se |X| denota o nmero de

elementos do conjunto X e a o nmero de apertos de mo, temos

|(A1 A2 An1 )| = |A1 | + |A2 | + + |An1 |


= (n 1) + (n 2) + + 2 + 1
(n 1)n
= = a.
2
Portanto, n2 n 2a = 0 deve admitir admite uma raiz inteira, maior
ou igual a 12. Deste modo, basta descobrirmos para que valores de

a < 75 a equao acima admite alguma raiz inteira n 12. Denotemos


as razes da equao por n1 e n2 e suponhamos que n1 12. Das

relaes (
n1 n2 = 2a,
n1 + n2 = 1,
conclumos que n2 = n1 1 11. Assim, podemos deduzir que

n2 n1 11 12 = 132, pois n2 11 e n1 12.


Observe que o mesmo raciocnio nos leva a concluir que se n1 13
ento n2 n1 = 2a 12 13 = 156, o que nos daria a 78, sendo
2.3 Equao do Segundo Grau 59

isto impossvel pois a < 75. Assim, a raiz positiva para tal equao

no pode ser maior ou igual que 13, restando somente n1 = 12 como


soluo. De fato, essa soluo possvel, se considerarmos a = 66.

Logo, haviam 12 pessoas na festa.

2.3.3 Equaes Biquadradas


A deduo da soluo da equao do segundo grau nos permite resolver

equaes de grau mais alto, desde que elas se apresentem numa forma

peculiar, que nos permita reduzi-las a uma equao do segundo grau.

Por exemplo,

Exemplo 2.30. Resolva a equao

x4 2x2 + 1 = 0. (2.16)

Apesar da equao acima ser de grau quatro, podemos solucion-la

utilizando o que aprendemos at agora. O truque ser denotar por y


2
o valor x .

Soluo: Denote por y = x2 . Neste caso, temos que 0 = y 2 2y + 1 =


(y 1)2 . Logo, y = 1. Assim, x2 = y = 1, de onde segue-se que x = 1

ou x = 1.

De modo geral consideremos a equao

ax2k + bxk + c = 0, k N, (2.17)

e faamos a mudana y = xk . Ento, a equao se transforma na

seguinte;

ay 2 + by + c = 0, (2.18)
60 2 Equaes e Inequaes

a qual j sabemos resolver. Logo, se (2.18) no possui soluo ento

(2.17) tambm no ter soluo e no caso em que y = seja uma

raiz de (2.18) ento as solues para (2.17), correspondentes raiz ,


podem ser encontradas resolvendo a equao simples

xk = ,

a qual tem as seguintes possibilidades:


uma nica soluo: x= k
se k mpar;

nenhuma soluo: se <0 e k par;


duas solues: x=k se >0 e k par.

2.3.4 O Mtodo de Viti


A maneira que Franois Viti (1540-1603) descobriu para resolver a

equao do segundo grau baseia-se em relacionar a equao

ax2 + bx + c = 0 (2.19)

como uma equao do tipo

Ay 2 + B = 0, (2.20)

onde A e B so nmeros que dependem de a, b, c, de modo que qualquer


soluo da equao (2.20) determinar uma soluo da equao (2.19).

Note que a ltima equao possui solues

r r
B B B
y1 = e y2 = , se 0.
A A A
2.3 Equao do Segundo Grau 61

Para fazer isso, usamos o seguinte truque: escrevendo x = u+v como

a soma de duas novas variveis u e v , a equao (2.19) se escreve como

a(u + v)2 + b(u + v) + c = 0,

a qual, desenvolvendo o quadrado, equivale a

au2 + 2auv + av 2 + bu + bv + c = 0.

Agrupando convenientemente, podemos escrever a expresso acima

como uma equao na varivel v, isto ,

av 2 + (2au + b)v + au2 + bu + c = 0.

Assim, podemos obter uma equao do tipo da equao (2.20), esco-

lhendo o valor de u de modo que o termo (2au + b)v se anule. Esco-

lhendo u = b/2a temos que


 2
2 b b b2 b2
av + a b + c = 0 av 2 + + c = 0,
2a 2a 4a 2a
o que equivalente a

b2 + 4ac
av 2 + = 0.
4a
Observando que a equao assumiu a forma da equao (2.20), temos

que suas solues so


r r
b2 4ac b2 4ac
v1 = e v2 = , se = b2 4ac 0.
4a2 4a2
Lembrando que u = b/2a e que x=u+v temos que as solues da

equao (2.19) so

b b
x1 = + v1 e x2 = + v2 ,
2a 2a
como j obtivemos anteriormente.
62 2 Equaes e Inequaes

2.4 Inequaes

Inequaes aparecem de maneira natural em vrias situaes dentro

do contexto matemtico, assim como no prprio dia a dia.

Exemplo 2.31. Numa loja de esportes as bolas de tnis Welson en-

traram em promoo, passando a custar cada uma trs reais. Pedro

que um assduo jogador de tnis quer aproveitar ao mximo a oferta

da loja, mas ele s dispe de cem reais. Qual a maior quantidade

possvel de bolas que Pedro pode comprar?

Soluo. Se denotamos por x o nmero de bolas que Pedro compra,

ento devemos achar o maior valor possvel de x tal que

3x 100. (2.21)

Notemos que o problema se reduz a encontrar o maior mltiplo

positivo de 3 que seja menor ou igual a 100. Observe que 99 = 3 33


o maior mltiplo de 3 menor ou igual a 100, pois 3 34 = 102 > 100
e Pedro no teria oramento para efetuar a compra. Logo, a soluo

x = 33, ou seja, Pedro poder comprar 33 bolas.

Observemos que no exemplo anterior o que zemos foi achar o

maior valor inteiro de x tal que 3x100 < 0; porm note que qualquer
nmero x real menor que 100/3 satisfaz que 3x 100 < 0. Isto um

caso particular de resoluo de uma inequao, chamada inequao do

primeiro grau.
2.5 Inequao do Primeiro Grau 63

2.5 Inequao do Primeiro Grau

Uma inequao do primeiro grau uma relao de uma das formas

abaixo

ax + b < 0, ax + b > 0,
(2.22)
ax + b 0, ax + b 0,

onde a, b R e a 6= 0.
O conjunto soluo de uma inequao do primeiro grau o con-

junto S de nmeros reais que satisfazem a inequao, isto , o conjunto

de nmeros que quando substitudos na inequao tornam a desigual-

dade verdadeira. Para achar tal conjunto ser de vital importncia

tomar em conta as seguintes propriedades das desigualdades entre dois

nmeros

Invarincia do sinal por adio de nmeros reais: sejam

a e b nmeros reais tais que a b, ento a+c b+c para

qualquer nmero real c. O mesmo vale com as desigualdades do

tipo: <, ou >.

Invarincia do sinal por multiplicao de nmeros reais


positivos: sejam a e b nmeros reais tais que a b, ento
ac bc para qualquer nmero real positivo c. Resultados an-

logos valem para as desigualdades do tipo: <, ou >.

Mudana do sinal por multiplicao de nmeros reais


negativos: sejam a e b nmeros reais tais que a b, ento ac
bc para qualquer nmero real negativo c. Resultados anlogos

valem para as desigualdades do tipo: <, ou >.


64 2 Equaes e Inequaes

Vejamos como solucionar as inequaes estritas

ax + b < 0 e ax + b > 0.
Para isto, dividimos a anlise em dois casos.

Caso 1: a>0
Inequao ax + b < 0: neste caso, dividindo por a obtemos

que x + b/a < 0 e somando b/a, em ambos os membros desta

ltima inequao, temos que x < b/a. Portanto,

S = {x R; x < b/a},
o qual representamos no seguinte desenho:

S

b/a

Inequao ax + b > 0: procedendo do mesmo modo que o

caso anterior, obtemos que o conjunto soluo vem dado por

S = {x R; x > b/a},
representado no desenho abaixo:

S

b/a

Caso 2: a<0
Inequao ax + b < 0: neste caso, quando dividimos por ao
sinal da inequao se inverte, obtendo assim que x + b/a > 0,
logo temos que x > b/a e, consequentemente,

S = {x R; x > b/a},
cuja representao na reta a seguinte:
2.5 Inequao do Primeiro Grau 65

S

b/a

Inequao ax + b > 0: similarmente, o conjunto soluo vem

dado por

S = {x R; x < b/a},
cuja representao a seguinte:

S

b/a

Observao 2.32. Notemos que se queremos resolver as inequaes

ax + b 0 e ax + b 0, ento o conjunto soluo S em cada um

dos casos acima continua o mesmo acrescentado apenas do ponto x=


b/a.

Vejamos agora um exemplo simples.

Exemplo 2.33. Para resolver a inequao 8x 4 0, primeiramente

dividimos por 8 a inequao (prevalecendo o sinal da desigualdade)

e imediatamente adicionamos 1/2 em ambos os membros da mesma,

para obter x 4/8 + 1/2 1/2, ou seja,

S = {x R; x 1/2}.

A seguir damos alguns exemplos que podem ser resolvidos usando

inequaes lineares.

Exemplo 2.34. Sem fazer os clculos, diga qual dos nmeros

a = 3456784 3456786 + 3456785 e b = 34567852 3456788

maior?
66 2 Equaes e Inequaes

Soluo. Se chamamos de x ao nmero 3456784 ento das denies


2
de a e b temos que a = x (x + 2) + (x + 1) e b = (x + 1) (x + 4).
2 2
Logo, a = x + 3x + 1 e b = x + x 3. Se supomos que a b, ento

x2 + 3x + 1 x2 + x 3,

e somando x2 x+3 a ambos os membros desta desigualdade obtemos

2x + 4 0.

A soluo desta inequao do primeiro grau o conjunto dos xR


tais que x 2, mas isto falso, desde que x = 3456784. Logo,
nossa suposio inicial de a ser menor ou igual a b falsa, sendo ento
a > b.

O prximo exemplo j foi tratado antes (ver Problema 2.7), porm

apresentamos a seguir uma soluo diferente usando inequaes do

primeiro grau.

Exemplo 2.35. Um quadrado mgico 33 um quadrado de lado 3

dividido em 9 quadradinhos de lado 1 de forma tal que os nmeros de 1

at 9 so colocados um a um em cada quadradinho com a propriedade

de que a soma dos elementos de qualquer linha, coluna ou diagonal

sempre a mesma. Provar que no quadradinho do centro de tal quadrado

mgico dever aparecer, obrigatoriamente, o nmero 5.

Soluo. Primeiramente observamos que a soma 1+2+3++9 =


45, logo como h trs linhas e em cada uma destas guram nmeros

diferentes temos que a soma dos elementos de cada linha 15. Logo,

a soma dos elementos de cada coluna ou diagonal tambm 15.

Chamemos de x o nmero que aparece no centro do quadrado

mgico, como mostra o desenho a seguir.


2.5 Inequao do Primeiro Grau 67

Agora fazemos as seguintes observaes:

O nmero x no pode ser 9, pois nesse caso em alguma linha,

coluna ou diagonal que contm o quadrado central aparecer

o nmero 8, que somado com 9 d 17 > 15 e isto no pode

acontecer.

O nmero x no pode ser 1, pois nesse caso formaria uma linha,

coluna ou diagonal com o nmero 2 e um outro nmero que

chamamos de y, ento 1 + 2 + y = 15 y = 12, o qual

impossvel.

Feitas as observaes anteriores, temos ento que o nmero x forma


uma linha, coluna ou diagonal com o nmero 9 e algum outro nmero

que chamamos de z, logo

z = 15 (x + 9) 1 6 x 1,

de onde segue que x 5.


x aparece numa linha, coluna ou diagonal
Por outro lado, o nmero

com o nmero 1 e algum outro nmero que chamamos de s, logo

s = 15 (x + 1) = 14 x 9, de onde temos que x 5. Finalmente,


como 5 x 5 segue-se que x = 5.

Exemplo 2.36. Num tringulo com lados de comprimento a, b e c


traamos perpendiculares desde um ponto arbitrrio P, sobre o lado

de comprimento c, at cada um dos lados restantes (ver a Figura 2.1).

Se estas perpendiculares medem x e y e a > b, ento


68 2 Equaes e Inequaes

(a) Qual a posio onde deve ser colocado P de maneira que ` = x+y
seja mnimo?

(b) Qual a posio onde deve ser colocado P de maneira que ` = x+y
seja mximo?

C
a
b
x P y
B c A

Figura 2.1: No desenho, os segmentos que partem do ponto P so perpen-

diculares aos lados AC e BC

Soluo. Denotemos por S a rea do tringulo e notemos que divi-

dindo este em dois tringulos menores: um com base a e altura x e

outro com base b e altura y , temos que


ax by
+ = S,
2 2
de onde se segue que

ax = 2S by
2S by
x= .
a
Somando y em ambos os lados da ltima igualdade, obtemos

2S by
x+y = +y
a
2S by + ay
=
a
2S a b
= + y,
a a
2.6 Inequao do Segundo Grau 69

logo

` = + y,

onde
2S ab
= e = .
a a
Agora notemos que 0 y hb , onde hb denota a altura relativa ao

lado de comprimento b no tringulo dado. Como positivo, por ser

a > b, temos ento que 0 y hb e, portanto, + y


+ hb , de onde
0 ` + hb .

Resumindo, o valor mnimo de ` atingido quando y = 0, portanto


P deve ser colocado no vrtice A, e o valor mximo obtido quando
y = hb , portanto P deve ser colocado no vrtice B .

2.6 Inequao do Segundo Grau

Agora passamos a discutir a soluo das inequaes do segundo grau,

que possuem um maior grau de diculdade quando comparadas com

as inequaes do primeiro grau. Ser de vital importncia o uso das

propriedades da funo quadrtica ax2 + bx + c, estudadas no captulo


anterior.

Uma inequao do segundo grau uma relao de uma das formas

abaixo
ax2 + bx + c < 0, ax2 + bx + c > 0,
(2.23)
ax2 + bx + c 0, ax2 + bx + c 0,

onde a, b, c R e a 6= 0. Por simplicidade, chamaremos o nmero a de


2
coeciente lder da funo quadrtica ax + bx + c.
70 2 Equaes e Inequaes

Por exemplo, para resolver a inequao x2 3x + 2 > 0 fatoramos


2
o trinmio usando que as razes da equao x 3x + 2 = 0 so 1 e 2,

isto ,

x2 3x + 2 = (x 1)(x 2).

O trinmio toma valores positivos quando o produto (x 1)(x 2) for


positivo, ou seja, quando os fatores (x 1) e (x 2) tenham o mesmo

sinal:

Ambos positivos:

x1>0x>1

x 2 > 0 x > 2,

logo x > 2.

Ambos negativos:

x1<0x<1

x 2 < 0 x < 2,

logo x < 1.

Portanto, x2 3x + 2 > 0 se, e somente se, x<1 ou x > 2.


A seguir explicamos como podemos resolver a inequao do se-

gundo grau de forma geral.

Suponhamos primeiramente que queremos resolver a inequao

ax2 + bx + c > 0. (2.24)


2.6 Inequao do Segundo Grau 71

Notemos que valem as seguintes igualdades:

 
2 b 2 c
ax + bx + c = a x + x +
a a
b2 b2
 
2 b c
=a x + x+ 2 2 +
a 4a 4a a
 2
  2  (2.25)
2 b b b c
=a x + x+ 2 a
a 4a 4a2 a
 2
b
=a x+ ,
2a 4a

onde = b2 4ac. Considerando esta igualdade, dividimos em vrios

casos:

Caso 1: = b2 4ac > 0. Nesta situao procedemos tomando

em conta o sinal de a.

(a > 0). Usando (2.25) notamos que basta resolver a inequao

 2
b
a x+ > 0.
2a 4a

Como a > 0, multiplicando por 1/a em ambos os membros da

desigualdade anterior o sinal desta no muda, obtendo-se ento

 2
b
x+ 2 > 0.
2a 4a

Agora usamos que >0 para obtermos que


72 2 Equaes e Inequaes

 2  2 !2
b b
x+ 2 = x+
2a 4a 2a 2a
! !
b+ b
= x+ x+
2a 2a
! !
b b +
= x x
2a 2a
= (x )(x ) > 0,

b b+
onde = 2a
e = 2a
so as razes de ax2 + bx + c = 0.
Agora notamos que (x )(x ) > 0 se os fatores (x ) e

(x ) so ambos positivos ou ambos negativos. No primeiro

caso (ambos positivos) temos que x > e x > , mas como

< , ento x > . No segundo caso (ambos negativos), temos


que x < e x < , logo x < , novamente por ser < .

Resumindo, a soluo da inequao vem dada pelo conjunto

S = {x R; x < ou x > },

com a seguinte representao na reta:

S S

(a < 0). Esta situao bem similar anterior, a nica dife-

rena que ao multiplicar por 1/a o sinal se inverte tendo ento


que resolver a inequao
 2
b
x+ < 0,
2a 4a2
2.6 Inequao do Segundo Grau 73

a qual equivalente a provar (seguindo os mesmos passos do

caso anterior) que

(x )(x ) < 0,

b b+
com = 2a
e = 2a
razes de ax2 + bx + c = 0.
Notemos que a desigualdade acima vlida sempre que os sinais

dos fatores (x ) e (x ) forem diferentes. Por exemplo,


se x > 0 e x < 0 temos ento que x deve satisfazer a

desigualdade < x < , mas isso impossvel considerando que

neste caso > , por ser a < 0. No caso restante, se x < 0

e x > 0 temos ento que < x < , o que possvel.

Portanto, o conjunto soluo, neste caso, dado por

S = {x R; < x < },

cuja representao na reta :

Caso 2: = b2 4ac = 0. Usando novamente (2.25), devemos

resolver a inequao
 2
b
a x+ > 0,
2a
b
a qual vlida para qualquer x 6= 2a , se a > 0 e sempre falsa, se

a < 0.
Caso 3: = b2 4ac < 0. Neste caso, quando a positivo todos

os valores de x reais so soluo para (2.24), pois a desigualdade


 2
2 b
ax + bx + c = a x + > 0,
2a 4a
74 2 Equaes e Inequaes


sempre satisfeita, dado que 4a > 0. Por outro lado, se a negativo
no temos nenhuma soluo possvel para a inequao (2.24) j que

 2
2 b
ax + bx + c = a x +
2a 4a

sempre negativo, dado que 4a < 0.

Observao 2.37. Para a desigualdade do tipo

ax2 + bx + c < 0

so obtidos resultados similares, seguindo o mesmo processo descrito

anteriormente. Alm disso, para as inequaes

ax2 + bx + c 0 e ax2 + bx + c 0

os resultados so os mesmos, acrescentados apenas dos pontos ,


ou b/2a, dependendo do caso.

Exemplo 2.38. Provar que a soma de um nmero positivo com seu

inverso sempre maior ou igual que 2.

Soluo. Seja x > 0, ento devemos provar que

1
x+ 2.
x
Partimos da seguinte desigualdade, que sabemos vale para qualquer

x R:
(x 1)2 0

logo

x2 2x + 1 0 x2 + 1 2x.
2.6 Inequao do Segundo Grau 75

Se x positivo, podemos dividir ambos os membros da ltima desi-

gualdade sem alterar o sinal da mesma, ou seja,

1
x+ 2,
x
conforme queramos provar.

2.6.1 Mximos e Mnimos das Funes Quadrti-


cas
A funo quadrtica f (x) = ax2 + bx + c, como j foi observado ante-

riormente, satisfaz a identidade


 2
2 b
ax + bx + c = a x + , (2.26)
2a 4a
onde = b2 4ac. O valor mnimo (mximo ) da funo quadrtica

f (x) o menor (maior) valor possvel que pode assumir f (x) quando

fazemos x percorrer o conjunto dos reais.

Da igualdade (2.26) segue-se que, quando a > 0 o valor mnimo


b b
do trinmio obtido quando x = 2a e este vale f ( 2a ) = 4a .

Similarmente, quando a < 0 o valor mximo do trinmio obtido


b b
quando x= 2a , valendo tambm f ( 2a ) = 4a
Exemplo 2.39. Sejam a, b reais positivos tais que a + b = 1. Provar

que ab 1/4.
Soluo. Notemos que ab = a(1 a) = a2 + a. Denindo f (a) =
a2 + a, basta provar que f (a) 1/4 para qualquer 0 < a < 1.
Completando o quadrado a funo f (a), obtemos

f (a) = (a2 a) = (a2 a + 1/4 1/4) = (a 1/2)2 + 1/4,

logo este assume seu valor mximo igual a 1/4, quando a = 1/2.
76 2 Equaes e Inequaes

Alguns problemas de mximos ou mnimos no parecem que pos-

sam ser resolvidos achando o mximo ou mnimo de funes quadr-

ticas. Porm, estes problemas podem ser reformulados de forma tal

que isto seja possvel. Vejamos um exemplo.

Exemplo 2.40. Na gura abaixo ABCD um retngulo inscrito den-

tro do crculo de raio r. Encontre as dimenses que nos do a maior

rea possvel do retngulo ABCD.

D C
r y
x

A B

Soluo. A rea do retngulo vem dada pela frmula

A = 2x 2y = 4xy.

Usando o teorema de Pitgoras, temos que


y= r 2 x2 , (2.27)

logo, substituindo esta ltima igualdade na frmula de rea anterior,

obtemos

A = 4x r2 x2 .

No muito difcil nos convencermos de que as dimenses, que nos

do a maior rea possvel para o retngulo ABCD, so as mesmas

que nos do o mximo para o quadrado desta rea, ou seja, basta

encontrar as dimenses que maximizam A2 . A vantagem que tem esta


2.7 Miscelnea 77

reformulao do problema que A2 tem uma expresso mais simples,

dada por

A2 = 16x2 (r2 x2 ) = 16r2 x2 16x4 .

Agora fazemos a mudana z = x2 , para obter

 2
r2
A2 = 16z 2 + 16r2 z = 16 z 2
+ 4r4 ,

r2
de onde segue que o menor valor de A2 obtido quando z = 2
e

portanto quando x= r . Usando agora a igualdade (2.27) temos que


2
r
r2 r
y= r2 = .
2 2
Ento, o retngulo de maior rea possvel o quadrado de lado
2r

2
= r 2.

2.7 Miscelnea

Nesta seo combinamos a teoria desenvolvida nos tpicos anteriores

para resolver outros tipos de equaes com um nvel de complexidade

maior.

2.7.1 Equaes Modulares


Uma equao modular aquela na qual a varivel incgnita aparece

sob o sinal de mdulo. Por exemplo, so equaes modulares

(a) |2x 5| = 3;

(b) |2x 3| = 1 3x;


78 2 Equaes e Inequaes

(c) |3 x| |x + 1| = 4.

Para resolver equaes modulares se usam basicamente trs mto-

dos:

(1) eliminao do mdulo pela denio;

(2) elevao ao quadrado de ambos os membros da equao;

(3) partio em intervalos.

Ilustramos a seguir estes mtodos com os exemplos dados em (a),

(b) e (c).

Exemplo 2.41. Resolver a equao |2x 5| = 3.

Soluo: O mtodo (1) pode ser utilizado para resolver esta equao.

Para isto, usamos a denio de mdulo:



a se a 0,
|a| =
a se a < 0.

de onde segue-se a propriedade: seja b um nmero no negativo, ento

|a| = b a = b ou a = b.

Logo, x soluo da equao se, e somente se, x satisfaz uma das

equaes de primeiro grau a seguir:

2x 5 = 3 ou 2x 5 = 3.

Da primeira equao obtemos a soluo x1 = 4 e da segunda obtemos

a soluo x2 = 1.
2.7 Miscelnea 79

Exemplo 2.42. Resolver a equao |2x 3| = 1 3x

Soluo: Resolveremos esta equao pelos mtodos (1) e (2).

Mtodo (1): Aplicando a denio de mdulo temos que resolver a


equao equivalente a resolver os sistemas mistos


2x 3 0, 2x 3 < 0,
(a) ou (b)
2x 3 = 1 3x, (2x 3) = 1 3x.

O sistema (a) no tem soluo visto que a soluo da equao do

primeiro grau 2x 3 = 1 3x 5x = 4 x = 4/5 a qual no satisfaz


a desigualdade 2x 3 0. Por outro lado, no sistema (b) a soluo

da equao (2x 3) = 1 3x tem por soluo x = 2 a qual satisfaz

a inequao 2x 3 < 0. Logo, a nica soluo da equao x = 2.

Mtodo (2): Observemos que a equao equivalente ao sistema


misto
1 3x 0,
(2x 3)2 = (1 3x)2 .

Resolvendo agora a equao de segundo grau (2x 3)2 = (1 3x)2 a


2
qual equivalente a 5x + 6x 8 = 0, temos que as possveis solues
sox1 = 4/5 e x2 = 2, mas x1 descartada pois no satisfaz que
13x1 0. Assim, a soluo do sistema misto e, portanto, da equao
modular apenas x2 = 2.

Exemplo 2.43. |3 x| |x + 1| = 4.
Soluo. Neste caso usaremos o mtodo de partio em intervalos que

consiste no seguinte: marcamos na reta real os valores onde |3 x|


e |x + 1| se anulam, neste caso, x1 = 3 e x2 = 1. Com isto a reta
80 2 Equaes e Inequaes

numrica dividida em 3 intervalos x < 1, 1 x 3 e x > 3.


Agora analisamos a equao em cada intervalo:

Intervalo x < 1: Neste caso a equao modular toma a forma

3 x (x 1) = 4 4 = 4,

Portanto, todo o intervalo x < 1 soluo.

Intervalo 1 x 3: Neste caso a equao modular toma a forma

3 x (x + 1) = 4 2 2x = 4,

de onde segue-se que x = 1. Portanto, neste intervalo a soluo

x = 1.
Intervalo x > 3: Neste caso a equao modular toma a forma

3 + x (x + 1) = 4 4 = 4,

o que uma contradio. Portanto, neste intervalo no temos soluo.

Em resumo, a soluo da equao modular o intervalo x 1.

2.7.2 Um Sistema de Equaes No lineares


O seguinte exemplo nos mostra como podemos combinar a tcnica de

resoluo de sistemas lineares e de equaes de segundo grau para

resolver sistemas mais complicados.

Exemplo 2.44. Resolva o sistema de equaes:



x2 + 3x (x2 2)3 = 3,

x2 + 3x + (x2 2)3 = 1.
2.8 Exerccios 81

Soluo. Propomos a seguinte mudana de variveis:


u= x2 + 3x e v = (x2 2)3 .

Assim, o sistema se converte no sistema de equaes do primeiro grau



u v = 3,
u + v = 1,

o qual tem como soluo u=2 e v = 1. Verique! Assim,


x2 + 3x = 2 x2 + 3x = 4,

sendo x=1 e x = 4 as solues desta equao do segundo grau.

Por outro lado

(x2 2)3 = 1,
de onde x2 2 = 1, sendo x = 1 e x = 1 as solues desta equao.
Logo, a soluo do sistema x = 1, que a nica que satisfaz u = 2

e v = 1 simultaneamente.

2.8 Exerccios

1. Observe as multiplicaes a seguir:

(a) 12.345.679 18 = 222.222.222


(b) 12.345.679 27 = 333.333.333
(c) 12.345.679 54 = 666.666.666

Para obter 999.999.999 devemos multiplicar 12.345.679 por quan-


to?
82 2 Equaes e Inequaes

2. Outro dia ganhei 250 reais, incluindo o pagamento de horas

extras. O salrio (sem horas extras) excede em 200 reais o que

recebi pelas horas extras. Qual o meu salrio sem horas extras?

3. Uma torneira A enche sozinha um tanque em 10 h, uma torneira


B enche o mesmo tanque sozinha em 15 h. Em quantas horas

as duas torneiras juntas enchero o tanque?

4. O dobro de um nmero, mais a sua tera parte, mais a sua quarta

parte somam 31. Determine o nmero.

5. Uma certa importncia deve ser dividida entre 10 pessoas em

partes iguais. Se a partilha fosse feita somente entre 8 dessas

pessoas, cada uma destas receberia R$5.000,00 a mais. Calcule

a importncia.

6. Roberto disse a Valria: Pense um nmero, dobre esse nmero,

some 12 ao resultado, divida o novo resultado por 2. Quanto

deu?  Valria disse 15 ao Roberto, que imediatamente reve-

lou o nmero original que Valria havia pensado. Calcule esse

nmero.

7. Por 2/3 de um lote de peas iguais, um comerciante pagou

R$8.000,00 a mais do que pagaria pelos 2/5 do mesmo lote. Qual

o preo do lote todo?

3a+6 2a+10
8. Determine um nmero real a para que as expresses 8
e
6
sejam iguais.

9. Se voc multiplicar um nmero real x por ele mesmo e do resul-

tado subtrair 14, voc vai obter o quntuplo do nmero x. Qual

esse nmero?
2.8 Exerccios 83

10. Eu tenho o dobro da idade que tu tinhas quando eu tinha a

tua idade. Quando tu tiveres a minha idade, a soma das nossas

idades ser de 45 anos. Quais so as nossas idades?

11. Um homem gastou tudo o que tinha no bolso em trs lojas. Em

cada uma gastou 1 real a mais do que a metade do que tinha ao

entrar. Quanto o homem tinha ao entrar na primeira loja?

12. Com os algarismos x, y e z formam-se os nmeros de dois alga-

rismos xy yx, cuja soma


e o nmero de trs algarismos zxz .
Quanto valem x, y e z ?

13. Quantos so os nmeros inteiros de 2 algarismos que so iguais

ao dobro do produto de seus algarismos?

14. Obter dois nmeros consecutivos inteiros cuja soma seja igual a

57.

15. Qual o nmero que, adicionado ao triplo do seu quadrado, vale

14?

16. O produto de um nmero positivo pela sua tera parte igual a

12. Qual esse nmero?

17. Determine dois nmeros consecutivos mpares cujo produto seja

195.

18. A diferena entre as idades de dois irmos 3 anos e o produto

de suas idades 270. Qual a idade de cada um?

19. Calcule as dimenses de um retngulo de 16 cm de permetro e

15 cm2 de rea.
84 2 Equaes e Inequaes

8
20. A diferena de um nmero e o seu inverso . Qual esse
3
nmero?

21. A soma de dois nmeros 12 e a soma de seus quadrados 74.

Determine os dois nmeros.

22. Um pai tinha 30 anos quando seu lho nasceu. Se multiplicarmos

as idades que possuem hoje, obtm-se um produto que igual

a trs vezes o quadrado da idade do lho. Quais so as suas

idades?

23. Os elefantes de um zoolgico esto de dieta juntos. Num perodo

de 10 dias devem comer uma quantidade de cenouras igual ao

quadrado da quantidade que um coelho come em 30 dias. Em

um dia os elefantes e o coelho comem juntos 1.444 kg de cenoura.

Quantos kilos de cenoura os elefantes comem em 1 dia?

24. Sejam 1 e 2 as razes do polinmio ax2 + bx + c, com a 6= 0.


Calcule as seguintes expresses, em funo de a, b e c:

1 + 2
(a) ;
2

(b) 1 + 2 ;

4 + 4 .
(c) 1 2

25. O nmero 3 a raiz da equao x2 7x 2c = 0. Nessas

condies, determine o valor do coeciente c.

26. Encontre o polinmio p(x) = 2x4 +bx3 +cx2 +dx+e que satisfaz
a equao p(x) = p(1 x).
2.8 Exerccios 85

27. (OBM) Dois meninos jogam o seguinte jogo. O primeiro esco-

lhe dois nmeros inteiros diferentes de zero e o segundo monta

uma equao do segundo grau usando como coecientes os dois

nmeros escolhidos pelo primeiro jogador e 1.998, na ordem

que quiser (ou seja, se o primeiro jogador escolhe a e b o se-


gundo jogador pode montar a equao 1.998x2 + ax + b = 0 ou
ax2 + 1.998x + b = 0 etc.) O primeiro jogador considerado ven-

cedor se a equao tiver duas razes racionais diferentes. Mostre

que o primeiro jogador pode ganhar sempre.

28. (OBM) Mostre que a equao x2 + y 2 + z 2 = 3xyz tem innitas

solues onde x, y, z so nmeros inteiros.

29. (Gazeta Matemtica, Romnia) Considere a equao

a2 x2 (b2 2ac)x + c2 = 0,

onde a, b e c so nmeros inteiros positivos. Se n N tal que

p(n) = 0, mostre que n um quadrado perfeito.

30. (Gazeta Matemtica, Romnia) Sejam a, b Z. Sabendo que a

equao

(ax b)2 + (bx a)2 = x,

tem uma raiz inteira, encontre os valores de suas razes.

31. (Gazeta Matemtica, Romnia) Resolva a equao:

2x2
 
= x.
x2 + 1
Obs.: [x] o menor inteiro maior ou igual a x.
86 2 Equaes e Inequaes

32. Demonstrar que:

(a) n4 + 4 no primo se n > 1;


(b) generalize, mostrando que n4 + 4n no primo, para todo

n > 1.

33. Para fazer 12 bolinhos, preciso exatamente de 100 g de acar,

50 g de manteiga, meio litro de leite e 400 g de farinha. Qual a

maior quantidade desses bolinhos que serei capaz de fazer com

500 g de acar, 300 g de manteiga, 4 litros de leite e 5 kg de

farinha ?

34. Dadas as fraes

966666555557 966666555558
e ,
966666555558 966666555559
qual maior?

35. Achar o maior valor inteiro positivo de n tal que

n200 < 5300 .

36. Achar o menor valor inteiro positivo de n tal que

1 2 3 n
10 11 10 11 10 11 10 11 > 100000.

37. Nove cpias de certas notas custam menos de R$ 10,00 e dez

cpias das mesmas notas (com o mesmo preo) custam mais de

R$ 11,00. Quanto custa uma cpia das notas?

38. Se enumeram de 1 at n as pginas de um livro. Ao somar estes

nmeros, por engano um deles somado duas vezes, obtendo-se

o resultado incorreto: 1.986. Qual o nmero da pgina que foi

somado duas vezes?


2.8 Exerccios 87

39. Determine os valores de a para os quais a funo quadrtica


2
ax ax + 12 sempre positiva.

40. Ache os valores de x para os quais cada uma das seguintes ex-

presses positiva:

x x3 x2 1
(a) (b) (c)
2
x +9 x+1 x2 3x
41. Resolver a equao:

[x]{x} + x = 2{x} + 10,

onde [x] denota a parte inteira de x. Por exemplo, [2, 46] = 2 e

[5, 83] = 5. O nmero {x} chamado parte fracionria de x e

denido por {x} = x [x].

42. Mostre que entre os retngulos com um mesmo permetro, o de

maior rea um quadrado.

43. Entre todos os tringulos issceles com permetro p xado, ache

as dimenses dos lados daquele que possui a maior rea.

44. (OBM Jnior 1993)

dada uma equao do segundo grau x2 + ax + b = 0, com

razes inteiras a1 e b1 . Consideramos a equao do segundo grau


x + a1 x + b1 = 0. Se a equao x2 + a1 x + b1 = 0 tem razes
2

2
inteiras a2 e b2 , consideramos a equao x + a2 x + b2 = 0. Se a
2
equao x +a2 x+b2 = 0 tem razes inteiras a3 e b3 , consideramos
2
a equao x +a3 x+b3 = 0. E assim por diante. Se encontramos

uma equao com < 0 ou com razes que no sejam inteiros,

encerramos o processo.
88 2 Equaes e Inequaes

Por exemplo, se comeamos com a equao x2 = 0 podemos

continuar o processo indenidamente. Pede-se:

(a) Determine uma outra equao que, como x2 = 0, nos per-

mita continuar o processo indenidamente;

(b) Determine todas as equaes do segundo grau completas a

partir das quais possamos continuar o processo indenida-

mente.
Referncias Bibliogrcas
[1] AIGNER, M. e ZIEGLER, G. (2002). As Provas esto

no Livro. Edgard Blcher.

[2] GARCIA, A. e LEQUAIN, I. (2003). Elementos de l-

gebra. Projeto Euclides, IMPA.

[3] LIMA, E. L.; CARVALHO, P. C. P.; WAGNER, E. e

MORGADO, A.C. (2004). A Matemtica do Ensino M-

dio. Volume 1. Sociedade Brasileira de Matemtica.

[4] LIMA, E.L.; CARVALHO, P. C. P.; WAGNER, E. e

MORGADO, A.C. (2004). A Matemtica do Ensino M-

dio. Volume 2. Sociedade Brasileira de Matemtica.

[5] LIMA,E.L.; CARVALHO,P. C. P.; WAGNER,E. e

MORGADO,A.C. (2004). A Matemtica do Ensino M-

dio. Volume 3. Sociedade Brasileira de Matemtica.

[6] LIMA, E.L.; CARVALHO, P. C. P.; WAGNER,E. e

MORGADO, A.C. (2001). Temas e Problemas. Socie-

dade Brasileira de Matemtica.

[7] LIMA, E.L. (2001). lgebra Linear. Sociedade Brasileira

de Matemtica.

285
286 REFERNCIAS BIBLIOGRFICAS

[8] MORAIS FILHO, D. C. (2007). Um Convite Matem-

tica. EDUFCG.

[9] MORGADO, A.; CARVALHO, J.; CARVALHO, P.;

FERNANDEZ, P. (1991). Anlise Combinatria e Pro-

babilidade . Sociedade Brasileira de Matemtica.

[10] RIBENBOIM, P. (2001). Nmeros Primos: Mistrios e

Recordes. Sociedade Brasileira de Matemtica.

[11] SANTOS, J. P. O. (1993) Introduo Teoria dos N-

meros. IMPA.

[12] SANTOS, J. P. O.; MELLO, M. P. e MURARI, I. T.

C. (2006). Introduo Anlise Combinatria. Editora

Unicamp.

[13] SOARES, M. G. (2005). Clculo em uma Varivel Com-

plexa. Sociedade Brasileira de Matemtica.


Mestrado Profissional
em Matemtica em Rede Nacional

Iniciao Matemtica

Autores:

Krerley Oliveira Adn J. Corcho

Unidade II:

Captulos III e IV
3
Divisibilidade

Os nmeros governam o mundo.

Plato

A teoria dos nmeros o ramo da Matemtica que estuda os mis-

trios dos nmeros e teve sua origem na antiga Grcia. Os belssimos

problemas ligados a esta rea constituem, at hoje, uma das princi-

pais fontes inspiradoras dos amantes da Matemtica. Alm disso, essa

rea possui vrias aplicaes teis a humanidade, como por exemplo,

o processo de criptograa usado em transaes pela Internet.

Alguns problemas em teoria dos nmeros demoram sculos para

serem resolvidos, como por exemplo o ltimo teorema de Fermat , que


arma que no existe nenhum conjunto de inteiros positivos x, y, z e n

com n maior que 2 que satisfaa xn + y n = z n . Esse problema foi ob-


jeto de fervorosas pesquisas durante mais de 300 anos e foi nalmente

demonstrado em 1995 pelo matemtico Andrew Wiles.

Ainda hoje persistem muitas questes naturais e simples sem res-

posta. Por exemplo, ningum sabe mostrar (apesar de todo mundo

89
90 3 Divisibilidade

acreditar que verdade!) todo natural par soma de dois pri-


que

mos. Essa a famosa conjectura de Goldbach . Essa simplicidade de se


anunciar problemas e a extrema diculdade em resolv-los faz desta

rea um grande atrativo para os matemticos do mundo todo.

Este captulo ser dedicado ao estudo de algumas propriedades

bsicas relativas aos nmeros inteiros.

3.1 Conceitos Fundamentais e Diviso Eu-

clidiana

Denotamos por Z o conjunto dos nmeros inteiros formado pelo con-

junto dos nmeros naturais N = {1, 2, 3, . . .} munido do zero e dos

nmeros negativos. Ou seja, Z = {. . . , 3, 2, 1, 0, 1, 2, 3, . . .}.


Comeamos observando que a soma, diferena e produto de nme-

ros inteiros tambm sero nmeros inteiros. Entretanto, o quociente

de dois inteiros pode ser um inteiro ou no.

Uma das propriedades fundamentais dos nmeros naturais que uti-

lizaremos ao longo do texto o conhecido princpio da boa ordenao ,


que arma o seguinte:

Princpio da Boa Ordenao: todo subconjunto no vazio A N


possui um elemento menor que todos os outros elementos deste, ou
seja, existe a A tal que a n para todo n A.
Por exemplo, se A o conjunto dos nmeros pares, o menor ele-

mento de A o nmero 2. Por outro lado, observamos que o conjunto

dos nmeros inteiros no goza da boa ordenao.

Apesar do princpio da boa ordenao parecer inocente e natural,

muitos resultados importantes a respeito dos nmeros naturais decor-


3.1 Conceitos Fundamentais e Diviso Euclidiana 91

rem do mesmo, como veremos ao longo de todo este captulo.

Denio 3.1. Sejam a e b inteiros. Dizemos que a divide b se existe


um inteiro q tal que b = aq . Tambm usaremos as frases a divisor

de b ou b mltiplo de a para signicar esta situao.

Usaremos a notao a | b para representar todas as frases equi-

valentes ditas anteriormente. Se a no for divisor de b, ento escre-

veremos a - b.

Exemplo 3.2. 7 | 21 pois 21 = 7 3. Por outro lado 3 - 8 pois


considerando o conjunto M = {3m, m N} = {3, 6, 9, 12, . . .} dos
mltiplos positivos de 3 vemos que 8 no pertence ao mesmo.

A seguinte proposio um bom exerccio para entender os con-

ceitos enunciados acima.

Proposio 3.3. Sejam a, b e c nmeros inteiros. Ento,


(a) se a | b e b | c ento a | c;

(b) se a | b e a | c ento a | (b + c) e a | (b c);

(c) se a e b so positivos e a | b ento 0 < a b;

(d) se a | b e b | a ento a = b ou a = b.

Demonstrao. Se a|b e b|c ento existem inteiros q1 e q2 tais que

b = aq1 (3.1)

c = bq2 . (3.2)
92 3 Divisibilidade

Substituindo (3.1) em (3.2) temos que

c = aq1 q2 = aq, onde q = q1 q2 Z, (3.3)

provando isto a armao feita em (a).

Agora provaremos (b). Com efeito, se a | b e a | c valem as

igualdades

b = aq1 , q1 Z (3.4)

c = aq2 , q2 Z. (3.5)

Operando com os ambos lados das igualdades (3.4) e (3.5) temos que

b + c = a(q1 + q2 ) e b c = a(q1 q2 ),
| {z } | {z }
rZ sZ

obtendo assim o resultado desejado.

Continuamos agora com a prova de (c). De fato, se a | b, sendo

ambos positivos, ento b = aq com

q 1. (3.6)

Logo, multiplicando por a ambos lados de (3.6) temos (como a posi-


tivo) que

b = aq a > 0,

como espervamos.

Finalmente, provaremos (d). Com este propsito observamos que

se a|b e b|a |a| divide |b| e |b| divide |a|. Portanto, pelo item
ento

(c) temos que |a| |b| e |b| |a|, ou seja, |a| |b| |a|. Logo,

|a| = |b| e consequentemente a = b ou a = b.


3.1 Conceitos Fundamentais e Diviso Euclidiana 93

Exemplo 3.4. Prove que o nmero N = 545362 7 no divisvel por


5.

Soluo. Vamos mostrar isso utilizando o mtodo do absurdo. Se


45362
este nmero fosse divisvel por 5, ento 5 7 = 5q . Logo, 7 =
45362
5 5q , ou seja, 7 seria divisvel por 5, o que um absurdo.

O prximo passo de nossa discusso ver o que acontece quando

um nmero no divisvel por outro. Por exemplo, analisemos se 31

divisvel por 7 e para isto listaremos a diferena entre 31 e os mltiplos

positivos de 7, isto :

r1 = 31 7 1 = 24,
r2 = 31 7 2 = 17,
r3 = 31 7 3 = 10,
r4 = 31 7 4 = 3,
r5 = 31 7 5 = 4,
r6 = 31 7 6 = 11,
.
.
.

Claramente 31 no divisvel por 7, pois caso contrrio teramos

que alguma das diferenas acima seria igual a zero, o que impossvel

pois as diferenas rq = 31 7q com 1 q 4 so todas positivas

e com q 5 so todas negativas. Entretanto, notamos que entre as

diferenas positivas a nica que menor que 7 corresponde ao caso

q = 4. O resultado seguinte nos diz o que acontece no caso geral da

diviso de um inteiro b por um inteiro positivo a.


94 3 Divisibilidade

Teorema 3.5 (Diviso Euclidiana) . Dados dois inteiros a e b, sendo


a positivo, existem nicos inteiros q e r tais que

b = aq + r, 0 r < a.

Se a - b, ento r satisfaz a desigualdade estrita 0 < r < a.


Demonstrao. Por simplicidade, suporemos que b positivo. Se b<
a, basta tomar q=0 e r = b.
b = a, ento tomamos q = 1 e
Se r = 0.
Assim, assumiremos tambm que b > a > 0. Consideremos o conjunto

R = {b aq Z; b aq 0} N {0} (3.7)

Notemos que o conjunto R no vazio, pois b a R, j que

b a > 0. Deste modo, pelo princpio da boa ordenao temos que


R admite um menor elemento, que denotaremos por r. Claramente
r = b aq 0, para algum q 0. Alm disso, r < a pois caso
contrrio

r = b aq a b a(q + 1) 0. (3.8)

Por outro lado,

a > 0 b a(q + 1) < b aq. (3.9)

Das desigualdades (3.8) e (3.9) segue que

0 b a(q + 1) < b aq,

contradizendo o fato de que r = baq o menor elemento no negativo


de R.
Agora provaremos que de fato r e q, escolhidos desta forma, so

nicos. Com efeito, suponhamos que existem outros inteiros r1 e q1


tais que

b = aq1 + r1 , 0 r1 < a.
3.1 Conceitos Fundamentais e Diviso Euclidiana 95

Ento resulta que aq + r = aq1 + r1 . Logo,

(r r1 ) = (q1 q)a; (3.10)

sendo assim, rr1 mltiplo de a. Mas, em virtude de a < rr1 < a,


o nico valor que r r1 pode tomar, sendo este mltiplo de a,

r r1 = 0. Portanto, r = r1 , de onde se deduz diretamente de (3.10)


que q = q1 .

Os nmeros q e r no enunciado do teorema acima so chamados,

respectivamente, de quociente e resto da diviso de b por a.


Um resultado imediato da diviso euclidiana o seguinte.

Corolrio 3.6. Dados dois nmeros naturais a e b com 1 < a b,


existe um nmero natural n tal que

na b < (n + 1)a.

Demonstrao. Pela diviso euclidiana, existem nicos q, r N com

0r<a tais que b = aq + r. Assim

aq b = aq + r < aq + a = a(q + 1).

Basta agora tomar q=n para obtermos o resultado.

Os exemplos a seguir apresentam a utilidade do Teorema 3.5.

Exemplo 3.7. Se a um natural com a 3, ento a2 deixa resto 1


na diviso por a 1. Consequentemente, a 1 divide a2 1.

Soluo. Usando a identidade a2 1 = (a 1)(a + 1) temos que


a2 = (a 1)(a + 1) + 1 com 1 < a 1, de onde segue o resultado.
96 3 Divisibilidade

O prximo exemplo, como veremos, motiva a procura de cami-

nhos ecientes para encontrar o resto que deixa um nmero quando

dividido por outro.

Exemplo 3.8. Um turista brasileiro chega a Cuba e troca parte de


seu dinheiro na casa de cmbio, recebendo 175 notas de 50 pesos e
213 notas de 20 pesos. Ele decide trocar este dinheiro pela maior
quantidade possvel das famosas moedas de 3 pesos cubanos, porque
elas tm gravada a imagem do guerrilheiro Che Guevara. Quanto
sobrou do dinheiro depois de fazer a troca pelas moedas?

Soluo. Para resolver este problema basta achar o resto que deixa o

nmero n = 175 50 + 213 20 quando dividido por 3. Entretanto,

queremos destacar que no preciso fazer os produtos e a soma envol-

vidos no nmero n. Em lugar de fazer isto substitumos cada nmero

que aparece em n pelo resto que este deixa na diviso por 3, formando
assim um novo nmero n1 , ou seja,

n1 = 1 2 + 0 2 = 2.

Agora procuramos o resto que n1 deixa na diviso por 3, que obvi-

amente 2. A surpresa que este resto o mesmo que deixa n na

diviso por 3. Logo, sobraram 2 pesos depois de fazer a troca.

A soluo do exemplo anterior uma aplicao particular do se-

guinte lema que de muita utilidade na resoluo de problemas.

Lema 3.9 (Lema dos Restos) . A soma e o produto de quaisquer dois


nmeros naturais deixa o mesmo resto que a soma e o produto dos
seus restos, na diviso por um inteiro positivo a.
3.1 Conceitos Fundamentais e Diviso Euclidiana 97

Demonstrao. Sejam n1 , n2 Z. Fazendo a diviso com resto de

ambos os nmeros por a temos que

n1 = aq1 + r1 e n2 = aq2 + r2 ,

com 0 r1 , r2 < a. Ento,

n1 n2 = (aq1 + r1 )(aq2 + r2 )
= a2 q1 q2 + aq1 r2 + aq2 r1 + r1 r2
(3.11)
= a(aq1 q2 + q1 r2 + q2 r1 ) + r1 r2
= aq + r1 r2 ,

onde q = aq1 q2 +q1 r2 +q2 r1 . Agora dividimos r1 r2 por a para obtermos

r1 r2 = ap + r, p Z, 0 r < a. (3.12)

Das igualdades (3.11) e (3.12) segue que

n1 n2 = aq + ap + r = a(p + q) + r, 0 r < a. (3.13)

Portanto, de (3.12) e (3.13) conclumos que os restos que deixam n1 n2


e r1 r2 na diviso por a so iguais, cando provado o resultado para o

produto. A prova para a soma anloga.

Observao 3.10. A vantagem do lema que em certos problemas


que envolvem nmeros muito grandes podemos substituir estes por n-
meros muito menores e mais confortveis para trabalhar.
Vejamos como aplicar o lema dos restos nos seguintes exemplos a

seguir.

Exemplo 3.11. Prove que o produto de dois nmeros naturais con-


secutivos sempre divisvel por 2.
98 3 Divisibilidade

Soluo. Se n N temos que provar que an = n(n + 1) divisvel por

2. Quando fazemos a diviso de n por 2 temos duas possibilidades

para o resto: r=0 ou r = 1. Analisemos os dois casos por separado.

[r = 0] Neste caso o resto que deixa an na diviso por 2 o

mesmo que o resto que deixa 0(0+1)=0, logo an divisvel por

2.

[r = 1] Neste caso podemos substituir an por 1(1+1)=2 e o

resto que este ltimo deixa quando dividido por 2 0, logo an


tambm divisvel por 2.

Mostraremos agora como utilizar o exemplo anterior pra resolver

um dos problemas da 1a Olimpada Brasileira de Matemtica.

Exemplo 3.12. Prove que se n mpar, ento n2 1 mltiplo de


8.
Soluo. Como n mpar, podemos escrever n = 2m + 1, para algum

k Z. Logo

n2 1 = (2m + 1)2 1 = 4m2 + 4m + 1 1 = 4m2 + 4m.

Assim,

n2 1 = 4m(m + 1).
Observe que de acordo com o exemplo 3.11, m(m + 1) mltiplo de

2. Portanto, m(m + 1) = 2q para algum q Z, de aonde

n2 1 = 4m(m + 1) = 4 2q = 8q,

como queramos demonstrar.


3.2 Bases Numricas 99

Exemplo 3.13. Prove que em qualquer tringulo retngulo com lados


inteiros, pelo menos um deles mltiplo de 3.

Soluo. Comecemos analisando quais so os restos possveis para a

diviso por 3 de um nmero que quadrado. De acordo com o lema

dos restos temos a seguinte tabela para os restos de n e n2 , na diviso

por 3:

n n2
0 0

1 1

2 1

Resumindo, se um nmero no mltiplo de 3 ento o resto da diviso

de seu quadrado por 3 deve ser igual a 1.

Agora denotemos por a e b os catetos e por c a hipotenusa. Supo-

nhamos que nenhum deles divisvel por 3. Ento a2 e b2 deixam resto


1 na diviso por 3. Logo, a2 + b 2 deixa resto 12 + 12 = 2 na diviso
por 3; mas isto uma contradio pois, pelo Teorema de Pitgoras,

a2 + b 2 = c 2 e c2 deixa resto 1 quando dividido por 3.

3.2 Bases Numricas

Comeamos esta seo com uma brincadeira interessante.

Joo, ao sair da aula de matemtica do professor Peitgoras, en-

controu Pedro e lhe props a seguinte brincadeira:

 Pense numa pea de domin, Pedro. Vou adivinhar que pea

essa usando uma frmula mgica.

 Ok, Joo. Pode comear, j pensei.


100 3 Divisibilidade

x y

Figura 3.1: Pea de Domin

- Escolha um dos nmeros na pea e multiplique por 5. Depois

disso some trs a esse resultado. Multiplique agora o nmero que voc

obteve por dois. Some isto com o outro nmero da pea. Qual foi o

resultado?

 Foi 40.

 Ento a pea que voc escolheu foi a 3 com 4!

 Como voc acertou? Me ensina!

Claro que de mgico Joo no tinha nada e decidiu contar seu

segredo a Pedro.

O jogo funciona assim: cada parte da pea de domin pode ser

considerada como um dos dgitos de um nmero de 2 algarismos, o qual

denotamos por n = xy = 10x + y (veja a Figura 3.1). Acompanhando

os passos de Joo, temos que:

(5x + 3)2 + y = 40 10x + y = 34, (3.14)

que claramente, tem por solues: x=3 e y = 4, usando a represen-

tao de 34 na base decimal.

No sistema de numerao decimal, tambm conhecido como sis-

tema numrico na base 10, todo nmero pode ser representado como

uma sequncia de 10 smbolos, constitudos pelo 0 (zero) e os alga-

rismos 1, 2, 3, . . . , 9. Por exemplo, 345 escreve-se na base decimal da


3.2 Bases Numricas 101

seguinte forma

345 = 300 + 40 + 5 = 3 102 + 4 10 + 5,

assim como 2768 se escreve da forma

2768 = 2000 + 700 + 60 + 8 = 2 103 + 7 102 + 6 10 + 8.

De modo geral, se denotamos por a = an an1 . . . a1 a0 o nmero inteiro


positivo formado pelos algarismos an , an1 , . . . , a1 e a0 , nessa ordem,

ento a se escreve na base decimal da forma

a = an 10n + an1 10n1 + . . . + a1 10 + a0 (3.15)

Antes de provar alguns dos critrios de divisibilidade mais po-

pulares do sistema de numerao decimal, provamos uma identidade

muito til.

Lema 3.14. Sejam a, b, n N. Temos que

an bn = (a b)(an1 + an2 b + + abn2 + bn1 ).

Consequentemente, se 0 < b < a, ento a b divide an bn .

Demonstrao. Primeiro provaremos que a propriedade vale para b=


1. Com efeito, considerando a soma geomtrica

s = 1 + a + a2 + + an1

e multiplicando s por a temos que

as = (a + a2 + + an1 ) + an = s 1 + an .
102 3 Divisibilidade

Assim, (a 1)s = as s = an 1, de onde se segue que

an 1 = (a 1)(an1 + an2 + + a + 1). (3.16)

Da temos a validade para b = 1.


an bn = bn ( ab )n 1 .
 
Para bN qualquer, observe que Usando

esta expresso e (3.16) tem-se

an bn = bn ( ab 1) ( ab )n1 + ( ab )n2 + + ( ab ) + 1
 

= (a b)bn1 ( ab )n1 + ( ab )n2 + + ( ab ) + 1


 
(3.17)

= (a b)(an1 + an2 b + + abn2 + bn1 ),


obtendo-se a igualdade clamada.

Proposio 3.15 (Critrios de Divisibilidade) . Seja a = an . . . a1 a0


um inteiro positivo, ento
(a) a divisvel por 10 se, e somente se, a0 for igual a 0;
(b) a divisvel por 3 ou por 9 se, e somente se, a soma dos seus
dgitos divisvel por 3 ou por 9, respectivamente;
(c) a divisvel por 5 se, e somente se, a0 for igual a 0 ou 5.
Demonstrao. Utilizando a representao decimal (3.15) temos que

a = 10(an 10n1 + an1 10n2 + + a1 ) + a0 .

Ento, pela Proposio 3.3-(b) tem-se que 10 | a se, e somente se,

10 | a0 , prondose-se assim o critrio (a).

Para provar (b) observemos que

a = an 10n + an1 10n1 + + 10a1 + a0


= an (10n 1) + an1 (10n1 1) + + (10 1)a1 (3.18)

+ an + an1 + + a1 + a0 .
3.2 Bases Numricas 103

Pelo Lema 3.14 temos que 10j 1 = 9qj para todo 1 j n, da

segue-se

a = 9(an qn + an1 qn1 + + a1 ) + an + an1 + + a1 + a0 .

Ento, aplicando novamente o item (b) da Proposio 3.3 temos que

9|a se, e somente se, 9 | (an + an1 + + a1 + a0 ).

A prova para o caso da divisibilidade por 3 segue de maneira idntica,

logo ca provado o item (b).

A prova do item (c) segue de maneira muito semelhante e deixamos

a mesma a cargo do leitor.

Exemplo 3.16. Prove sem fazer muitas contas que o nmero

N = 13424136 + 1234567890

divisvel por 3.

Soluo. Note que no precisamos fazer a soma dos nmeros ante-

riores. Para mostrar isso, basta aplicar o item (b) da Proposio 3.3 e

o item (b) da Proposio 3.15, observando que cada um dos nmeros

acima divisvel por 3, pois a soma de seus dgitos um mltiplo de

3.

Finalizamos esta seo com uma aplicao da diviso euclidiana

que nos mostra que, analogamente representao decimal, qualquer

nmero admite uma representao nica em qualquer outra base nu-

mrica.
104 3 Divisibilidade

Teorema 3.17 (Bases Numricas) . Dados a, b N, com b > 1, exis-


tem nicos nmeros naturais r0 , r1 , . . . , rn tais que 0 ri b 1,
0 i n, e satisfazendo

a = rn bn + rn1 bn1 + + r1 b + r0 .

A representao acima dita representao de a na base b e usaremos


a notao

a = (rn cn1 . . . r1 r0 )b ,

para fazer referncia a esta.

Demonstrao. Apliquemos sucessivamente a diviso euclidiana como

segue:

a = bq0 + r0 , r0 < b,
q0 = bq1 + r1 , r1 < b,
q1 = bq2 + r2 , r2 < b,
. . . .
. . . .
. . . .

qj1 = bqj + rj , rj < b,

e assim por diante. Como a > q0 > q1 > q2 > > qj1 , para algum
j = n deveremos ter que qn1 < b. Logo, qj = 0 para todo j n,
assim como rj = 0 para todo j n + 1. Das igualdades acima, para
3.2 Bases Numricas 105

1 j n, tem-se

a = bq0 + r0 ,
bq0 = b2 q1 + br1 ,
b2 q1 = b3 q2 + b2 r2 ,
(3.19)
. . .
. . .
. . .

bn1 qn2 = bn qn + bn1 rn1


bn qn1 = bn+1 0 + bn rn .

Efetuando a soma de todas as igualdades em (3.19) obtemos

a = rn bn + rn1 bn1 + + r1 b + r0 .

A unicidade dos nmeros ri vem da unicidade dos restos na diviso

euclidiana.

Observao 3.18. O sistema de numerao na base 2 tambm co-


nhecido como sistema binrio e o sistema habitualmente utilizado no
funcionamento dos computadores.

Exemplo 3.19. Se deseja pesar qualquer nmero inteiro de gramas de


ouro, entre 1g e 100g , numa balana de dois pratos, onde os pesos s
podem ser usados no prato esquerdo da balana. Mostre que a escolha
adequada de 7 pesos diferentes suciente para realizar esta tarefa.

Demonstrao. Usando o sistema de numerao em base 2 temos que

qualquer nmero a tal que 1 a 100 pode ser expressado de forma

nica como

a = r6 26 + r5 25 + r4 24 + r3 23 + r2 22 + r1 2 + r0 1,
106 3 Divisibilidade

com ri {0, 1}, 0 i 1. Observe que 2n 128, com n 7, logo

estas potncias no so consideradas. notemos tambm que o fato de

cada ri ser 0 ou 1 nos diz que no precisamos repetir nenhum dos

pesos na realizao de qualquer pesada. Logo, os pesos

1, 22 , 23 , 24 , 25 , 26

so sucientes para realizar as pesadas de gramas de ouro entre 1g e

100g .

3.3 Mximo Divisor Comum e Mnimo Ml-

tiplo Comum

Nesta seo estudaremos dois conceitos fundamentais, que aparecem

naturalmente em vrios problemas de divisibilidade, assim como a

relao existente entre eles.

3.3.1 Mximo Divisor Comum

O primeiro destes conceitos est relacionado com os inteiros positivos

que dividem simultaneamente a dois inteiros prexados e denomi-

nado mximo divisor comum .


Daqui por diante s consideraremos os divisores positivos dos n-

meros.

Denio 3.20 (Mximo Divisor Comum) . Sejam a e b inteiros dife-


rentes de zero. O mximo divisor comum, resumidamente mdc, entre
a e b o nmero d que satisfaz as seguintes condies:

(a) d um divisor comum de a e b, isto , d | a e d | b;


3.3 Mximo Divisor Comum e Mnimo Mltiplo Comum 107

(b) d o maior inteiro positivo com a propriedade (a).

Neste caso, denotamos o mdc entre a e b por d = mdc(a, b) ou por


d = (a, b). Se (a, b) = 1, ento dizemos que a e b so primos entre si.

Exemplo 3.21. Observando que 12 = 6 2, 18 = 6 3 temos que


mdc.(12, 18) = 6. Por outro lado, mdc. (4, 15) = 1, logo os nmeros 4
e 15 so primos entre si.

Vejamos agora algumas das propriedades mais importantes dos

divisores comuns de dois inteiros.

Proposio 3.22. Sejam a e b dois inteiros. Ento valem as seguintes


armaes.

(a) Se a mltiplo de b, ento (a, b) = b.

(b) Se a = bq + c, c 6= 0, ento o conjunto dos divisores comuns dos


nmeros a e b coincide com o conjunto dos divisores comuns dos
nmeros b e c. Particularmente, (a, b) = (b, c).

Demonstrao. Comeamos com a prova de (a). Com efeito, todo

divisor comum dos nmeros a e b um divisor de b. Reciprocamente,


usando que a mltiplo de b, todo divisor de b tambm um divisor

de a, ou seja, um divisor comum dos nmeros a e b. Portanto, o

conjunto dos divisores comuns dos nmeros a e b igual ao conjunto

dos divisores de b. Como o maior divisor de b ele mesmo, resulta que

(a, b) = b.
Vejamos (b). Usando o item (b) da Proposio 3.3 temos que

todo divisor comum de a e b tambm divide c e, consequentemente,

um divisor deb e c. Pela mesma razo todo divisor comum de b e c


tambm divide a e, consequentemente, um divisor de a e b. Portanto
108 3 Divisibilidade

os divisores comuns de a e b so os mesmos que os divisores comuns

de b e c. Particularmente, tambm coincidem os maiores divisores

comuns, ou seja, (a, b) = (b, c).

O teorema a seguir uma das ferramentas bsicas na resoluo

de problemas que envolvem o mdc entre dois nmeros. O resultado

foi provado pela primeira vez por Claude-Gaspard Bachet de Mziriac

(1581-1638) e mais tarde generalizado para polinmios por tienne

Bzout (1730-1783). Frequentemente, na literatura se enuncia este

resultado como teorema (ou identidade) de Bzout, esquecendo-se o

nome de Bachet.

Teorema 3.23 (Teorema de Bachet-Bzout) . Se d o mdc de a e b,


ento existem nmeros inteiros x0 e y0 tais que d = (a, b) = ax0 + by0 .

Demonstrao. Considere a combinao linear ax + by , onde x e y


percorrem todos os inteiros. Este conjunto de inteiros, denotado por

Ca,b = {ax + by; x, y Z},

inclui valores positivos e negativos. Alm disso, escolhendo x = y = 0,


vemos que Ca,b tambm contm o zero.

Pelo princpio da boa ordenao, podemos escolher x0 e y0 tais que

= ax0 +by0 seja o menor nmero inteiro positivo contido no conjunto


Ca,b .
Agora mostraremos que |a e | b. Provaremos que |a e o

outro segue analogamente. Usaremos para este propsito o mtodo de

reduo ao absurdo, ou seja, vamos supor que -a e obteremos uma

contradio.
3.3 Mximo Divisor Comum e Mnimo Mltiplo Comum 109

Usando a diviso euclidiana, de - a segue que existem inteiros q


e r tais que a = q + r com 0 < r < . Portanto,

r = a q = a q(ax0 + by0 ) = a(1 qx0 ) + b(qy0 )

e assim r est no conjunto Ca,b , o que contradiz a hiptese de ser o

menor elemento positivo contido em Ca,b .


Uma vez que divide a e b s resta provar que = d. Com efeito,

desde que d = (a, b), podemos escrever a = da1 , b = db1 e

= ax0 + by0 = d(a1 x0 + b1 y0 ).

Assim d | . Logo pela parte (c) da Proposio 3.3, conclumos que

d . Agora d < impossvel pois d = mdc(a, b), e portanto

d = = ax0 + by0 .

A seguinte proposio resume algumas consequncias importantes

da demonstrao dada ao teorema de Bzout.

Proposio 3.24. Sejam d, N e a, b, c Z. Ento valem as


seguintes armaes:

(a) Se d | a e d | b, ento d | (a, b).

(b) O mdc.(a, b) o menor valor positivo de ax + by , onde x e y


percorrem todos os nmeros inteiros.

(c) (a, b) = (a, b).

(d) Se d | a e d | b, ento ( ad , db ) = d1 (a, b). Consequentemente,


 
a b
, = 1.
(a, b) (a, b)
110 3 Divisibilidade

(e) Se (a, c) = (b, c) = 1, ento (ab, c) = 1.

(f) Se c | ab e (b, c) = 1, ento c | a.

Demonstrao. A prova de (a) consequncia imediata da igualdade

(a, b) = ax0 + by0 anunciada no teorema de Bzout; assim como (b)

segue diretamente da demonstrao dada a este teorema.

Para provar (c), primeiro observamos que

(a)x + (b)y = (ax + by) onde x, y Z.

Usando o item (a) e o fato de ser positivo, da igualdade acima segue


que


(a, b) = min (a)x + (b)y > 0; x, y Z

= min ax + by ; x, y Z
= (a, b).

A armao feita em (d) segue diretamente de (c), observando que

   
a b a b
(a, b) = d , d =d , .
d d d d

Continuamos com a prova de (e). De (a, c) = (b, c) = 1, temos que

existem inteiros xj e yj , j = 1, 2, tais que

ax1 + cy1 = 1,
bx2 + cy2 = 1.

Multiplicando lado a lado as igualdades obtemos

(x1 x2 )ab + (ax1 y2 + y1 bx2 + cy1 y2 )c = 1.


|{z} | {z }
x y
3.3 Mximo Divisor Comum e Mnimo Mltiplo Comum 111

Ento, usando o item (b) e a igualdade acima resulta que (ab, c) = 1.


Finalmente, provaremos (f ). Das hipteses temos que existem in-

teiros x0 e y0 tais que

bx0 + cy0 = 1.

Multiplicamos a igualdade acima por a em ambos lados para obtermos

abx0 + acy0 = a.

Por outro lado, ab = cq para algum inteiro q. Usando esta condio

na ltima igualdade temos que

cqx0 + acy0 = c(qx0 + ay0 ) = a,

logo c | a.

3.3.2 Algoritmo de Euclides

Apesar de conhecermos propriedades tericas do mdc entre dois intei-

ros, encontr-lo de fato pode ser uma tarefa complicada, sem auxlio

das ferramentas corretas. Lembrando o seu signicado, o leitor talvez

pudesse pensar que devemos calcular todos os divisores de a, todos

os divisores de b e descobrir qual o maior elemento comum aos dois

conjuntos.

Para achar o mdc se faz uso de um importante mtodo denominado

algoritmo de Euclides .

Teorema 3.25 (Algoritmo de Euclides) . Dados dois inteiros positivos,


a e b, aplicamos sucessivamente a diviso euclidiana para obter a se-
112 3 Divisibilidade

guinte sequncia de igualdades





b = aq1 + r1 , 0 r1 < a,





a = r1 q2 + r2 , 0 r2 < r1 ,

r = r q + r , 0 r3 < r2 ,

1 2 3 3
(3.20)





rn2 = rn1 qn + rn , 0 rn < rn1 ,






r
n1 = rn qn+1 ,

at algum rn dividir rn1 . Assim, o mdc.(a, b) = rn , ou seja, o


ltimo resto no-nulo no processo de diviso anterior.

Observao 3.26. Quando lidamos com nmeros pequenos achar o


mdc uma tarefa fcil pois podemos calcular o mdc valendo-nos das
fatoraes dos nmeros envolvidos. No entanto, quando estamos tra-
balhando com nmeros grandes o algoritmo de Euclides, em geral,
mais fcil que a fatorao, podendo ser esta ltima bem difcil.

Demonstrao do algoritmo de Euclides. Comeamos observando que

o processo de diviso (3.20) nito. Com efeito, a sequncia de nme-

rk estritamente decrescente e est contida no conjunto


ros inteiros

{r Z, 0 r < a}, portanto no pode conter mais do que a intei-


ros positivos. Examinando as igualdades (3.20) de cima para baixo e

usando a Proposio 3.22 temos que

(a, b) = (a, r1 ) = (r1 , r2 ) = = (rn1 , rn ) = rn .


3.3 Mximo Divisor Comum e Mnimo Mltiplo Comum 113

Observao 3.27. Notemos que o teorema de Bzout tambm pode


ser obtido como consequncia do processo de diviso (3.20). Com
efeito, podemos escrever

rn = rn2 rn1 qn o
rn = rn2 (rn3 rn2 qn1 )qn .
rn1 = rn3 rn2 qn1

Logo, conseguimos escrever rn em termos de rn2 e rn3 . Utilizando a


expresso rn2 = rn4 rn3 qn2 podemos escrever rn como combina-
o de rn3 e rn4 . Repetindo este processo vrias vezes, conclumos
que existem x, y Z tais que

d = rn = xr1 + yr2 .

Ora, como r1 = b aq1 e r2 = a r1 q2 = a(1 + q1 q2 ) bq2 , ento,


substituindo estes valores na ltima igualdade obtemos o Teorema de
Bzout.

Exemplo 3.28. Achar o mximo divisor comum dos nmeros 471 e


1.176.

Soluo. Aplicando o algoritmo de Euclides obtemos a seguinte sequn-

cia de divises com resto:

1176 = 471 2 + 234,


471 = 234 2 + 3,
234 = 78 3,

ento o mdc(471, 1176) = 3.


2n + 8
Exemplo 3.29. Provar que a frao irredutvel para todo
4n + 15
nmero natural n.
114 3 Divisibilidade

Soluo. Usando o algoritmo de Euclides temos que

4n + 15 = (2n + 8) 1 + 2n + 7,
2n + 8 = (2n + 7) 1 + 1,
2n + 7 = (2n + 7) 1.

Ento o mdc. (4n + 15, 2n + 8) = 1 e portanto 4n + 15 e 2n + 8 so

primos entre si para qualquer valor de n.

Exemplo 3.30. Achar o mdc.(111


| .{z
. . 111}, 11
| .{z
. . 11})
100 vezes 60 vezes

Soluo. Primeiro escrevemos os nmeros na base decimal, isto ,

111 . . 111} = 1099 + 1098 + + 1,


| .{z
100 vezes

11 . . 11} = 1059 + 1058 + + 1.


| .{z
60 vezes

Aplicamos agora o algoritmo de Euclides para obter as seguintes igual-

dades

111 . . 111} = (1059 + 1058 + + 1)1040 + 1039 + 1038 + + 1,


| .{z
100 vezes

1059 + 1058 + + 1 = (1039 + 1038 + + 1)1020 +


+ 1019 + 1018 + + 1,
1039 + 1038 + + 1 = (1019 + 1018 + + 1)1020 +
+ 1019 + 1018 + + 1.

Disso resulta que

19 18
| {z } | {z } = 10 + 10 + + 1 = |11 .{z
mdc.(111 . . . 111, 11 . . . 11) . . 11} .
100 vezes 60 vezes 20 vezes
3.3 Mximo Divisor Comum e Mnimo Mltiplo Comum 115

3.3.3 Mnimo Mltiplo Comum

Agora passamos ao segundo conceito importante desta seo. O mesmo

est relacionado com os inteiros positivos que so simultaneamente

mltiplos de dois inteiros prexados e denominado mnimo mltiplo


comum.

Denio 3.31 (Mnimo Mltiplo Comum) . Sejam a e b inteiros


diferentes de zero. O mnimo mltiplo comum, resumidamente mmc,
entre a e b o inteiro positivo m que satisfaz as seguintes condies:

(a) m um mltiplo comum de a e b, isto , a | m e b | m;

(b) m o menor inteiro positivo com a propriedade (a).

Neste caso, denotamos o mmc entre a e b por m = mmc(a, b) ou por


m = [a, b].

Resumimos a seguir algumas das propriedades fundamentais do

mmc de dois inteiros.

Proposio 3.32. Sejam a, b, c Z e Z. Ento valem as se-


guintes armaes:

(a) se c mltiplo comum de a e b, ento [a, b] | c;

(b) [a, b] = [a, b];

(c) |ab| = [a, b] (a, b).

Demonstrao. Comeamos com a prova de (a). A diviso com resto

de c por [a, b] nos d

c = [a, b]q + r, 0 r < [a, b]. (3.21)


116 3 Divisibilidade

Da igualdade anterior, basta provar que r = 0 para obter o resultado

desejado. Suponhamos, pelo contrrio, que 0 < r < [a, b]. Notemos

que tanto a como b dividem c e [a, b]. Logo, pelo item (b) da Pro-

posio 3.3 e a igualdade (3.21), temos que a e b tambm dividem r,


ou seja, r mltiplo comum de a e b e no pode ser menor que [a, b],
contradizendo nossa suposio.

Prosseguimos com a prova de (b). Observemos que [a, b] mlti-

plo comum de a e b, logo pelo item (i) vale que

[a, b] [a, b]. (3.22)

Por outro lado, [a, b] = q1 a = q2 b, para alguns inteiros q1 e q2 ;


[a,b]
logo, um mltiplo comum de a e b. Portanto,

[a, b]
[a, b] [a, b] [a, b]. (3.23)

Das igualdades (3.22) e (3.23) segue que

[a, b] [a, b] [a, b],

de onde vem diretamente o resultado.

Para provar (c) podemos supor sem perda de generalidade que a e

b so positivos devido s igualdades

[a, b] = [a, b] = [a, b] = [a, b].

Dividiremos a prova em dois casos:

Caso 1: (a, b) = 1.
Sabemos que b | [a, b] e [a, b] = qa, para algum q N. Ento b | qa
e alm disso (a, b) = 1. Logo, pelo item (v) da Proposio 3.24 temos

que b | q . Portanto, b q e consequentemente

ab aq = [a, b]. (3.24)


3.3 Mximo Divisor Comum e Mnimo Mltiplo Comum 117

Entretanto, da denio de [a, b] vale que

[a, b] ab. (3.25)

Das desigualdades (3.24) e (3.25) segue que ab [a, b] ab. Assim,

ab = [a, b] = [a, b] 1 = [a, b] (a, b).

Caso 2: (a, b) > 1.  


a
Da parte (c) da Proposio 3.24 sabemos que , b
(a,b) (a,b)
= 1.
Aplicando o caso anterior vale que
   
a b a b a b
= , , .
(a, b) (a, b) (a, b) (a, b) (a, b) (a, b)
Multiplicamos esta ltima igualdade por (a, b)2 e usamos o item (b)

provado anteriormente, assim como a parte (d) da Proposio 3.24

para obter
   
a b a b
ab = (a, b) , (a, b) , = [a, b] (a, b).
(a, b) (a, b) (a, b) (a, b)

Exemplo 3.33. Dois amigos passeiam de bicicleta, na mesma dire-


o, em torno a uma pista circular. Para dar uma volta completa um
deles demora 15 minutos e o outro demora 18 minutos. Eles partem
juntos e combinam interromper o passeio quando os dois se encontra-
rem pela primeira vez no ponto de partida. Quantas voltas deu cada
um?
Soluo. Denotemos por n1 e n2 , respectivamente, o nmero de voltas
que d cada um dos amigos. Notemos que o tempo total da corrida

o menor valor positivo de T que satisfaz as igualdades

T = 15n1 = 18n2 ,
118 3 Divisibilidade

ou seja
15 18
T = [15, 18] = = 90.
3
Portanto, n1 = 6 e n2 = 5.

Finalizamos esta seo com um exemplo que nos fornece uma bela

interpretao geomtrica do mnimo mltiplo comum. O mesmo foi

proposto na Olimpada Brasileira de Matemtica.

Exemplo 3.34. Um retngulo de lados inteiros AB = m e CD = n,


dividido em quadrados de lado 1. Em cada um dos vrtices ele possui
um pequeno orifcio. Um raio de luz entra no retngulo por um dos
vrtices, na direo da bissetriz do ngulo reto, e reetido sucessi-
vamente nos lados do retngulo. Quantos quadrados so atravessados
pelo raio de luz?

D C

A B

Figura 3.2: Interpretao geomtrica do mmc

Soluo. Se zermos alguns testes preliminares dando valores a me


n, veremos que em cada caso a resposta coincidir com o mmc( m,n).

Provemos que isto de fato vale para m e n quaisquer. Para realizar a

prova nos auxiliaremos da Figura 3.2.


3.3 Mximo Divisor Comum e Mnimo Mltiplo Comum 119

Primeiramente, notemos que cada vez que o raio de luz atravessa

um quadrado ele avana uma unidade tanto na direo horizontal como

na direo vertical. Usando este fato fazemos as observaes a seguir.

Se o raio entra pelo vrtice A, ter que atravessar m quadrados

at chegar ao ladoBC , imediatamente mais m para chegar ao


lado AD , depois mais m para chegar novamente ao lado BC , e

assim sucessivamente. Alm disso, depois do raio percorrer pm

quadrados, com p N, estar batendo no lado BC ou no lado

AD.

Analogamente o raio bater no lado AB ou no lado DC se, e

somente se, atravessar qn quadrados, com q N.

Somente nos vrtices B, C e D do retngulo pode acontecer que

o raio incidente saia do retngulo, terminando assim o processo

de reexo.

Usando as observaes acima fcil ver que o raio chegar a um

vrtice quando chegar simultaneamente a dois lados perpendiculares

do retngulo. Portanto, deve ter atravessado um nmero x de quadra-


dos tal que x = pm = qn, ou seja, x dever ser um mltiplo comum

de m
n. claro que a primeira vez que o raio chega a um vrtice
e o

nmero x o menor mltiplo comum de m e n, isto , x = [m, n].

Finalmente, observamos que nenhum dos quadrados atravessado

duas vezes no percurso do raio de A at bater no primeiro vrtice, pois


como vemos na gura numa das direes os quadrados atravessados

sero todos cinzas e na outra direo, sero todos brancos.


120 3 Divisibilidade

3.3.4 Equaes Diofantinas Lineares

Consideremos a equao

ax + by = c, (3.26)

onde a, b, c Z, com a 6= 0 e b 6= 0.
A equao (3.26) chamada de equao diofantina linear e uma

soluo desta qualquer par de inteiros (x, y) que satisfaam (3.26).

conhecido que todos os pontos do plano, com coordenadas (x, y),


que satisfazem a igualdade (3.26) representam, geometricamente, uma

reta. Logo, as solues de uma equao diofantina linear so os pontos

de coordenadas inteiras do plano cartesiano, que esto dispostos sobre

a reta que esta representa. Por exemplo, os pontos (1, 2) e (1, 1)


so solues da equao diofantina 3x 2y = 1, veja a Figura 3.3.
3
y
2

1

0

x
-1

-2

-3
-3 -2 -1 0 1 2

Figura 3.3: A equao da reta ` 3x 2y = 1.

Naturalmente nos perguntamos: sempre possvel achar solues

para uma equao diofantina linear? A resposta no; o prximo

resultado nos diz quando isto possvel. Alm disso, se uma equao

diofantina linear tem uma soluo na verdade ela tem uma innidade

de solues.
3.3 Mximo Divisor Comum e Mnimo Mltiplo Comum 121

Proposio 3.35. A equao diofantina linear


ax + by = c, a, b, c Z, com a 6= 0 e b 6= 0, (3.27)

tem soluo se, e somente se, d | c, onde d = (a, b). Alm disso,
se (x0 , y0 ) uma soluo, ento o conjunto de solues de (3.27)
constitudo por todos os pares de inteiros (x, y) da forma:
x = x0 + t db e y = y0 t ad , t Z. (3.28)

Demonstrao. Primeiramente suponhamos que (x0 , y0 ) uma soluo


de (3.27), logo ax0 + by0 = c. Usando que d = (a, b) sabemos que
existem inteiros q1 e q2 , tais que dq1 = a e dq2 = b. Portanto, se
verica a igualdade

dq1 x0 + dq2 y0 = d(q1 x0 + q2 y0 ) = c,

de onde segue obviamente que d | c.


Reciprocamente, suponhamos que d|c e portanto c = qd com q
inteiro. O teorema de Bzout nos garante a existncia de dois inteiros,

x0 e y0 , ax0 + by0 = d. Multiplicando


tais que ambos os lados desta

ltima igualdade por q temos que

ax0 q + by0 q = c,

logo o par (x1 , y1 ), com x1 = x 0 q e y1 = y0 q , soluo da equao

diofantina.

Resta provar agora que temos innitas solues da forma (3.28).

Com efeito, sendo (x, y) uma outra soluo qualquer alm de (x0 , y0 ),
vale que ax0 + by0 = c = ax + by , de onde ax0 + by0 = ax + by . Desta

igualdade obtemos a(x x0 ) = b(y0 y) e dividimos esta ltima por

d para obtermos
a b
(x x0 ) = (y0 y).
d d
122 3 Divisibilidade

Como ( ad , db ) = 1, ento temos que


a
d
| (y0 y) e
b
d
| (x x0 ). Logo,

existe inteiro t tal que

x = x0 + t db e y = y0 t ad .

Por outro lado, fcil vericar que para qualquer inteiro t as expresses
achadas acima para x e y resolvem a equao diofantina.

A seguir damos um exemplo de como proceder para resolver equa-

es diofantinas.

Exemplo 3.36. Achar todas as solues inteiras da equao


12x + 33y = 27.

Soluo. Observemos que (12, 33) = 3 e que 3 | 27, logo a equa-

o tem innitas solues. Como sabemos, basta achar uma delas

e teremos as restantes. Para achar esta soluo particular podemos

trabalhar de duas maneiras, que descrevemos a seguir:

Alternativa 1: reduzimos a equao forma equivalente

4x + 11y = 9,

e por tentativa e erro vemos que x0 = 5 e y0 = 1 solucionam a

mesma. Ento pela Proposio 3.35 temos que

x = 5 + 11t e y = 4t 1, t Z,

esgotam todas as solues que procuramos.

Alternativa 2: aplicamos o algoritmo de Euclides para achar o

mdc (12, 33), obtendo os seguintes resultados:

33 = 12 2 + 9,
12 = 9 1 + 3,
9 = 3 3 + 0.
3.4 Nmeros Primos e Compostos 123

Da segunda e primeira igualdades temos, respectivamente, que

3 = 12 9 1 e 9 = 33 12 2.

Usando estas duas obtemos

3 = 12 (33 12 2) 1
= 12 33 + 12 2
= 3 12 1 33,

ou seja, achamos x0 = 3 e y0 = 1, garantidos pelo teorema de


Bzout, que validam 3 = 12x0 +33y0 . Multiplicamos por 9 esta ltima

igualdade para obter

27 = 12(9x0 ) + 33(9y0 ).

Portanto, x
e0 = 9x0 = 27 e ye0 = 9y0 = 9 resolvem, particularmente,
a equao diofantina. Analogamente, como na alternativa anterior,

podemos escrever a soluo geral da forma:

x = 27 + 11s e y = 4s 9, s Z.

3.4 Nmeros Primos e Compostos

Ao longo da histria da Matemtica, os nmeros primos foram pro-

tagonistas de clebres problemas que motivaram o desenvolvimento

de teorias e tcnicas pelas mentes mais frteis, como Fermat, Euler e

Gauss. At hoje muitos desses problemas, simples de enunciar, que

envolvem nmeros primos so desaos intelectuais para toda a huma-

nidade.
124 3 Divisibilidade

Esta seo ser dedicada ao estudo de propriedades bsicas dos

nmeros primos. Todo nmero natural n maior do que 1 tem pelo

menos 2 divisores, claramente 1 e n. Isto motiva a seguinte denio.

Denio 3.37 (Nmeros Primos e Compostos) . Um inteiro positivo


n 2 dito primo se os nicos divisores que ele tem so 1 e ele
prprio; caso contrrio, dito composto.

Observao 3.38. De modo geral o nmero 1 no considerado nem


primo nem composto.

Exemplo 3.39. Os nmeros 2, 3, 5, 7, e 11 so primos e os nmeros


10, 15, 35 e 348 so compostos.

Exemplo 3.40. O nmero n = 220 254 composto.


Soluo. Escrevemos n de outra forma, com o objetivo de facilitar

nosso trabalho. Com efeito, observemos que

n = (210 )2 (252 )2 = 10242 6252 ,

logo composto por ser diferena de quadrados. Alm disso,

n = 10242 6252 ,
= (1024 625)(1024 + 625),
(3.29)
= 399 1649,
= 3 133 1649.

Portanto, podemos concluir que 3 | n.

Proposio 3.41. Seja n > 1 um nmero inteiro. Ento


(a) o menor divisor de n diferente de 1 um nmero primo;
3.4 Nmeros Primos e Compostos 125

(b) se n composto, o seu menor divisor diferente de 1 no maior



que n. Em outras palavras, se n no possui divisores diferentes

de 1, menores ou igual que n, ento n primo.

Demonstrao. Comeamos provando (a). Seja p o menor divisor de

n, diferente de 1. Se p fosse composto teria algum divisor q tal que

1 < q < p; mas


q|p e p | n,
o que nos diz que q | n, e isto contradiz a hiptese levantada sobre p.
Para provar (b) denotamos por p o menor divisor de n, diferente

de 1. Portanto, n = pq com q p. Multiplicando ambos lados da

desigualdade por p obtemos

n = pq p2 ,

e consequentemente vale n p.

Agora vamos enunciar um dos resultados mais clssicos da Mate-

mtica, que garante a existncia de innitos nmeros primos. At

onde se conhece, a demonstrao a seguir foi a primeira demonstrao

escrita utilizando o mtodo de reduo ao absurdo e devida a Eu-

clides cerca de 300 a.C. Para outras seis provas, incluindo a moderna

prova de Fustenberg, recomendamos os livros [1] e [10].

Teorema 3.42 (Teorema de Euclides). A quantidade de nmeros pri-


mos innita.

Demonstrao. Faremos a prova por reduo ao absurdo. Suponha

que existe uma quantidade nita de nmeros primos e denotemos estes

por

p1 , p2 , p3 , . . . , pk .
126 3 Divisibilidade

Consideremos o nmero

n = p1 p2 p 3 pk + 1

e chamemos de q o seu menor divisor primo. Obviamente q no coin-

cide com nenhum dos nmeros pi , 1 i k , pois caso contrrio,

como ele divide n, teria que dividir 1, o que impossvel. Logo, te-

mos uma contradio hiptese de termos uma quantidade nita de

primos.

Os nmeros primos tambm podem ser caracterizados da seguinte

maneira:

Proposio 3.43. Um inteiro positivo p primo se, e somente se,


satisfaz a seguinte propriedade:

p | ab = p | a ou p|b (3.30)

onde a, b Z.

Demonstrao. Primeiramente, suponhamos que p primo e que p - b,


logo (p, b) = 1. Ento, pelo item (f ) da Proposio 3.24 temos que

p | a.
Reciprocamente, suponhamos que, a propriedade 3.30 vlida e

alm disso vamos supor, pelo absurdo, que p no primo. Ento,

p = d1 d2 , com 1 < d1 < p, 1 < d2 < p. (3.31)

De (3.30) segue que p | d1 ou p | d2 ; consequentemente

p d1 , ou p d2 , (3.32)

contradizendo isto o armado em (3.31).


3.5 Procurando Primos 127

3.5 Procurando Primos

Os nmeros primos alm de belos e desaadores do ponto de vista

matemtico, so extremamente importantes para as atividades usuais

de nosso dia a dia. Por exemplo, nenhuma transao bancria ou pela

internet estaria segura sem o uso de nmeros primos muito grandes.

Assim, surge naturalmente a pergunta de como podemos produzi-los

em grandes quantidades. Essa pergunta sempre intrigou os matem-

ticos e continua sem soluo at os dias atuais. Apesar deles serem

abundantes, em quantidade innita de acordo com o Teorema 3.42,

no existe nenhum mtodo razovel de produo de nmeros primos,

mesmo tendo em mos a alta tecnologia de hoje em dia. Porm, ao

longo do tempo algumas frmulas e algoritmos se mostraram teis

para a descoberta de nmeros primos.

3.5.1 O Crivo de Eratstenes

O crivo de Eratstenes um algoritmo que nos permite achar todos

os nmeros primos que so menores ou iguais que um natural N dado.

Segundo a tradio, este mtodo foi criado pelo matemtico grego

Eratstenes (285-194 a.C.).

O mtodo consiste nos seguintes passos: escrevemos os nmeros de

forma ordenada a partir de 2, isto ,

2, 3, 4, 5, 6, 7, 8, 9, 10, 11, 12, 13, 14, 15, 16, 17, . . . , n (3.33)

Observamos que o primeiro primo que aparece em (3.33) 2 e

imediatamente apagamos da lista (3.33) todos os mltiplos de

2 maiores que ele, por serem compostos; resta assim a seguinte


128 3 Divisibilidade

lista

2, 3, 5, 7, 9, 11, 13, 15, 17 . . .

O primeiro nmero no apagado que aparece na lista restante

3, que tambm primo. Imediatamente apagamos da lista todos

os mltiplos de 3 maiores que ele, por serem compostos; resta

agora a lista

2, 3, 5, 7, 11, 13, 17, . . .

O primeiro nmero no apagado que aparece na lista que restou

do passo anterior 5, que tambm primo. Imediatamente

apagamos da lista todos os mltiplos de 5 maiores que ele, por

serem compostos.

Repetimos este processo at que o primeiro nmero no apagado



da lista em questo seja maior que n, pois graas Proposio
3.41-(b) a partir desse momento todos os nmeros restantes so

os primos menores ou iguais que n..



Por exemplo, se n = 40, temos que 40 = 6, 324555. Ento,

aplicando o mtodo:

2 3 4 5 6 7 8 9 10

11 12 13 14 15 16 17 18 19 20

21 22 23 24 25 26 27 28 29 30

31 32 33 34 35 36 37 38 39 40

Passo 1: ordenamos os nmeros

2 3 5 7 9

11 13 15 17 19

21 23 25 27 29

31 33 35 37 39
3.5 Procurando Primos 129

Passo 2: tiramos os mltiplos de 2

2 3 5 7

11 13 17 19

23 25 29

31 35 37

Passo 3: tiramos os mltiplos de 3

2 3 5 7

11 13 17 19

23 29

31 37

Passo 4: tiramos os mltiplos de 5

Como 72 = 49 > 40, paramos agora.

Observao 3.44. Note que ao comear a apagar os mltiplos de um


nmero primo p podemos comear a apagar a partir de p2 , pois se
supomos que existe um nmero composto m no apagado menor que
p2 , temos que m = p1 q1 , sendo p1 seu menor divisor primo. Ento,

pelo item (b) da Proposio 3.41, p1 < m < p, logo m deveria ter
sido apagado pois mltiplo de um primo menor que p.

3.5.2 Primos de Mersenne

Marin Mersenne (1588-1648) foi um monge francs que nasceu na ci-

dade de Maine e foi um dos grandes inuenciadores da Matemtica


130 3 Divisibilidade

2 3 5 7 11 13 17 19 23 29

31 37 41 43 47 53 59 61 67 71

73 79 83 89 97 101 103 107 109 113

127 131 137 139 149 151 157 163 167 173

179 181 191 193 197 199 211 223 227 229

233 239 241 251 257 263 269 271 277 281

283 293 307 311 313 317 331 337 347 349

353 359 367 373 379 383 389 397 401 409

419 421 431 433 439 443 449 457 461 463

467 479 487 491 499 503 509 521 523 541

Tabela 3.1: Os primeiros 100 nmeros primos

francesa nos sculos XVI e XVII. Apaixonado pelos nmeros, teve en-

tre seus correspondentes Descartes, Fermat, Pascal e Galileu. Entre

suas vrias descobertas, ele estudou os nmeros da forma:

Mn = 2n 1.

Observe que vale o seguinte fato a respeito desses nmeros:

Proposio 3.45. Se Mn primo, ento n primo.


Demonstrao. Provar essa proposio equivale a mostrar que a sua

forma contrarrecproca vale. Ou seja, que se n composto, digamos

n = a.b, com a b > 1, ento Mn tambm composto. De fato,

usando o Lema 3.14, podemos decomp-lo do seguinte modo:

Ma.b = 2ab 1 = 2a(b1) 2a(b2) + + 2a + 1 2b 1 .


 
3.5 Procurando Primos 131

Porm, no verdade a recproca da armao acima. Por exem-

plo, Hudalricus Regius mostrou em 1536 que M11 = 211 1 = 2.047


no primo, j que 2.047 = 23 89.
Em 1643, Mersenne armou que para

n = 2, 3, 5, 7, 13, 17, 19, 31, 67, 127 e 257,

os valores de Mn so todos primos e para todos os outros valores de n


menores que 257, Mn composto.
Hoje sabemos que Mersenne errou na sua armao, esquecendo

trs valores de n onde Mn primo: 61, 89 e 107 e incluindo M67 e M257


como nmeros primos. Para mais informaes, sugerimos a pgina web
http://primes.utm.edu/mersenne/index.html.

Finalizamos esta seo, com um critrio interessante, devido ma-

temtica francesa Sophie Germain (1776-1831), que nos permite saber

quando um nmero no primo.

Proposio 3.46 (Identidade de Sophie Germain) . Dados a, b R,


vale a igualdade

a4 + 4b4 = (a2 + 2b2 + 2ab)(a2 + 2b2 2ab).

Demonstrao. A prova segue das seguintes igualdades:

a4 + 4b4 = a4 + 4a2 b2 + 4b4 4a2 b2


= (a2 + 2b2 )2 4a2 b2
= (a2 + 2b2 + 2ab)(a2 + 2b2 2ab).

Como aplicao desta identidade vejamos os seguintes exemplos.


132 3 Divisibilidade

Exemplo 3.47. qn = n4 + 4n composto, para todo n N.


Soluo. O conjunto dos nmeros naturais particionado em duas

classes disjuntas:o conjunto dos nmeros pares e o conjunto dos n-

meros mpares. Estudaremos cada classe por separado. Assim,

sen um nmero par, ento n = 2m para algum inteiro positivo


m 1. Deste modo,

n4 + 4n = (2m)4 + 42m = 16m4 + 24m ,


= 2 8m4 + 24m1 .


Portanto, neste caso, n4 + 4n 2. Logo, se n>1 qualquer


4 n
nmero inteiro positivo par temos que n +4 no um nmero

primo;

se n um nmero mpar, ento n = 2m + 1 para algum inteiro

positivo m 1. Assim,

n4 + 4n = (2m + 1)4 + 42m+1 = (2m + 1)4 + 4 42m


= (2m + 1)4 + 4 24m = (2m + 1)4 + 4 (2m )4 .

Logo, tomando a = 2m + 1 e b = 2m , o resultado uma con-

sequncia direta da identidade de Sophie Germain.

Exemplo 3.48. 520 + 230 um nmero composto.


Soluo. Escrevemos

4 4
520 + 230 = 554 + 22 228 = 55 + 4 27 ,

de onde podemos usar a Identidade de Sophie Germain com a = 55 e


7 20 30
b=2 para comprovar que o nmero 5 +2 composto.
3.5 Procurando Primos 133

3.5.3 O Teorema Fundamental da Aritmtica

Os nmeros primos so as clulas dos nmeros naturais, no sentido

de que qualquer nmero natural produto de nmeros primos. Por

exemplo,

560 = 56 10 = 7 8 5 2 = 7 2 2 2 5 2,

onde cada um dos fatores que aparecem no produto so nmeros pri-

mos. Perguntamo-nos, o que acontece se comeamos com uma outra

fatorao inicial de 560, por exemplo, 560 = 28 20. Vejamos:

560 = 28 20 = 14 2 10 2 = 7 2 2 5 2 2.

Surpreendentemente chegamos mesma representao anterior, salvo

a ordem dos fatores.

2 7
5
2

Figura 3.4: O nmero 560 composto de 4 clulas do tipo 2, uma clula

do tipo 7 e uma clula do tipo 5.

O fato observado acima vale para qualquer nmero natural maior

que 1. Especicamente, temos o seguinte resultado conhecido como

teorema fundamental da aritmtica .


134 3 Divisibilidade

Teorema 3.49 (Teorema Fundamental da Aritmtica) . Todo nmero


natural n maior que 1 pode ser escrito como um produto

n = p1 1 p2 2 p3 3 pmm , (3.34)

onde m 1 um nmero natural, i N e pi primo para todo


1 i m . Alm disso, a fatorao em (3.34) nica se exigirmos
que p1 < p2 < < pm .
Demonstrao. Seja n um inteiro maior que 1. Denotando por p1 seu

menor divisor primo tem-se que

n = p1 1 , 1 1 < n.

Se 1 = 1, entoN1 = p1 e a fatorao desejada obtida. Caso


contrrio, denotando por p2 o menor divisor primo de 1 tem-se que

n = p1 p2 2 , 1 2 < 1 .

Se2 = 1, ento n = p1 p2 e novamente chegamos fatorao desejada.


Caso contrrio, denotando por p3 o menor divisor primo de 2 tem-se

que

n = p1 p2 p3 3 , 1 3 < 2 .
Continuando este processo sucessivamente obtemos ento uma sequn-

cia estritamente decrescente de nmeros naturais n , ou seja,

n > 1 > 2 > 2 > > n > n+1 > 1,

Ento, pelo princpio da boa ordem, s pode existir uma quantidade

nita de ndices n tais que n > 1 e consequentemente n+1 = 1, de

onde segue que

n = p1 p2 pn .
3.5 Procurando Primos 135

Notemos que na representao acima os pi podem-se repetir, resul-

tando nalmente a representao desejada em (3.34).

Provaremos agora a unicidade de tal fatorao. Com efeito, supo-

nha que existem duas fatoraes:

p1 1 p2 2 p3 3 pmm = n = q11 q22 q33 qss

Pela Proposio 3.43 temos que cada pi divide algum qj , logo pi =


qj , por serem primos. Portanto, cada pi aparece no lado direito da

igualdade acima, e, um argumento anlogo nos d que cada qj tambm


aparece no lado esquerdo da igualdade. Ento, como os pi s e os qj s

so diferentes dois a dois e organizados crescentemente, temos m = s

e a igualdade se reduz a

p1 1 p2 2 p3 3 pmm = p1 1 p2 2 p3 3 pmm .

Suponhamos agora que 1 seja diferente de 1 ; sem perda de ge-

neralidade vamos supor que 1 < 1 . Portanto,

p2 2 p3 3 pmm = p1 1 1 p2 2 p3 3 pmm ,

e como 1 1 > 0 ento, pela Proposio 3.43 temos que p1 di-


vide algum pj , com j > 1, o que impossvel. Portanto, 1 = 1 .

Similarmente provamos que i = i , com i = 1, . . . , n.

Observao 3.50. O teorema fundamental da aritmtica foi enun-


ciado precisamente por Gauss (1777-1855). Seus antecessores, Fer-
mat, Euler, Lagrange e Legendre, utilizavam este teorema sem a preo-
cupao de t-lo enunciado ou demonstrado com preciso. Uma prova
alternativa deste teorema ser apresentada no Captulo 6, usando o
mtodo de induo.
136 3 Divisibilidade

Exemplo 3.51. Prove que um nmero n par se, e somente se, o


nmero 2 aparece na fatorao de n em fatores primos.
Soluo. Obviamente, se 2 aparece na fatorao em primos de N,
ento N par. Ora, se n par temos que n = 2q . Por outro lado qe
n se fatoram, respectivamente, como

q = q11 q22 qm
m
e n = p1 1 p2 2 ps s .
Logo,

2 q11 q22 qm
m
= p1 1 p2 2 ps s .
Pela unicidade da fatorao, para algum i, com 1 i s, o cor-

respondente pi deve ser igual a 2. Portanto, 2 aparece na fatorao de

n.
Exemplo 3.52. Seja A = {1, 2, 3, 4, 5, 6, 7}. possvel decompor
o conjunto A em dois subconjuntos disjuntos tais que o produto dos
elementos de um seja igual ao produto dos elementos do outro?
Soluo. Mostraremos que impossvel fazer esta decomposio. Com

efeito, suponha que existem tais conjuntos, A1 = {p1 , p2 , . . . , pr } e

A2 = {q1 , q2 , . . . , qs }. Ento

p1 p2 pr = q1 q 2 qs
| {z } | {z }

e alm disso, como os conjuntos A1 e A2 so disjuntos, temos que o

nmero 5 aparece no produto ou no produto , mas no em ambos

simultaneamente. Por outro lado, o Teorema 3.49 nos diz que a fatora-

o em primos de igual fatorao em primos de , logo o nmero


5 deveria aparecer tanto no produto como no produto , contra-

dizendo isto o fato anterior. Portanto no existe uma decomposio

com as condies exigidas.


3.5 Procurando Primos 137

Exemplo 3.53. Encontre todos os nmeros inteiros e positivos n com


a propriedade de que o conjunto

A = {n, n + 1, n + 2, n + 3, n + 4, n + 5}

pode ser particionado em dois subconjuntos tais que o produto dos


elementos de um dos subconjuntos seja igual ao produto dos elementos
do outro.

Demonstrao. Digamos que seja possvel essa decomposio para al-

gum n e vamos denotar os conjuntos que obtemos com a decomposio


por A1 e A2 . Observando a decomposio dos elementos dos subcon-

juntos em fatores primos, temos que todo fator primo de A1 tambm

dever pertencer a A2 . No conjunto dos seis nmeros s podemos ter

um mltiplo de 7, por isso no podemos tomar n como mltiplo deste

primo. Analogamente para primos maiores que 7. Analisando o primo

5, conclumos que n e n + 5 so mltiplos de 5, pois se no, cairamos


na anlise anterior. Assim, os nmeros n + 1, n + 2, n + 3 e n + 4 so

da forma 2 3 . Como entre eles existem dois mpares, logo teremos

duas potncias de 3 cuja diferena 2, um absurdo. Assim, no existe

n que satisfaz as condies do enunciado.

Finalizamos esta seo com um exemplo que mostra como podemos

combinar os fatos estudados para resolver problemas mais difceis

Exemplo 3.54. Encontre todos os nmeros que so formados por 4


algarismos da forma aabb e que sejam quadrados perfeitos.
138 3 Divisibilidade

Soluo. Como o nmero aabb um quadrado perfeito, signica que:

n2 =aabb
n2 =103 a + 102 a + 10b + b = 103 + 102 a + (10 + 1) b


n2 =1100 a + 11 b
n2 =11 100a + b = 11 99a + a + b .
 

Como 11 primo fcil ver, usando a Proposio 3.43, que 112 | N 2 .


Segue-se ento que 11 | (99a+a+b). Portanto, 11 | (a+b). Como aabb

tem 4 algarismos, segue-se que a 6= 0; portanto a {1, 2, 3, . . . , 9} e

b {0, 1, 2, . . . , 9}. De onde a + b 18. Logo, necessariamente


devemos ter a + b = 11. Podemos observar que a 6= 1, pois se a = 1

ento b = 10. Analogamente, b 6= 0, 1. Portanto,

a {2, 3, 4, . . . , 9} e b {2, 3, 4, . . . , 9}.

Como em todo nmero quadrado perfeito o algarismo das unidades

somente pode acabar em 0, 1, 4, 5, 6 e 9. Segue-se que

b {4, 5, 6, 9}.

Certamente b 6= 5, pois todo nmero que acaba em 5 quando elevado


ao quadrado sempre acaba em 25. Assim,

b {4, 6, 9}.

Se b = 4, ento a = 7. Neste caso o nmero seria 7.744 que

um quadrado perfeito;

Se b = 6, ento a = 5. Neste caso o nmero seria 5.566 que no

um quadrado perfeito;
3.6 Exerccios 139

Se b = 9, ento a = 2. Neste caso o nmero seria 2.299 que no

um quadrado perfeito.

Finalmente, a nica soluo possvel aabb = 7.744 = 882 .

3.6 Exerccios

1. Encontre o resto que deixa

(a) 2001 2002 2003 2004 + 20052 quando dividido por 7;

(b) 2100 quando dividido por 3;

28
(c) (1237156 + 34) quando dividido por 111.

2. Provar que o nmero n5 + 4n divisvel por 5 para qualquer

nmero natural n.

3. Prove que se n mpar

(a) n3 n divisvel por 24;

(b) n2 1 divisvel por 8;

(c) n2 + (n + 2)2 + (n + 4)2 + 1 divisvel por 12.

4. O nmero 21093 2 divisvel por 10932 ?

5. Prove que (999994)1234567890 1 divisvel por 333331.

6. O nmero N = 42005 + 20054 primo?

7. Demonstre que o nmero 1 |000 {z


. . . 00} 1 composto.
2006 zeros
140 3 Divisibilidade

8. Utilizando o fato de que o resto de um quadrado quando dividido

por 4 s pode ser 0 ou 1, d uma outra soluo para o problema

do Exemplo 3.54.

9. Dados trs inteiros, x, y, z , tais que x2 + y 2 = z 2 , mostre que x


e y no so ambos mpares e que xy mltiplo de 6.

10. Demonstre que o quadrado de um inteiro da forma 8n ou 8n+1


ou 8n + 4.

11. Trs nmeros primos p, q e r, maiores que 3, formam uma pro-


gresso aritmtica, ou seja, q = p + d e r = p + 2d. Prove que d
divisvel por 6.

12. Demonstrar que existem innitos nmeros primos da forma 4m+


3 e da forma 6m + 5, onde m Z.

13. Encontrar o ltimo dgito dos nmeros

(a) 19892005 ;
(b) 777777 + 250 ;
(c) 1 + 22 + 32 + + 20052 .

14. Prove que a soma dos quadrados de cinco nmeros consecutivos

no um quadrado perfeito.

15. Prove que 1 |00 {z


00} 5 |00 {z
00} 1 no um cubo perfeito.
100zeros 100zeros

16. Seja b um inteiro positivo. Enuncie e prove o critrio de divisi-

bilidade por b no sistema de numerao de base b.

17. Prove que os nmeros


3.6 Exerccios 141

1 1 1
(a) n = 1 ++ + + , com n > 1,
2 3 n
1 1 1
(b) n = + + + , com n > 0,
3 5 2n + 1
no so inteiros.

18. Considere o polinmiop(n) = am nm + am1 nm1 + + a0 de


grau m 1 com coecientes inteiros e n N. Prove que p(n)

um nmero composto para innitos valores de n.

Sugesto: Use o fato de que existe a N tal que = |p(a)| > 1


e mostre que divide a p(k + a), para todo k Z.

19. Dizemos que um conjunto An formado por n inteiros positivos

escritos no sistema binrio (base 2) regular se, para qualquer

s inteiro no negativo a quantidade de nmeros de An que con-


s
templam 2 na representao binria par. Dizemos que An

irregular se, pelo menos para algum s, este nmero mpar. De-

monstre que um sistema irregular pode se converter em regular

excluindo-se apenas um nico elemento do mesmo, e, um sistema

regular pode se converter em irregular excluindo-se qualquer um

dos seus elementos.

20. Seja n um inteiro positivo. Demonstrar que todos os coecientes

do desenvolvimento do binmio de Newton (a + b)n so mpares


s
se, e somente se, n da forma 2 1.

21. Prove que se (x0 , y0 ) uma soluo da equao diofantina linear


ax by = 1, ento a rea do tringulo cujos vrtices so (0, 0),
(b, a) e (x0 , y0 ) 1/2.
142 3 Divisibilidade

22. Qual a menor distncia possvel entre dois pontos (x1 , y1 ) e

(x2 , y2 ), com coordenadas inteiras, situados sobre a reta denida


pela equao diofantina ax + by = c?
4
O Princpio da Casa dos
Pombos

Uma vez um matemti o me falou que o verdadeiro prazer no est

em a har a verdade, mas em pro urar por ela.

Leo Tolstoy

Um interessante instrumento elementar para tratar problemas mate-

mticos relacionados existncia de elementos de conjuntos validando

certas exigncias o chamado princpio de Dirichlet , tambm conhe-


cido como princpio da casa dos pombos (PCP) . Este princpio foi
usado por Dirichlet (1805-1859) para resolver problemas na Teoria

dos Nmeros, entretanto ele possui um grande nmero de aplicaes

em diversos ramos da Matemtica como Combinatria e Geometria.

A seguir enunciamos a verso mais simples do PCP.

Proposio 4.1 (PCP  Verso Simples) . Se distribumos N + 1


pombos em N casas, ento alguma das casas contm dois ou mais
pombos.

143
144 4 O Princpio da Casa dos Pombos

P1 P2 PN
C1 C2 CN

PN +1

Figura 4.1: Em cada casa Cj , 1 j N , coloca-se um nico pombo,

denotado por Pj . O pombo restante, denotado por PN +1 , deve ir para

alguma das casas, juntando-se ao que j se encontrava contido nela

Demonstrao. A prova deste princpio muito fcil e decorre de fa-

zer uma simples contagem dos pombos contidos em todas as casas de-

pois de distribudos. Com efeito, suponhamos pelo contrrio que em

cada casa no existe mais do que um pombo, ento contando todos

os pombos contidos nas N casas no teremos mais do que N pombos,

contradizendo isto a hipteses de termos N +1 pombos distribudos

nas N casas (ver Figura 4.1).

No difcil detectar quando o princpio pode ser usado, mas a

principal diculdade para aplic-lo reside em identicar, em cada pro-

blema, quem faz papel de pombos e quem faz papel de casas.

Nas seguintes sees discutiremos vrios exemplos de diferentes

naturezas onde o princpio da casa dos pombos aplicado com sucesso.


4.1 Primeiros Exemplos 145

4.1 Primeiros Exemplos

Exemplo 4.2. Numa oresta crescem 1.000 jaqueiras. conhecido


que uma jaqueira no contm mais do que 600 frutos. Prove que
existem 2 jaqueiras na oresta que tm a mesma quantidade de frutos.

Soluo. Temos 1.000 jaqueiras, representando os pombos, e 601 casas

identicadas pelos nmeros 0, 1, 2, 3, . . . , 600. O nmero k associado

a cada casa signica que nela sero colocadas jaqueiras que tm exa-

tamente k frutos. Como 1000 > 602 = 601 + 1, o PCP nos garante

que existem duas jaqueiras com a mesma quantidade de frutos.

Exemplo 4.3. Em uma reunio h n pessoas. Mostre que existem


duas pessoas que conhecem exatamente o mesmo nmero de pessoas.

Soluo. Os pombos neste caso so as n pessoas. As casas so enume-

radas com os nmeros 0, 1, 2, . . . , n 1, indicando estes que na mesma


sero colocadas pessoas que tm essa quantidade de conhecidos. No-

temos que uma das casas enumeradas com 0 ou n 1 permanece


desocupada, pois a possibilidade de conhecer 0 e n 1 pessoas no

acontece simultaneamente. Logo, nas n 1 casas restantes haver

uma ocupada por dois ou mais pombos, depois de serem distribudos.

Portanto, existem no mnimo duas pessoas com o mesmo nmero de

conhecidos.

Exemplo 4.4. Dados 8 nmeros inteiros mostre que existem dois


deles cuja diferena divisvel por 7.

Soluo. Consideramos os 8 nmeros como sendo os pombos e as casas

como sendo os 7 possveis restos na diviso por 7. Como temos 8=


7+1 nmeros o PCP nos diz que existem dois nmeros dentro dos
146 4 O Princpio da Casa dos Pombos

8 dados que tm o mesmo resto quando divididos por 7. Finalmente,

observamos que se dois nmeros deixam o mesmo resto na diviso por

7 ento a diferena entre eles divisvel por 7.

Uma forma alternativa e muito til na qual pode-se apresentar o

princpio da casa dos pombos a seguinte:

Proposio 4.5 (PCP  Verso Alternativa) . Se a soma de n n-


meros naturais igual S , ento existe pelo menos um deles que no
maior que S/n, assim como existe pelo menos um deles que no
menor que S/n.
Exemplo 4.6. Numa famlia formada por 5 pessoas a soma das idades
de 245 anos. Prove que podem ser selecionados 3 membros da famlia
cuja soma das idades no menor que 147.
Soluo. 5 5!

Temos um total de
3
= 3!2!
= 10 trios diferentes formados

por membros da famlia. Alm disso, cada pessoa aparece exatamente


4 4!

em
2
= 2!2!
=6 trios. Ento, denotando por Ej a soma das idades

dos membros de cada trio Tj , j = 1, 2 . . . 10, temos que

E1 + E2 + + E10 = 6 245 = 1470;


1470
consequentemente existe algum trio Tj tal que Ej 10
= 147.

4.2 Uma Verso mais Geral

A seguinte verso mais geral do PCP bastante til na resoluo de

alguns problemas.

Proposio 4.7 (PCP  Verso Geral). Se distribumos N k + 1 pom-


bos em N casas, ento alguma das casas contm pelo menos k + 1
pombos.
4.2 Uma Verso mais Geral 147

A prova deste enunciado mais geral similar anterior. Com efeito,

suponhamos pelo contrrio que em cada casa no existe mais do que

k pombos, ento contando todos os pombos contidos nas N casas no

teremos mais do que Nk pombos, contradizendo isto a hipteses de

termos Nk + 1 pombos distribudos nas N casas.

Notemos que se k = 1, esta verso mais geral coincide com a verso


mais simples.

Exemplo 4.8. Num colgio com 16 salas so distribudas canetas nas


cores preta, azul e vermelha para realizar uma prova de concurso. Se
cada sala recebe canetas da mesma cor ento prove que existem pelo
menos 6 salas que receberam canetas da mesma cor.
Soluo. Fazendo a diviso com resto de 16 por 3 temos que 16 =
3 5 + 1. Consideramos as 16 salas como sendo os pombos e as trs

cores, preto, azul e vermelho como sendo as casas. Logo, podemos

colocar cada sala em uma das trs cores. Assim, o PCP com N =3
e k = 5 nos d que existe uma casa com pelo menos 6 pombos, ou seja,
existem no mnimo 6 salas que receberam canetas da mesma cor.

Exemplo 4.9. Uma equipe formada por seis alunos de Matemtica


selecionada para representar o Brasil numa olimpada internacional.
Mostre que necessariamente existem trs deles que se conhecem mu-
tuamente, ou trs deles que no se conhecem mutuamente.
Soluo. Resolveremos o problema com o auxlio da Figura 4.2. Cada

aluno Aj , com j = 1, 2, . . . , 6, representado por um dos vrtices de

um hexgono regular. Quando dois alunos se conhecem traamos o

segmento de reta que liga os vrtices correspondentes com uma linha

contnua; caso contrrio traamos este segmento com uma linha pon-

tilhada. Logo, usando este esquema, o problema equivale a provar


148 4 O Princpio da Casa dos Pombos

que sempre existe um tringulo de lados contnuos ou um tringulo de

lados pontilhados com vrtices no conjunto A = {A1 , A2 , . . . , A6 }.


Temos 5 segmentos (pombos) incidindo no vrtice A1 , cada um

deles contnuo ou pontilhado (estes dois tipos de linhas so conside-

radas como as casas). Como 5 = 2 2 + 1, pelo PCP temos que 3

dos 5 segmentos so contnuos ou pontilhados. Suponhamos que 3 so

contnuos (caso contrrio o argumento similar) e denotemos estes

por A1 A3 , A1 A4 e A1 A6 (ver Figura 4.2). Se algum dos segmentos

A3 A4 , A3 A6 ou A4 A6 for contnuo ento este segmento junto aos que

se ligam com A1 formam um tringulo de lados contnuos. Por outro

lado, se nenhum deles for contnuo, ento eles formam um tringulo

de lados pontilhados, completando isto a demonstrao.

A3 A2

A4 A1

A5 A6

Figura 4.2: O tringulo A1 A2 A5 indica que os alunos A1 , A2 e A5 no se

conhecem mutuamente e o tringulo A1 A4 A6 indica que os alunos A1 , A4


e A6 se conhecem mutuamente
4.3 Aplicaes na Teoria dos Nmeros 149

4.3 Aplicaes na Teoria dos Nmeros

Nesta seo apresentamos alguns exemplos de aplicaes do PCP na

Teoria dos Nmeros. A primeira delas :

Exemplo 4.10. Se n e m so nmeros naturais, ento o conjunto


A = {m + 1, m + 2, . . . , m + n} possui algum divisor de n.

Soluo. Temos n nmeros diferentes no conjunto acima. Vamos utili-

zar o mtodo de reduo ao absurdo. Se no existisse nenhum mltiplo

de n, quando dividssemos os nmeros do conjunto A por n, os res-

tos pertenceriam ao conjunto B = {1, 2, . . . , n 1}, que possui n 1


elementos. Logo, devem existir dois nmeros m + i e m + j , com

1 i < j n tais que o resto da diviso de m + i por n o mesmo


que o resto da diviso de m + j por n. Logo, m + j (m + i) um

mltiplo de n, o que implica que n > j i 1 mltiplo de n menor

que n (absurdo!). Logo, deve existir algum mltiplo de n no conjunto

A.

Como consequncia desse exemplo, podemos resolver o prximo

problema.

Exemplo 4.11. Demonstrar que todo inteiro tem um mltiplo cuja


representao decimal comea com o bloco de dgitos 1234567890.

Soluo. m e n so inteiros positivos, pelo exemplo anterior um


Se

dos nmero m + 1, m + 2, . . . , m + n mltiplo de n. Assim, dado n


n+1
um inteiro qualquer, escolhe-se m = 123456789010 . Deste modo,

todos os inteiros m + 1, m + 2, . . . , m + n comeam com 1234567890 e

algum deles mltiplo de n.


150 4 O Princpio da Casa dos Pombos

Exemplo 4.12. Dado um nmero inteiro positivo n, mostre que existe


um mltiplo de n que se escreve com os algarismos 0 e 1 apenas. (Por
exemplo, se n = 3, temos 111 ou 1.101 etc.)

Soluo. Consideramos os n+1 nmeros

1, 11, 111, 1111, . . . , 111


| {z 1} (4.1)
n+1vezes

como sendo os pombos e n casas enumeradas com os nmeros

0, 1, 2, 3, . . . , n 1,

ou seja, com os possveis restos na diviso por n. Similarmente ao

exemplo anterior existem dois nmeros na lista (4.1) que deixam o

mesmo resto na diviso por n e, portanto, a diferena entre o maior e

o menor mltiplo de n. Obviamente a diferena entre dois nmeros

quaisquer da lista (4.1) resulta em um nmero formado apenas pelos

algarismos 0 e 1.

Exemplo 4.13. Prove que entre n + 1 elementos escolhidos no con-


junto {1,2,3, . . . , 2n} existem dois que so primos relativos.

Soluo. A escolha das casas e dos pombos neste exemplo no to b-

via. Os pombos representam os n + 1 nmeros escolhidos do conjunto


{1, 2, . . . , 2n} e as casas so escolhidas como sendo os n conjuntos:

Cj = {2j 1, 2j}, 1 j n.

Logo, pelo PCP, quando distribumos os n + 1 nmeros nos n conjun-


tos Cj , 1 j n, dois deles caro juntos em algum conjunto Cj , ou

seja, estes nmeros sero consecutivos e portanto primos entre si.


4.4 Aplicaes Geomtricas 151

Finalizaremos esta seo com uma outra prova do teorema de

Bachet-Bzout, (veja o Teorema 3.23).

Exemplo 4.14. Seja d = (a, b) o mdc entre os nmeros naturais a e


b. Ento, existem x e y nmeros inteiros tais que

ax + by = d.

Soluo. Denotando por m = a/d e n = b/d, podemos supor que a e

b so primos entre si. Realmente, se podemos escrever

mx + ny = 1

ento, substituindo os valores de m e n na equao acima, temos que

ax + by = d.
Se (a, b) = 1, considere a sequncia A = {a, 2a, . . . , ba}. Armamos

que existe algum nmero no conjunto A que deixa resto 1 quando

dividido por b. De fato, se isso no ocorresse, teramos b nmeros em

A deixando no mximo b 1 restos diferentes quando divididos por


b. Logo, pelo PCP, dois deles, digamos ia e ja com b > j > i 1,
devem deixar o mesmo resto quando divididos por b. assim, (j i)a

divisvel por b. Como estamos supondo que (a, b) = 1, temos que b

deve dividir j i > 0. Como b > j i, temos um absurdo.

Assim, algum dos nmeros em a deixa resto 1 quando divididos

por b. Digamos que esse nmero seja ax. Logo, ax 1 mltiplo de

b, onde ax 1 = by , o que encerra nossa prova.

4.4 Aplicaes Geomtricas

Na geometria tambm encontramos belas aplicaes do PCP. Vejamos


os problemas a seguir para constatar isto.
152 4 O Princpio da Casa dos Pombos

Exemplo 4.15. Mostre que se tomamos cinco pontos quaisquer sobre


um quadrado de lado 1, ento pelo menos dois deles no distam mais

que 2/2.

Soluo. Vamos dividir o quadrado em quatro quadradinhos de lado

1/2, como mostra a gura. Logo, pelo PCP pelo menos dois deles de-

vem estar no mesmo quadradinho, uma vez que temos 4 quadradinhos

e 5 pontos. Logo, como a maior distncia num quadrado a diagonal,

o Teorema de Pitgoras nos garante que a distncia desses dois pontos



no mximo 2/2, como queramos mostrar.

Exemplo 4.16. Na regio delimitada por um tringulo equiltero de


lado 4 so marcados 10 pontos no interior deste. Prove que existe ao
menos um par destes pontos cuja distncia entre eles no maior que

3.

Soluo. Dividimos o tringulo equiltero de lado 4 em 16 tringulos

equilteros menores de lado 1, conforme a Figura 4.3.

Agora pintamos os tringulos nas cores branco e cinza de maneira

que dois tringulos vizinhos, isto , com um lado comum, so pintados

de cores diferentes. Se tivssemos dois pontos no mesmo tringulo a

distncia mxima possvel entre eles seria 1 e o problema estaria resol-

vido. Se tivssemos pontos em tringulos vizinhos, a maior distncia



possvel entre eles seria 3 e tambm isto resolveria o problema. Se

no tivssemos nenhum dos casos anteriores, no seria difcil ver que


4.5 Miscelnea 153

C

E





A D B

Figura 4.3: O tringulo DBE equiltero de lado 3

os 10 pontos deveriam estar situados sobre os 10 tringulos brancos,

contendo cada tringulo exatamente um ponto. Dividindo o tringulo

DBE em 4 tringulos congruentes de lado 3/2 pelo PCP temos que


pelo menos dois dos 6 pontos contidos em DBE esto num destes 4

tringulos, logo a distncia entre eles no maior que 3/2 < 3. Com
isto terminamos nossa prova.

4.5 Miscelnea

Os problemas que apresentamos a seguir usam o PCP combinado com

outras idias que so muito empregadas nas suas solues.

Exemplo 4.17. Em cada quadradinho de um tabuleiro 3 3 colocado


um dos nmeros: -1, 0 ou 1. Prove que entre todas as somas das
linhas, colunas e diagonais do tabuleiro h duas que so iguais. Por
exemplo, no tabuleiro abaixo a soma da segunda linha 2, que coincide
com a soma da terceira coluna.
154 4 O Princpio da Casa dos Pombos

-1 -1 1

1 0 1

0 -1 0

Soluo. SejaS = a1 + a2 + a3 , onde cada a1 , a2 e a3 podem tomar


valores: 1, 0 e 1. Ento, temos 7 valores possveis para S (casas),

que so: 3, 2, 1, 0, 1, 2, 3.

O tabuleiro 33 tem 3 linhas, 3 colunas e 2 diagonais, portanto, ao

somarmos os elementos de cada uma das linhas, colunas e diagonais,

obteremos 8 nmeros (pombos). Como existem somente 7 valores

possveis para estes nmeros, pelo PCP pelo menos dois deles devem

ser iguais.

Exemplo 4.18. Dado qualquer conjunto A formado por 10 nmeros


naturais escolhidos entre 1 e 99, inclusos, demonstre que existem dois
subconjuntos disjuntos e no vazios de A tal que a soma dos seus res-
pectivos elementos igual.

Soluo: conhecido que A tem 210 1 = 1.023 subconjuntos no-

vazios diferentes. A soma dos elementos de cada um deles d uma

quantidade menor do que 1.000, pois o subconjunto com no mximo

10 elementos de maior soma possvel o formado por 90, 91, . . . , 99,


e nesse caso 90 + 91 + + 99 = 945. Agora consideramos os pombos

como sendo os 1.023 subconjuntos distintos de A e as casas como

sendo as somas possveis dos elementos de cada um dos conjuntos.

Logo, como o nmero de conjuntos maior que o nmero de somas

possveis, devem existir dois conjuntos B e C de A, de tal modo que

a soma dos elementos de B igual soma dos elementos de C. Se B


4.5 Miscelnea 155

e C so disjuntos, acabou a prova. Se no, considere D = B B C


e E = C B C . Logo, os conjuntos D e E so disjuntos e a soma
dos seus elementos a mesma, pois retiramos de ambos a mesma

quantidade.

Exemplo 4.19. Qual o maior nmero de quadradinhos de um ta-


buleiro de 8 8 que podem ser pintados de preto, de forma tal que
qualquer arranjo de trs quadradinhos, como mostra a Figura 4.4, te-
nha pelo menos um dos quadradinhos no pintado de preto?

Figura 4.4: Tridomins

Soluo. Primeiramente, pintamos o tabuleiro de 88 como um tabu-


leiro de jogar xadrez, ou seja, 32 quadradinhos pintados de branco e

32 quadradinhos pintados de preto (ver Figura 4.5).

Figura 4.5: Tabuleiro de xadrez


156 4 O Princpio da Casa dos Pombos

Notemos que uma vez pintado o tabuleiro desta forma satisfeita

a exigncia do problema, pois nunca temos 2 quadradinhos vizinhos

(quadradinhos com um lado comum) pintados de preto.

Mostraremos agora que se pintamos 33 quadradinhos de preto en-

to a condio exigida no problema falha. De fato, se dividimos o

tabuleiro em 16 quadrados de 22 (casas) e pintamos 33 quadra-

dinhos de preto (pombos); ento, como 33 = 16 2 + 1, pela verso


geral do PCP um dos 16 quadrados de 2 2 contm 3 quadradinhos
pintados de preto. Portanto, este ltimo contm um arranjo como na

Figura 4.4 completamente pintado de preto.

Resumindo, o nmero mximo de quadradinhos que podemos pin-

tar de preto 32.

Exemplo 4.20. Dados sete nmeros reais arbitrrios, demonstre que


existem dois deles, digamos x e y , tais que
xy 1
0
1 + xy 3
Soluo. Primeiramente observamos que a expresso
xy
1+xy
nos faz pen-

sar na frmula
tan tan
tan( ) = . (4.2)
1 + tan tan
Sejam x1 , x2 , , x7 os sete nmeros selecionados arbitrariamente.

Lembramos que a funo tangente uma bijeo entre o intervalo

( 2 , 2 ) e os nmeros reais R, logo para cada xi , 1 i 7, existe um


i ( 2 , 2 ) tal que tan(i ) = xi . Dividimos o intervalo ( 2 , 2 ) em

seis subintervalos de comprimento , como mostra o desenho a seguir.
6

Pelo PCP dois dos nmeros i pertencem ao mesmo subintervalo.

Denotemos os mesmos por i1 e i2 e suponhamos, sem perda de


4.6 Exerccios 157

i1 i2
2

2
6

generalidade, que i1 i2 . Ento vale


0 i2 i1 .
6
Usando o fato de que a tangente uma funo crescente e a frmula

(4.2) temos que


tan(0) tan(i2 i1 ) tan( ).
6
Equivalentemente,
xi 2 xi 1 1
0 .
1 + xi 2 xi 1 3

4.6 Exerccios

1. Seja C um conjunto formado por cinco pontos de coordenadas

inteiras no plano. Prove que o ponto mdio de algum dos seg-

mentos com extremos em C tem tambm coordenadas inteiras.

2. O conjunto dos dgitos 1, 2, ..., 9 dividido em trs grupos.

Prove que o produto dos nmeros de algum dos grupos deve ser

maior que 71.

3. Prove que se N mpar ento para qualquer bijeo

p : IN IN
158 4 O Princpio da Casa dos Pombos

do conjunto IN = {1, 2, . . . , N } o produto P (p) = (1 p(1))(2


p(2)) (N p(N )) necessariamente par.
(Dica: O produto de vrios fatores par se, e somente se, um dos

fatores par.)

4. Dado um conjunto de 25 pontos no plano tais que entre quaisquer

3 deles existe um par com distncia menor que 1. Prove que

existe um crculo de raio 1 que contm pelo menos 13 dos 25

pontos dados.

5. Prove que entre quaisquer 5 pontos escolhidos dentro de um

tringulo equiltero de lado 1 sempre existe um par deles cuja

distncia no maior que 0,5.

6. Marquemos todos os centros dos 64 quadradinhos de um ta-

buleiro de xadrez de 8 8. possvel cortar o tabuleiro com 13

linhas retas que no passem pelos pontos marcados e de forma

tal que cada pedao de recorte do tabuleiro tenha no mximo

um ponto marcado?

7. Prove que existem duas potncias de 3 cuja diferena divisvel

por 1.997.

8. So escolhidos 6 nmeros quaisquer pertencentes ao conjunto

A = {1, 2, 3, . . . , 10}.

Prove que existem dois desses seis nmeros cuja soma mpar.

9. Seja x um nmero real arbitrrio. Prove que entre os nmeros

x, 2x, 3x, . . . , 101x


4.6 Exerccios 159

existe um tal que sua diferena com certo nmero inteiro menor

0,011.

10. Mostre que entre nove nmeros que no possuem divisores pri-

mos maiores que cinco, existem dois cujo produto um qua-

drado.

11. Um disco fechado de raio um contm sete pontos, cujas distn-

cias entre quaisquer dois deles maior ou igual a um. Prove que

o centro do disco um destes pontos.

12. Na regio delimitada por um retngulo de largura quatro e altura

trs so marcados seis pontos. Prove que existe ao menos um



par destes pontos cuja distncia entre eles no maior que 5.

13. Seja a um nmero irracional. Prove que existem innitos nme-

ros racionais r = p/q tais que |a r| < 1/q 2 .

14. Suponha que cada ponto do reticulado plano pintado de vermelho


ou azul. Mostre que existe algum retngulo com vrtices no reticulado
e todos da mesma cor.

15. Um certo livreiro vende pelo menos um livro por dia. Sabendo que o
livreiro vendeu 463 livros durante 305 dias consecutivos, mostre que
em algum perodo de dias consecutivos o livreiro vendeu exatamente
144 livros.
Referncias Bibliogrcas
[1] AIGNER, M. e ZIEGLER, G. (2002). As Provas esto
no Livro. Edgard Blcher.

[2] GARCIA, A. e LEQUAIN, I. (2003). Elementos de l-


gebra. Projeto Euclides, IMPA.

[3] LIMA, E. L.; CARVALHO, P. C. P.; WAGNER, E. e

MORGADO, A.C. (2004). A Matemtica do Ensino M-


dio. Volume 1. Sociedade Brasileira de Matemtica.

[4] LIMA, E.L.; CARVALHO, P. C. P.; WAGNER, E. e

MORGADO, A.C. (2004). A Matemtica do Ensino M-


dio. Volume 2. Sociedade Brasileira de Matemtica.

[5] LIMA,E.L.; CARVALHO,P. C. P.; WAGNER,E. e

MORGADO,A.C. (2004). A Matemtica do Ensino M-


dio. Volume 3. Sociedade Brasileira de Matemtica.

[6] LIMA, E.L.; CARVALHO, P. C. P.; WAGNER,E. e

MORGADO, A.C. (2001). Temas e Problemas. Socie-

dade Brasileira de Matemtica.

[7] LIMA, E.L. (2001). lgebra Linear. Sociedade Brasileira


de Matemtica.

285
286 REFERNCIAS BIBLIOGRFICAS

[8] MORAIS FILHO, D. C. (2007). Um Convite Matem-


tica. EDUFCG.

[9] MORGADO, A.; CARVALHO, J.; CARVALHO, P.;

FERNANDEZ, P. (1991). Anlise Combinatria e Pro-


babilidade . Sociedade Brasileira de Matemtica.

[10] RIBENBOIM, P. (2001). Nmeros Primos: Mistrios e


Recordes. Sociedade Brasileira de Matemtica.

[11] SANTOS, J. P. O. (1993) Introduo Teoria dos N-


meros. IMPA.

[12] SANTOS, J. P. O.; MELLO, M. P. e MURARI, I. T.

C. (2006). Introduo Anlise Combinatria. Editora

Unicamp.

[13] SOARES, M. G. (2005). Clculo em uma Varivel Com-


plexa. Sociedade Brasileira de Matemtica.
Mestrado Profissional
em Matemtica em Rede Nacional

Iniciao Matemtica

Autores:

Krerley Oliveira Adn J. Corcho

Unidade III:

Captulos V e VI
160
5
Contagem

Toda vez que puder, onte.

Fran is Galton

Neste captulo discutiremos problemas envolvendo a contagem de


elementos de um conjunto nito dado. Por exemplo, responderemos
perguntas do tipo: de quantos modos podemos distribuir 32 selees
nacionais de futebol em seis grupos de quatro times cada?
Para solucionar questes como esta, utilizaremos como ferramentas
bsicas os princpios aditivo e multiplicativo da contagem. Veremos
tambm que o uso simultneo destes princpios ser muito til para
resolver problemas com certos nveis de complexidade. Alm disso,
sero abordados os conceitos de permutaes, arranjos e combinaes,
sendo estes de muita importncia por serem os alicerces de um ramo
da matemtica denominado combinatria.
Antes de prosseguirmos daremos algumas denies e notaes que
sero teis ao longo de todo o captulo. Dado um conjunto A deno-
tamos por |A| a quantidade de elementos que este possui. O produto
cartesiano de n conjuntos A1 , A2 , . . . , An1 e An o conjunto denido

161
162 5 Contagem

por

A1 A2 An := (a1 , a2 , . . . , an ); ai Ai , i = 1, 2, . . . , n ,

onde cada elemento (a1 , a2 , . . . , an ) chamado de n-upla ordenada.


Denotaremos o conjunto vazio com o smbolo . O leitor que deseja
rever os conceitos bsicos da teoria de conjuntos, pode ach-los muito
bem expostos em [3].

5.1 Princpio Aditivo da Contagem

O princpio aditivo da contagem garante que dados dois conjuntos


nitos que no tm elemento em comum, o nmero de elementos da
unio exatamente a soma do nmero de elementos de cada um, ou
seja, se A1 e A2 so disjuntos (isto , A1 A2 = ), ento
|A1 A2 | = |A1 | + |A2 |.

Apesar de sua simplicidade, muitos problemas podem ser resolvi-


dos utilizando esse simples princpio. A seguir enunciamos uma ex-
tenso deste princpio para um nmero nito qualquer de conjuntos.

Princpio Aditivo da contagem: Dados os conjuntos nitos A1 ,


A2 , . . . , An dois a dois disjuntos (isto , Ai Aj = , i 6= j ),
temos que
|A1 A2 An | = |A1 | + |A2 | + + |An |.

Exemplo 5.1. Em Macei entraram em cartaz 4 lmes distintos e


2 peas de teatro. Se Pedro Vtor s tem dinheiro para assistir a um
lme ou a uma pea de teatro, diga quantos so os possveis programas
de Pedro Vtor.
5.1 Princpio Aditivo da Contagem 163

Soluo. Denotemos por f1 , f2 , f3 e f4 os quatro lmes que esto em


cartaz e por t1 e t2 as duas peas de teatro. Agora, representemos pelo
par (i, j), com 0 i 4 e 0 j 2, o programa que consiste em as-
sistir ao lme fi e pea tj (caso i = 0 ou j = 0 isso signica que no
ser assistido a nenhum lme ou a nenhuma pea, respectivamente).
Pelas limitaes econmicas do Pedro Vtor temos que ele s pode
escolher um programa dentro dos seguintes conjuntos disjuntos:
 
A1 = (1, 0), (2, 0), (3, 0), (4, 0) e A2 = (0, 1), (0, 2) .
Logo, no total so |A1 A2 | = |A1 | + |A2 | = 6 programas distintos,
entre os quais Pedro Vtor ter que escolher um.
Exemplo 5.2. Numa reunio havia um certo nmero de pessoas e
todos os presentes apertaram as mos entre si. Sabendo-se que ao todo
foram feitos 66 cumprimentos, calcule o nmero de pessoas presentes
reunio.
Soluo. Vamos enumerar as pessoas com os nmeros do conjunto
P = {1, 2, . . . , n}. A cada aperto de mo associaremos um par (i, j),
signicando que a pessoa i apertou a mo da pessoa j . Assim, os
apertos de mo envolvendo a pessoa 1 foram:

A1 = {(1, 2), (1, 3), . . . , (1, n)}.


Do mesmo modo, denimos os apertos de mo envolvendo a pessoa 2
que no envolvem a pessoa 1, como:
A2 = {(2, 3), (2, 4), . . . , (2, n)}.
Note que o aperto (2, 1) o mesmo que o aperto (1, 2), j que se 1
aperta a mo de 2, ento 2 aperta a mo de 1. Analogamente,
Ai = {(i, i + 1), (i, i + 2), . . . , (i, n)}, para 1 i n.
164 5 Contagem

Note que Ai Aj = para i 6= j . Observe tambm que todos os


apertos aparecem em um dos conjuntos Ai . Assim, A1 An
contm todos os apertos de mo. Logo, pelo princpio aditivo:
|A1 A2 An | = |A1 | + |A2 | + . . . |An |
= (n 1) + (n 2) + + 2 + 1
(n 1)n
= = 66.
2
Resolvendo em n, temos que n = 12.

Vimos que o princpio aditivo nos fornece o nmero de elementos


de qualquer unio de conjuntos dois a dois disjuntos. Discutiremos
agora uma extenso do princpio para qualquer unio de conjuntos,
no necessariamente dois a dois disjuntos.
Proposio 5.3. Sejam A1 e A2 dois conjuntos nitos quaisquer.
Ento,
|A1 A2 | = |A1 | + |A2 | |A1 A2 |.
Demonstrao. Observe que
A1 A2 = (A1 A2 ) A2

onde a unio dois a dois disjunta. Pelo princpio aditivo, temos que
|A1 A2 | = |A1 A2 | + |A2 |. (5.1)

Analogamente, aplicando novamente este princpio, temos que

|A1 | = |A1 A2 | + |A1 A2 |; (5.2)


A proposio segue imediatamente combinando as igualdades (5.1) e
(5.2).
5.1 Princpio Aditivo da Contagem 165

Para chegar a uma expresso anloga do princpio aditivo, vamos


fazer mais um caso, considerando agora trs conjuntos.

Corolrio 5.4. Sejam A1 , A2 e A3 trs conjuntos nitos quaisquer.


Ento,

|A1 A2 A3 | =|A1 | + |A2 | + |A3 |



|A1 A2 | + |A1 A3 | + |A2 A3 |
+ |A1 A2 A3 |.

Demonstrao. Pela Proposio 5.3 temos que,

|A1 (A2 A3 )| = |A1 | + |A2 A3 | |A1 (A2 A3 )|,

de onde,

|A1 A2 A3 | = |A1 | + |A2 A3 | |(A1 A2 ) (A1 A3 )|.

Novamente, pela Proposio 5.3 temos que,

|A1 A2 A3 | = |A1 | + |A2 | + |A3 | |A2 A3 | |(A1 A2 ) (A1 A3 )|.

Aplicando mais uma vez a Proposio 5.3 temos que,

|(A1 A2 ) (A1 A3 )| = |A1 A2 | + |A1 A3 | |(A1 A2 ) (A1 A3 ).

Combinando as duas ltimas igualdades obtemos

|A1 A2 A3 | =|A1 | + |A2 | + |A3 |



|A1 A2 | + |A1 A3 | + |A2 A3 |
+ |A1 A2 A3 | ,

como desejvamos.
166 5 Contagem

Para facilitar nossa escrita, vamos denotar por A1 A2 . . . Ak o con-


junto A1 A2 Ak . Assim, outra forma de enunciar o Corolrio
5.4 a seguinte:

[3 X 3 X X

A i = |Ai | |Ai1 Ai2 | + |Ai1 Ai2 Ai3 |.

i=1 i=1 1i1 <i2 3 1i1 <i2 <i3 3

De forma geral, dados os conjuntos nitos A1 , A2 , . . . , An , as ex-


presses anteriores nos levam a denir os nmeros:
n
X
S1 = |Ai |
i=1
X
S2 = |Ai1 Ai2 |,
1i1 <i2 n
..
.
X
Sk = |Ai1 Ai2 . . . Aik |,
1i1 <i2 <<ik n
..
.
Sn = |A1 A2 . . . An |.

Assim, a verso mais geral do princpio aditivo, tambm conhecida


como princpio de incluso e excluso , :

Princpio Aditivo - Verso Geral: Sejam A1 , A2 . . . , An


conjuntos nitos quaisquer. Ento,

[n

Ai = S1 S2 + S3 S4 + + (1)n1 Sn .

i=1

No iremos provar essa verso, mas o leitor pode (e deve!) mostr-la


como exerccio, repetindo os argumentos anteriores.
5.1 Princpio Aditivo da Contagem 167

Exemplo 5.5. No Colgio Fantstico foram entrevistados 78 estudan-


tes. Destes, 32 estavam fazendo um curso de francs; 40 um curso de
fsica; 30 um curso de matemtica; 23 um curso de histria; 19 francs
e fsica; 13 francs e matemtica; 15 fsica e matemtica; 2 francs e
histria; 15 fsica e histria; 14 matemtica e histria; 8 francs, f-
sica e matemtica; 8 francs, fsica e histria; 2 francs, matemtica
e histria; 6 fsica, matemtica e histria e 2 estavam fazendo todos
os quatro cursos. Quantos estudantes estavam fazendo pelo menos 1
curso nas 4 reas mencionadas?

Soluo. Denotemos por A1 , A2 , A3 , e A4 os conjuntos dos estudan-


tes que fazem francs, fsica, matemtica e histria, respectivamente.
Observemos que as igualdades

|A1 | = 32,
|A2 | = 40,
|A3 | = 30,
|A4 | = 23,

4
X
nos do que S1 = |Ai | = 125; as igualdades
i=1

|A1 A2 | = 19,
|A1 A3 | = 13,
|A1 A4 | = 2,
|A2 A3 | = 15,
|A2 A4 | = 15,
|A3 A4 | = 14,
168 5 Contagem

X
nos do que S2 = |Ai1 Ai2 | = 78; as igualdades
1i1 <i2 4

|A1 A2 A3 | = 8,
|A1 A2 A4 | = 8,
|A1 A3 A4 | = 2,
|A2 A3 A4 | = 6,

X
nos do que S3 = |Ai1 Ai2 Ai3 | = 24; assim como que S4 =
1i1 <i2 <i3 4
|A1 A2 A3 A4 | = 2.
[ 4

Segue-se ento, do princpio aditivo, que Ai = 125 78 + 24

i=1
2 = 69.

Denio 5.6. Denimos o complementar do conjunto A em relao


ao conjunto U como sendo um subconjunto de U dado por

Ac = x U; x
/A .

Figura 5.1: A rea branca corresponde a Ac e o conjunto U representado


por todo o retngulo
5.1 Princpio Aditivo da Contagem 169

Neste caso fcil vericar que os conjuntos A e Ac so disjuntos e


que U = A Ac . Segue-se do princpio aditivo que |U| = |A| + |Ac |;
portanto,
|Ac | = |U| |A|.
Analogamente, dados dois conjuntos A1 U e A2 U , temos que
A1 A2 e (A1 A2 )c so disjuntos e, alis, U = (A1 A2 ) (A1 A2 )c .
Novamente, pelo princpio aditivo, vale que
|U| = |A1 A2 | + |(A1 A2 )c |;

e consequentemente temos que


|(A1 A2 )c | = |U| (|A1 | + |A2 |) + |A1 A2 |.

Similarmente, dados trs conjuntos A1 U, A2 U e A3 U


podemos demonstrar que
|(A1 A2 A3 )c | = |U| (|A1 | + |A2 | + |A3 |)
+ (|A1 A2 | + |A1 A3 | + |A2 A3 |)
|A1 A2 A3 |.
Ento, usando a notao S0 = |U|, temos a seguinte proposio:
Proposio 5.7. Para toda famlia de subconjuntos Ai U , i =
1, 2, . . . , n, vale a relao:
!c
[ n 

Ai = S0 S1 S2 + S3 S4 + (1)n1 Sn

i=1

= S0 S1 + S2 S3 + S4 + (1)n Sn ,
ou resumidamente,
n !c
[ n
X
c c c
Ai = |A1 A2 An | = (1)j Sj .

i=1 j=0
170 5 Contagem

Observao 5.8. Observemos que na ltima relao da proposio


usamos a conhecida Lei de DeMorgan: o complementar da unio de
uma famlia nita de conjuntos, em relao a um conjunto U , a
interseco dos complementares de cada um deles.

5.2 Princpio Multiplicativo de Contagem

Comeamos esta seo discutindo um problema relacionado com o


apaixonante jogo de xadrez. Ele consiste no seguinte: queremos saber
de quantas maneiras diferentes podemos colocar duas torres num tabu-
leiro de xadrez de forma tal que nenhuma ataque a outra. Uma situa-
o como a que procuramos mostrada na Figura 5.2, pois lembramos
que torres s se movimentam na direo horizontal ou na direo verti-
cal do tabuleiro. Antes de prosseguir deixamos claro o seguinte: se na
Figura 5.2 trocamos a posio da torre a com a torre b consideraremos
isto como uma situao diferente.

Figura 5.2: Torres que no se atacam

Notemos o seguinte: uma vez que coloquemos uma das torres numa
5.2 Princpio Multiplicativo de Contagem 171

casa do tabuleiro no podemos colocar a segunda torre na mesma


linha ou coluna em que esta se encontra, pois ela seria ameaada.
Como cada linha e cada coluna contm 8 casas do tabuleiro, sendo
uma delas comum a ambas, ento temos 15 posies proibidas para
colocar a segunda torre, ou seja, ela s pode ser colocada em 6415 =
49 posies diferentes. Resumindo, por cada uma das 64 possveis
posies para a torre a temos 49 possibilidades diferentes para colocar
a torre b, totalizando 6449 = 3.136 formas diferentes de colocar ambas
as torres no tabuleiro sem que elas se ataquem.
O exemplo acima traz a essncia do que chamado princpio mul-
tiplicativo da contagem : se um evento A1 pode ocorrer de m maneiras
distintas e, se para cada uma dessas m maneiras possveis de A1 ocor-
rer, um outro evento A2 pode ocorrer de n maneiras distintas, ento o
nmero de maneiras de ocorrerem sucessivamente os eventos A1 e A2
m n.
Na linguagem matemtica: relembramos que dados dois conjuntos
A1 e A2 , podemos construir um par ordenado (a1 , a2 ) tomando um
elemento a1 A1 , denominado o primeiro elemento do par, e um
elemento a2 A2 , denominado o segundo elemento do par. O conjunto
A1 A2 constituido por todos os pares ordenados construdos dessa
forma. Assim sendo, a verso mais simples do princpio multiplicativo
nos garante que

|A1 A2 | = |A1 | |A2 |.

Uma extenso deste princpio para um nmero nito qualquer de


conjuntos a seguinte:

princpio multiplicativo da contagem: Dados os conjuntos


172 5 Contagem

nitos A1 , A2 , . . . , An temos que

|A1 A2 An | = |A1 | |A2 | |An |.

Note que neste princpio, no necessria nenhuma hiptese adi-


cional sobre os conjuntos Ai . Vamos agora dar alguns exemplos de
como aplicar esse princpio.

Exemplo 5.9. Em Macei entraram em cartaz 4 lmes distintos e


2 peas de teatro. Se agora o Pedro Vtor tem dinheiro para assistir
exatamente a um lme e a uma pea de teatro, diga quantos so os
possveis programas que Pedro Vtor pode fazer.

Soluo. Denotemos por f1 , f2 , f3 e f4 os quatro lmes que esto em


cartaz e por t1 e t2 as duas peas de teatro. Denamos os conjuntos

A1 = {f1 , f2 , f3 , f4 } e A2 = {t1 , t2 }.

Neste caso, as condies econmicas do Pedro Vtor permitem que


ele escolha um elemento do conjunto A1 e outro elemento do conjunto
A2 . Este tipo de escolha representa-se pelo conjunto


A1 A2 = (fi , tj ); 1 i 4 e 1 j 2 ,

onde cada par (fi , tj ) representa o programa que consiste em assistir


ao lme fi e pea tj . Logo, no total so |A1 A2 | = |A1 | |A2 | = 8
programas distintos.

Exemplo 5.10. Se numa loja de doces existem 9 tipos distintos de


balas e 5 tipos distintos de chiclete, diga quantas escolhas podemos
fazer para comprar somente uma bala e um chiclete.
5.2 Princpio Multiplicativo de Contagem 173

Soluo. Denotemos por b1 , b2 , b3 , b4 , b5 , b6 , b7 , b8 e b9 os nove tipos


distintos de balas e por c1 , c2 , c3 , c4 e c5 os cinco tipos distintos de
chicletes. Denamos os conjuntos

B = {b1 , b2 , b3 , b4 , b5 , b6 , b7 , b8 , b9 } e C = {c1 , c2 , c3 , c4 , c5 }.

Como precisamos comprar simultaneamente um elemento do conjunto


B e um elemento do conjunto C , ento o conjunto B C me d o
conjunto de todas as escolhas possveis. Logo, o nmero de escolhas
possveis para comprar simultaneamente um tipo de bala e um tipo
de chiclete |B C| = 9 5 = 45.

Exemplo 5.11. De quantas maneiras 2 pessoas podem estacionar seus


carros numa garagem com 10 vagas?

Soluo. Observando que a primeira pessoa pode estacionar seu carro


de 10 formas distintas e que a segunda pessoa pode estacionar seu
carro de 9 formas distintas, temos pelo princpio multiplicativo que
existem 9 10 = 90 formas possveis nas quais duas pessoas podem
estacionar seus carros numa garagem com 10 vagas.

Exemplo 5.12. Dado o nmero 720, diga

(a) quantos divisores inteiros e positivos ele possui;

(b) entre seus divisores inteiros e positivos, quantos so pares;

(c) entre seus divisores inteiros e positivos, quantos so mpares;

(d) dos divisores acima, quantos so quadrados perfeitos.


174 5 Contagem

Soluo. Pelo teorema fundamental da aritmtica, todo nmero in-


teiro positivo primo ou produto de primos. Observe que a decom-
posio de 720 em fatores primos vem dada por:

720 = 24 32 51 . (5.3)

Agora denamos os seguintes conjuntos:

A ={todos os divisores de 720 que so da forma 2k , onde k Z+ },


B ={todos os divisores de 720 que so da forma 3m , onde m Z+ },
C ={todos os divisores de 720 que so da forma 5n , onde n Z+ }.

Observemos que 0 k 4, pois se k > 4 ento pelo menos a potncia


25 deveria estar presente em (5.3); como isto no acontece segue-se
que 0 k 4, de modo que

A = 20 , 21 , 22 , 23 , 24 ,
seguindo o mesmo raciocnio, podemos demonstrar que 0 m 2 e
que 0 n 1. Assim,
 
B = 30 , 31 , 32 e C = 50 , 51 .

(a) O conjunto de todos os possveis divisores de 720 pode ser identi-


cado com o conjunto AB C . De onde o nmero de divisores
inteiros e positivos de 720 |AB C|. Porm, o princpio mul-
tiplicativo nos garante que |AB C| = |A||B||C|. Portanto,
o nmero de divisores inteiros e positivos de 720 5 3 2 = 30,
pois |A| = 5, |B| = 3 e |C| = 2.

(b) Para obter o conjunto de todos os divisores pares de 720 deve-


mos remover o elemento 20 do conjunto A. Assim, o conjunto de
5.2 Princpio Multiplicativo de Contagem 175

todos os divisores pares e positivos de 720 vem dado pelo con-


junto A {20 } B C . O princpio multiplicativo nos garante

que A {20 } B C = A {20 } |B| |C|. Portanto, o
nmero de divisores pares e positivos de 720 4 3 2 = 24,

pois A {20 } = 4, |B| = 3 e |C| = 2.

(c) Para obter o conjunto de todos os divisores mpares de 720 deve-


mos remover os elementos 21 , 22 , 23 e 24 do conjunto A. Assim,
o conjunto de todos os divisores mpares e positivos de 720 vem
dado pelo conjunto

A {21 , 22 , 23 , 24 } B C.

O princpio multiplicativo nos garante que



A {21 , 22 , 23 , 24 } B C = A {21 , 22 , 23 , 24 } |B| |C|.

Portanto, o nmero de divisores mpares e positivos de 720



1 3 2 = 6; pois A {21 , 22 , 23 , 24 } = 1, |B| = 3 e |C| = 2.

(d) Para obter o conjunto de todos os divisores de 720 que so qua-


drados perfeitos devemos car com as potncias pares nos con-
juntos A, B e C , respectivamente. Portanto, devemos remover
os elementos 21 , 23 do conjunto A. Tambm devemos remover o
elemento 31 do conjunto B . Finalmente do conjunto C devemos
remover o elemento 51 . Logo, o conjunto de todos os divisores
quadrados perfeitos e positivos de 720 vem dado pelo conjunto
  
D := A {21 , 23 } B {31 } C {51 } .

O princpio multiplicativo nos garante que



D = A {21 , 23 } B {31 } C {51 } .
176 5 Contagem

Portanto, o nmero de divisores quadrados perfeitos e positivos



de 720 3 2 1 = 6; pois A {21 , 23 } = 3, B {31 } = 2

e C {31 } = 1. Observe que {1, 4, 9, 16, 36, 144} o conjunto
dos divisores de 720 que so quadrados perfeitos.

Exemplo 5.13. Se um nmero natural n se fatora como

n = pk11 pk22 pkr r , (5.4)

onde os pi so nmeros primos distintos e cada ki Z+ , ento o


nmero de divisores positivos de n, denotado por d(n)

d(n) = (k1 + 1)(k2 + 1) . . . (kr + 1).

Soluo. Dena o conjunto

A1 ={todos os divisores de n que so da forma pm


1 , onde m Z },
1 +

e em geral, dena

Ai ={ todos os divisores de n que so da forma pm


i , onde t Z }.
i +

Observemos que mi pi , pois se mi > pi , ento pelo menos a potncia


pki i +1 deveria estar presente em (5.4);como isto no acontece segue-se
que mi pi , de modo que

Ai = p0i , p1i , p2i , . . . , pki i , para i = 1, 2, 3, . . . , ki .

imediato ver que Ai = ki + 1.
5.2 Princpio Multiplicativo de Contagem 177

O conjunto de todos os possveis divisores de n vem dado pelo


conjunto A1 A2 Ar , de onde se conclui que o nmero de
divisores inteiros e positivos de n
d(n) = |A1 A2 Ar | = |A1 | |A2 | |Ar |,
onde na ltima igualdade usamos o princpio multiplicativo. Portanto,
o nmero de divisores inteiros e positivos de n
d(n) = (k1 + 1)(k2 + 1) (kr + 1).

Exemplo 5.14. De quantas maneiras podemos escolher dois inteiros


de 1 a 20 de forma que a soma seja mpar?
Soluo. Observemos que
a soma de dois nmeros inteiros pares um nmero par. Com
efeito, para quaisquer a, b Z temos que 2a + 2b = 2(a + b);
a soma de dois nmeros inteiros mpares um nmero par. Com
efeito, para quaisquer a, b Z temos que (2a + 1) + (2b + 1) =
2(a + b + 1);

a soma de um nmero inteiro par com qualquer outro inteiro


mpar sempre um inteiro mpar. Com efeito, para quaisquer
a, b Z temos que 2a + (2b + 1) = 2(a + b) + 1.
Isto nos sugere denir os conjuntos
P = {2, 4, 6, 8, 10, 12, 14, 16, 18, 20},
I = {1, 3, 5, 7, 9, 11, 13, 15, 17, 19},
onde P I so todas as formas possveis de somar um nmero inteiro
par com outro mpar. O princpio multiplicativo nos garante que nossa
resposta |P I| = |P | |I| = 100, pois |P | = |I| = 10.
178 5 Contagem

5.3 Uso Simultneo dos Princpios Aditivo

e Multiplicativo

Aproveitamos esta seo para apresentar problemas um pouco mais


difceis que os tratados nas sees anteriores. Nestes problemas, pre-
cisaremos empregar simultaneamente o Princpio Aditivo e o princpio
multiplicativo. Vamos ao primeiro deles:

Exemplo 5.15. Sabemos que no incio da premiao da 1a fase da


Olimpada Alagoana de Matemtica existem 10 livros diferentes de
lgebra, 7 livros diferentes de combinatria e 5 livros diferentes de
geometria para homenagear os vencedores. Danielle a primeira a
pegar o prmio que consiste em 2 livros, com a condio de que estes
no podem ser da mesma matria. Diga quantas escolhas Danielle
pode fazer para pegar seu prmio.

Soluo. Denotemos por

A = {a1 , . . . , a10 }, C = {c1 , . . . , c7 } e G = {g1 , . . . , g5 },

os conjuntos de livros de lgebra, combinatria e geometria, respecti-


vamente. Observemos que |A| = 10, |C| = 7 e |G| = 5 e Danielle tem
as seguintes possibilidades de escolha:

escolher um livro de A e um livro de C . Neste caso, Danielle tem


|A C| = |A| |C| = 70 escolhas possveis (devido ao princpio
multiplicativo).

escolher um livro de A e um livro de G . Neste caso, Danielle tem


|A G| = |A| |G| = 50 escolhas possveis (devido ao princpio
multiplicativo) ou
5.3 Uso Simultneo dos Princpios Aditivo e Multiplicativo 179

escolher um livro de C e um livro de G . Neste caso, Danielle tem


|C G| = |C| |G| = 35 escolhas possveis (devido ao princpio
multiplicativo).
Agora o Princpio Aditivo nos garante que o nmero total de escolhas
que Danielle pode fazer 70 + 50 + 35 = 155.

Exemplo 5.16. H 18 moas e 12 rapazes, onde 5 deles so irmos


(3 moas e 2 rapazes) e os restantes no possuem parentesco. Diga
quantos casamentos so possveis naquela turma (sabendo que irmos
no se casam).
Soluo. Observemos que 15, entre as 18 moas, no tm parentesco
nenhum com os 12 rapazes, logo, pelo princpio multiplicativo temos
que possvel efetuar 15 12 = 180 casamentos diferentes entre eles.
Por outro lado, as 3 moas restantes podem efetuar casamento com 10
dos 12 rapazes, pois 2 deles so seus irmos. Novamente, pelo princpio
multiplicativo possvel realizar 310 = 30 casamentos diferentes neste
caso. Finalmente, o Princpio Aditivo nos d que podem ser realizados
um total de 180 + 30 = 210 casamentos.
Exemplo 5.17. Quantas palavras de 5 caracteres podem ser formadas
com as letras , e de modo que em cada palavra no falte nenhuma
dessas letras?
Soluo. Denamos os seguintes conjuntos,
U ={palavras de 5 caracteres s com as letras , e };
A ={palavras que esto em U e onde no aparece a letra };
A ={palavras que esto em U e onde no aparece a letra };
A ={palavras que esto em U e onde no aparece a letra }.
180 5 Contagem

Por exemplo,
a palavra A A ;

a palavra A ;

a palavra A .

Primeiramente, notemos que cada caracter de U pode ser escolhido


de 3 formas distintas. Segue-se ento do Princpio Multiplicativo que
existem 35 formas de escrever uma palavra de 5 caracteres usando um
alfabeto de 3 letras, isto ,

S0 = |U| = 35 = 243.

Calculemos agora |A |, isto , o nmero de palavras onde no apa-


rece a letra . Para isto, observemos que cada caractere em A pode
ser escolhido de 2 formas. Logo, o princpio multiplicativo nos garante
que existem 25 palavras em A , ou seja, |A | = 25 . Analogamente,
podemos mostrar que |A | = |A | = 25 . Portanto,

S1 = |A | + |A | + |A | = 25 + 25 + 25 = 96.

Prosseguimos com o clculo de |A A |, isto , do nmero de pala-


vras onde no aparecem as letras e ; portanto, cada caractere em
A A pode ser escolhido de 1 forma. Logo, o princpio multiplicativo
nos garante que existe 15 = 1 palavra em A A , ou seja, |A A | = 1.
Similarmente, podemos mostrar que |A A | = |A A | = 1. Portanto,

S2 = |A | + |A | + |A | = 3.

Por m, achamos |A A A |, que nos d o nmero de palavras onde


no aparecem as letras , e ; mas cada palavra em A A A tem
5.4 Permutaes Simples 181

que usar pelo menos um dos caracteres proibidos. Logo,

S3 = |A A A | = 0.

Finalmente, observamos que o conjunto das palavras de 5 caracte-


res que podem ser formadas com as letras , e de modo que em
cada palavra no falte nenhuma dessas letras exatamente o conjunto
Ac Ac Ac . Usando a Proposio 5.7, temos:

|Ac Ac Ac | =S0 S1 + S2 S3
=243 96 + 3 0
=150.

5.4 Permutaes Simples

Denimos o fatorial n! de um inteiro positivo n

n! = n (n 1) (n 2) 2 1

se n > 0 e 0! = 1, por conveno. Observe que o fatorial cresce muito


rapidamente quando n cresce. Por exemplo, para os 10 primeiros
valores de n

1!=1 2!=2 3!=6 4!=24 5!=120


6!=720 7!=5.040 8!=40.320 9!=362.880 10!=3.628.800

Denio 5.18. Uma permutao simples de n objetos distintos


qualquer agrupamento ordenado desses n objetos. Denotaremos por
Pn o nmero de todas as permutaes simples de n objetos dados.
182 5 Contagem

Por exemplo, todas as permutaes dos 3 elementos do conjunto


A = {a1 , a2 , a3 } so:

1 = (a1 , a2 , a3 ),
2 = (a1 , a3 , a2 ),
3 = (a2 , a1 , a3 ),
4 = (a2 , a3 , a1 ),
5 = (a3 , a1 , a2 ),
6 = (a3 , a2 , a1 ).

Proposio 5.19. Seja n 1. O nmero total de permutaes sim-


ples de n objetos O = {o1 , o2 , . . . , on } dado por Pn = n!
Demonstrao. claro que a frmula vale para n = 1. Vejamos agora
que existe a seguinte relao entre Pn e Pn1 para n 2:

Pn = nPn1 . (5.5)

Para comprovar isto, para cada i denamos Ai como sendo as permu-


taes dos n 1 objetos {o1 , . . . , oi1 , oi+1 , . . . , on }. Note que |Ai | =
Pn1 , para cada i = 1, 2, . . . , n. Assim, para obtermos uma permu-
tao dos n objetos, basta que xemos o objeto inicial oi e tomemos
um elemento do conjunto Ai , que uma permutao dos n 1 objetos
restantes. Pelo princpio aditivo, temos que:

Pn = |A1 | + |A2 | + + |An | = nPn1 .

Como a equao (5.5) vlida para todo n 2, podemos aplic-la


para n 1, obtendo:

Pn1 = (n 1)Pn2 ,
5.4 Permutaes Simples 183

de onde vem que


Pn = n(n 1)Pn2 .
Repetindo este argumento, obtemos que
Pn = n(n 1)(n 2) 3 2 1 = n!,

como queramos demonstrar.


Exemplo 5.20. De quantas maneiras podemos formar uma la com
4 pessoas?
Demonstrao. Observe que se enumeramos os lugares da la e enu-
meramos as pessoas, pa , pb , pc , pd , cada distribuio vai corresponder a
uma permutao do conjunto {1, 2, 3, 4}. Por exemplo, a distribuio
(pc , pa , pb , pd ) corresponde permutao (3, 1, 2, 4). Assim, o nmero
de distribuies na la 4! = 24.
Exemplo 5.21. De quantas maneiras k moas e k rapazes podem
formar pares para uma dana?
Soluo. Estando as moas em uma la e os rapazes em outra, pode-
mos enumer-los com nmeros de 1, 2, . . . , k . A uma permutao des-
ses nmeros, digamos (a1 , a2 , . . . , ak ) com ai {1, 2, . . . , k} faremos
uma associao da mulher i com o rapaz ai . Por exemplo, a permu-
tao (2, 1, 3, . . . , k) signica que a moa 1 danar com o rapaz 2, a
moa 2 com o rapaz 1, e a moa i com o rapaz i, para i 3.
Observe que toda associao de k moas e k rapazes produz uma
permutao, de modo que o nmero de associaes possveis das mo-
as com os rapazes igual ao nmero de permutaes dos elementos
do conjunto {1, 2, 3, . . . , k}. Pela Proposio 5.19 existem k! modos
diferentes de combinar as moas com os rapazes.
184 5 Contagem

5.5 Arranjos Simples

Denio 5.22. Consideremos n objetos e p um inteiro positivo tal


que 0 < p n. Um arranjo simples de classe p dos n objetos dados
uma seleo de p objetos distintos dentre estes que diferem entre si
pela ordem de colocao ou pela natureza de cada um, isto , o que
importa quem participa ou o lugar que ocupa. Denotaremos por Apn
o nmero de arranjos simples de classe p de n objetos.

Por exemplo, dados os objetos o1 , o2 e o3 todos os arranjos possveis


de classe 2 so: A1 = (o1 , o2 ), A2 = (o2 , o1 ), A3 = (o1 , o3 ), A4 =
(o3 , o1 ), A5 = (o2 , o3 ) e A6 = (o3 , o2 ).

Observao 5.23. Notemos que um arranjo simples de classe n de n


objetos dados no mais que uma permutao desses n objetos. Logo,
Pn = Ann = n!.

Proposio 5.24. Seja n 1. O nmero total de arranjos simples


de classe p de n objetos O = {o1 , o2 , . . . , on } dado por Apn = n!
(np)!
.

Demonstrao. Para n = 1 a frmula obviamente vlida. Similar-


mente ao caso das permutaes, primeiramente provaremos que para
n 2 vale a seguinte igualdade:

Apn = nAp1
n1 . (5.6)

Agora denimos os conjuntos Ai como sendo os arranjos simples de


classe p 1 dos n 1 objetos {o1 , . . . , oi1 , oi+1 , . . . , on }. Note que
n1 , para cada i = 1, 2, . . . , n. Assim, para obtermos um
|Ai | = Ap1
arranjo simples de classe p dos n objetos, basta que xemos o objeto
inicial oi e tomemos um elemento do conjunto Ai , que uma arranjo
5.5 Arranjos Simples 185

de classe p 1 dos n 1 objetos restantes. Pelo princpio aditivo,


temos que:

Apn = |A1 | + |A2 | + + |An | = nAp1


n1 .

Como nossa equao (5.6) vlida para todo n 2, podemos aplic-la


para n 1, obtendo:

Ap1 p2
n1 = (n 1)An2 ,

de onde vem que


Apn = n(n 1)Ap2
n2 .

Repetindo este argumento sucessivamente, obtemos que

p(p1)
Apn = n(n 1)(n 2) (n (p 2))An(p1)
= n(n 1)(n 2) (n p + 2)A1np+1 .

Notemos agora que A1np+1 = n p + 1; logo, da igualdade anterior


segue-se que

Apn = n(n 1)(n 2) (n p + 2)(n p + 1)


n(n 1)(n 2) (n p + 2)(n p + 1) (n p) 1
=
(n p) 1
n!
= ,
(n p)!

como desejvamos.

Agora vamos dar alguns exemplos de como aparecem problemas


prticos que requerem fazer este tipo de clculo. O primeiro dele tem
186 5 Contagem

a ver com a formao de palavras diferentes com um conjunto dado


de letras.
Um anagrama de uma palavra uma permutao de letras dessa
palavra para formar outra, a qual pode carecer de signicado. Por
exemplo:
um anagrama de amor roma;

um anagrama de celia alice;

um anagrama de caterina natercia;

um anagrama de elvis lives.


Exemplo 5.25. Quantos anagramas de p letras distintas podemos
formar com um alfabeto de 23 letras, sendo p < 23?
Soluo. Como as letras so diferentes, nosso problema consiste em
achar todos os arranjos de classe p de 23 objetos dados, que neste caso
so as 23 letras do alfabeto. Logo, este nmero
23!
Ak23 = .
(23 k)!

Exemplo 5.26. De quantos modos 2 pessoas podem se sentar em 5


cadeiras que esto em la?
Soluo. Este problema equivalente a achar o nmero total de ar-
ranjos de classe 2 de 5 objetos, correspondendo as 5 cadeiras aos 5
objetos e as duas pessoas indicando a ordem do arranjo. Logo, este
nmero dado por
5!
A25 = = 20.
3!
5.5 Arranjos Simples 187

Exemplo 5.27. Considere os dgitos 2, 3, 4, 5, 7 e 9. Supondo que a


repetio de dgitos no seja permitida, responda s seguintes pergun-
tas:

(a) Quantos nmeros de trs dgitos podem ser formados?

(b) Entre os achados em (a) quantos so pares?

(c) Entre os achados em (a) quantos so mpares?

(d) Entre os achados em (a) quantos so mltiplos de 5?

(e) Entre os achados em (a) quantos so menores do que 400?

Soluo. Seja O = {2, 3, 4, 5, 7, 9} nosso conjunto de objetos.

(a) A quantidade de nmeros de trs dgitos que podemos formar


sem repetio de algum deles claramente o nmero de arranjos
de classe 3 dos 6 dgitos de O, isto ,
6!
A36 = = 120.
3!

(b) Sabemos que em todo nmero par o ltimo dgito um mltiplo


de 2, isto , ele acaba em 0, 2, 4, 6 ou 8. Ento, em nosso caso
as nicas possibilidades so que o nmero termine em 2 ou 4.
Supondo que o ltimo dgito seja 2, temos que preencher as duas
casas restantes com os dgitos pertencentes ao conjunto O {2}.
Assim, existem A2|O{2}| = A25 = 5! 3!
= 20 nmeros dos achados
em (a) que nalizam em 2. De forma anloga, existem A2|O{4}| =
5!
A25 = 3! = 20 nmeros dos achados em (a) que nalizam em 4.
Logo, entre os nmeros achados em (a) existem 20 + 20 = 40
nmeros pares.
188 5 Contagem

(c) Todo conjunto de nmeros pode ser dividido em duas classes


disjuntas: a classe dos nmeros pares e a classe dos nmeros m-
pares que pertencem ao mesmo. Segue-se que dentre os nmeros
achados em (a) existem 120 40 = 80 nmeros mpares.

(d) Todo nmero mltiplo de 5 acaba em 0 ou 5; no nosso caso te-


mos que a nica possibilidade para o ltimo dgito 5. Assim
o problema consiste em preencher as duas casas restantes com
dgitos do conjunto O {5}. De onde se segue que a quanti-
dade de nmeros mltiplos de 5 existentes em (a) vem dada por
5!
A2|O{5}| = A25 = 3! = 20.

(e) Para obter os nmeros menores do que 400 a casa das centenas
s poder ser ocupada pelos dgitos 1, 2 ou 3. Como 1 / O,
temos que as nicas possibilidades em nosso caso so 2 ou 3.
Ento, supondo que o primeiro dgito do nmero seja 2, devemos
preencher duas casas restantes com os dgitos pertencentes a
O {2}. De forma anloga, existem A2|O{3}| = A25 = 5! 3!
= 20
nmeros dos achados em (a) e que comeam com 3. Logo, dentre
os nmeros achados em (a) existem 20 + 20 = 40 menores do que
400.

5.6 Combinaes Simples

O conceito de combinao simples surge naturalmente quando tenta-


mos responder seguinte pergunta: de quantas formas diferentes pode-
mos selecionar p objetos dentro de n objetos dados?
5.6 Combinaes Simples 189

Por exemplo, suponha que queremos enfeitar uma festa de aniver-


srio com bolas de dois tipos de cores e na loja onde as compraremos
existem bolas nas cores azul, verde e vermelha. De quantas formas
distintas podemos enfeitar nossa festa? claro que podemos enfeitar
a festa de 3 formas diferentes: com bolas em azul e verde; com bolas
em azul e vermelho ou com bolas em verde e vermelho.
Notemos que, ao contrrio do caso em que trabalhamos com arran-
jos, quando fazemos uma seleo de duas cores no estamos inte-
ressados na ordem em que elas foram escolhidas.

Denio 5.28. Consideremos n objetos e p um inteiro positivo tal


que 0 < p n. Uma combinao simples de classe p dos n objetos
dados uma seleo de p objetos distintos entre estes que diferem
entre si apenas pela natureza de cada um, isto , o que importa
simplesmente quem participa no grupo selecionado. Denotaremos por

n
p
o nmero de combinaes simples de classe p de n objetos.

Proposio 5.29. Seja n 1. O nmero total de combinaes


simples de classe p de n objetos O = {o1 , o2 , . . . , on } dado por
n
 n!
p
= p!(np)!
.

Demonstrao. Veremos a seguir que arranjos simples e combinaes


simples de classe p esto estreitamente relacionados. Com efeito, para
cada combinao simples formada por p objetos distintos de O pode-
mos gerar todos os arranjos simples de classe p formados por estes p
objetos. Basta para isto fazer todas as suas permutaes possveis.
Obtm-se assim p ! arranjos simples diferentes com esses p objetos.
Resumindo, para cada combinao simples de classe p formada com
p objetos diferentes de O podemos fazer p ! arranjos simples diferen-
tes de classe p com estes mesmos objetos; logo, no total, teremos a
190 5 Contagem

seguinte relao:  
n n!
p! = Apn = ,
p (n p)!
de onde segue-se que
 
n n!
= .
p p!(n p)!

Exemplo 5.30. De quantas formas diferentes podemos construir uma


palavra de tamanho n com i letras a e n i letras b?

Soluo. A soluo do problema equivale em escolher a posio das


i letras a em questo, uma vez que a posio das (n i) letras b
restantes estar determinada. Se enumeramos as posies das letras
de 1 a n, uma palavra ser formada ao xarmos a posio das i letras

a. Isso exatamente ni , j que corresponde ao nmero de grupos
com i elementos (posies com letra a) tomados em um conjunto de n
elementos (todas as posies), que diferem somente por sua natureza.

Exemplo 5.31. De quantas formas podemos dividir um grupo 5 pes-


soas em um grupo de duas e outro de trs?

Soluo. Temos 52 = 25!3! ! = 10 formas diferentes de escolher duas
pessoas do grupo. Por cada uma dessas escolhas o outro grupo de trs
pessoas automaticamente determinado; logo, temos 10 possibilidades
diferentes de fazer a diviso.

Exemplo 5.32. De quantos modos podemos dividir 6 pessoas em:


(a) Dois grupos de 3 pessoas cada?
5.6 Combinaes Simples 191

(b) Trs grupos de 2 pessoas cada?


Soluo. Comeamos por (a). primeira vista, parece que a resposta

deve ser n3 = 3!6!3! = 20, similarmente ao exemplo anterior. Porm,
aqui h um problema devido ao fato de estarmos dividindo em grupos
que tm a mesma quantidade de pessoas e, portanto, as permutaes
de cada dois grupos formados so consideradas divises iguais; logo,
devemos dividir o resultado por 2 !, obtendo assim 10 formas diferentes
de obter dois grupos com 3 pessoas cada.
Para resolver o item (b) seguimos os seguintes passos:
Primeiramente calcularemos o nmero de formas possveis para
dividir 6 pessoas em um grupo de 2 e outro grupo de 4; esta

quantidade vem dada por 62 = 4!6!2! .

Agora dividiremos as 4 pessoas restantes em um grupo de 2 e



outro grupo de 2; esta quantidade vem dada por 42 = 2!4!2! .
 
Pelo princpio multiplicativo temos que existem 62 42 = (2!)
6!
3 possi-

bilidades de dividir 6 pessoas em 3 grupos com duas pessoas cada.


Igualmente ao caso anterior, aqui as permutaes possveis de cada 3
grupos formados so consideradas iguais; logo, devemos dividir este
ltimo resultado por 3 !. Portanto, existem 15 formas diferentes de
dividir 6 pessoas em trs grupos de 2 pessoas cada.
Exemplo 5.33. Se voc possui 10 amigos, de quantas maneiras voc
pode escolher dois ou mais deles para jantar?
Soluo. Esquematizamos a soluo da seguinte maneira:
Primeiramente, vamos encontrar a quantidade de maneiras pelas

quais voc pode jantar com 2 amigos; isto feito de
10
2
formas
diferentes.
192 5 Contagem

Depois, vamos encontrar a quantidade de maneiras pelas quais



voc pode jantar com 3 amigos; isto feito de
10
3
formas dife-
rentes.

Em seguida, encontramos a quantidade de maneiras pelas quais



voc pode jantar com 4 amigos; isto feito de
10
4
formas dife-
rentes.

Em geral, o nmero de maneiras diferentes que voc tem de



jantar com p amigos dado por 10p .

Pelo princpio aditivo, temos que a quantidade de formas diferentes


que voc tem de jantar com 2 ou mais de seus amigos, dada por
       
10 10 10 10
+ + + + = 1013,
2 3 9 10

sendo este o nmero procurado.

Exemplo 5.34. De um grupo de 10 pessoas das quais 4 so mulheres,


quantas comisses de 5 pessoas podem ser formadas de modo que pelo
menos uma mulher faa parte?

Soluo. Sendo que o grupo tem 10 pessoas e 4 destas so mulheres,


segue-se que no grupo temos 6 homens. Para formar um grupo de 5
pessoas com pelo menos uma mulher, temos as seguintes alternativas:

Nosso grupo composto por uma mulher e 4 homens; neste caso


 
poderemos formar 41 64 = 60 comisses de 5 pessoas.

Nosso grupo composto por 2 mulheres e 3 homens; neste caso


 
poderemos formar 42 63 = 120 comisses de 5 pessoas.
5.7 O Binmio de Newton 193

Nosso grupo composto por 3 mulheres e 2 homens; neste caso


 
poderemos formar 43 62 = 60 comisses de 5 pessoas.

Nosso grupo composto por 4 mulheres e um homem; neste caso


 
poderemos formar 44 61 = 6 comisses de 5 pessoas.

Pelo princpio aditivo temos que possvel formar 246 comisses de 5


pessoas de modo que pelo menos uma mulher faa parte.

5.7 O Binmio de Newton

Nesta seo, estudaremos uma frmula que generaliza a conhecida


expresso
(a + b)2 = a2 + 2ab + b2 .

Essa frmula conhecida como o binmio de Newton ou frmula bino-


mial de Newton, devido ao Matemtico Isaac Newton (1642-1727). A
frmula binomial de Newton pode ser motivada pelas seguintes igual-
dades que so fceis de vericar:

   
1 1 1
(a + b) = a + b = a+ b,
0 1
     
2 2 2 2 2 2 2 2
(a + b) = a + 2ab + c = a + ab + b,
0 1 2
       
3 3 2 2 3 3 3 3 2 3 2 3 3
(a + b) = a + 3a b + 3ab + c = a + a b+ ab + b.
0 1 2 3

Os casos particulares acima podem ser estendidos para qualquer po-


tncia inteira positiva de a + b, ou seja, vale o seguinte resultado:
194 5 Contagem

Teorema 5.35 (Frmula Binomial de Newton) . Sejam a e b nmeros


reais e n N, ento
         
n n n n n1 1 n ni i n 1 n1 n n
(a+b) = a + a b + + a b + + a b + b .
0 1 i n1 n

Os nmeros ni , 0 i n, so chamados tambm de coecientes bino-
miais.
Demonstrao. Expandimos o binmio no produto de seus n fatores,
isto ,

(a + b)n = (a + b)(a + b) (a + b) . (5.7)


| {z }
nfatores

Se desenvolveremos o produto destes n fatores iguais acima obtemos


uma soma nita de termos da forma a1 a2 an , onde cada aj , 1
j n, toma valor a ou b. Notemos que em cada termo se o nmero
b aparece i vezes, ento o nmero a aparecer (n i) vezes, ou seja,
quando cada termo for multiplicado dever tomar valor igual a ani bi ,
para algum 1 i n. Por exemplo, os n termos
abb b = abn , bab b = abn , ..., bbb ba = abn

tm o mesmo valor . Assim, para calcular o coeciente do termo ai bni


que aparece na equao (5.7), basta responder seguinte pergunta: de
quantos modos podemos formar uma palavra com i letras a e (n i)
letras b? A resposta dessa pergunta foi estudada no Exemplo 5.30 e

simplesmente ni . Logo, a expresso na equao (5.7)
       
n n
n n n1 1 n 1 n1 n n
(a + b) = a + a b + + ab + b ,
0 1 n1 n
o que prova o teorema.
5.8 Contagem e Probabilidades 195

A frmula binomial de Newton nos d algumas propriedades interes-


santes dos coecientes binomiais que resumimos na prxima proposi-
o.
Proposio 5.36. Seja n N. As seguintes igualdades so vlidas:
    
(a) n
0
+ n
1
+ + n n
+ + n1
i
+ nn = 2n ;
   
(b) n0 n1 + + (1)i ni + + (1)n nn = 0.
Demonstrao. Para a letra (a), basta tomar a = b = 1 e expanda
2n = (1 + 1)n no Binmio de Newton. Para a letra (b), tome a = 1
e b = 1 e expanda 0 = (1 1)n no binmio de Newton, observando
que (1)n igual a 1 se n par, e igual a 1 se n mpar.

5.8 Contagem e Probabilidades

Uma das aplicaes interessantes da contagem de elementos de um


conjunto quando desejamos estudar a probabilidade de eventos alea-
trios. Por exemplo, se lanarmos um dado de seis faces, temos os
seguintes resultados possveis:

= {1, 2, . . . , 6}.

Se desejamos saber qual a chance de que ocorra um nmero


primo no lanamento, devemos contar quantos primos aparecem em
{1, 2, 3, 4, 5, 6} e dividir por 6. Ou seja, a chance de ocorrer um nmero
primo num lanamento de um dado de seis faces 3/6 = 0, 5.
Denimos a probabilidade de um subconjunto A como o n-
mero
|A|
p(A) = .
||
196 5 Contagem

Tambm chamamos o subconjunto de todos os resultados possveis


de espao amostral e um subconjunto A de de evento. Por exemplo,
podemos calcular a probabilidade de escolhermos um nmero par no
conjunto 1, 2, 3, . . . , 15. Neste caso, o conjunto est claro e igual a
= {1, 2, 3, . . . , 15}. O conjunto A A = {2, 4, 6, . . . , 14}. Logo,
|A| 7
p(A) = = .
|| 15
Assim, ca claro que a maior diculdade para calcular a proba-
bilidade de um evento contar quantos elementos pertencem a este
evento e quantos elementos pertencem ao espao amostral. A seguir,
veremos um exemplo mais elaborado onde aplicamos a noo de ar-
ranjo simples.

Exemplo 5.37. Calcular a probabilidade de que escolhendo um grupo


de 44 pessoas, existam pelo menos duas que fazem aniversrio no
mesmo dia do ano.
Soluo. Podemos reescrever isso do seguinte modo: num saco existem
bolas enumeradas com os nmeros 1, 2, . . . , 365 (correspondentes aos
dias do ano). Retiramos a bola b1 e anotamos o nmero que apareceu.
Devolvemos a bola ao saco e efetuamos uma nova retirada, anotando
novamente o nmero que aparece. Repetindo este processo 44 vezes,
obtemos uma lista com 44 nmeros. Assim, a pergunta se transforma
em: de quantos modos diferentes podemos escolher 44 bolas enume-
radas com os nmeros 1, 2, 3, . . . , 365 com reposio, tal que existam
pelo menos duas bolas com o mesmo nmero?
A primeira coisa que devemos fazer calcular o espao amostral,
de todas as possibilidades possveis de resultado. Como escolhemos
44 bolas enumeradas num saco, cada resultado possvel uma lista
5.9 Exerccios Propostos 197

(n1 , n2 , . . . , n44 ) com 44 nmeros. Observe que, pelo princpio multi-


plicativo, || = 36544 , pois temos 365 opes para escolher n1 , 365
opes para escolher n2 , etc.
A segunda pergunta trata-se de saber quantos resultados so favo-
rveis, ou seja, quantas so as escolhas tais que existam pelo menos
duas bolas com o mesmo nmero. Para isso mais fcil contar quantas
escolhas existem tais que os 44 nmeros so diferentes. Neste caso,
devemos escolher uma ordenao de 44 nmeros distintos entre 365.
Isso corresponde quantidade de arranjos de classe 44 num grupo de
365 elementos. Assim, conclumos que a probabilidade de que este
evento ocorra
365!
36544 A44
365 (365!44!)
p= =1 .
36544 36544
Obter um valor aproximado para o nmero acima com o computador
uma tarefa fcil nos dias atuais. Porm, aproximar expresses envolvendo
fatoriais (sem o uso do computador) um fato conhecido h muito tempo
pela humanidade, atravs da famosa frmula de Stirling.1 Com a ajuda
desta frmula, obtemos que p aproximadamente p = 0.93, como havamos
prometido no Captulo 1.

Alm dos exerccios abaixo, recomendamos a leitura de [9]. L, o


leitor encontrar material adicional sobre anlise combinatria, bem
como uma ampla variedade de problemas.

5.9 Exerccios Propostos

1. De quantas maneiras podemos escolher trs nmeros distintos do


conjunto I50 = {1, 2, 3, . . . , 49, 50} de modo que sua soma seja
1 Grosseiramente, a frmula de Stirling diz que o quociente entre n! e

2nn en n
est prximo de 1, para valores de n grandes.
198 5 Contagem

a) um mltiplo de 3?
b) um nmero par?
2. Considere o conjunto In = {1, 2, 3, . . . , n1, n}. Diga de quantos
modos possvel formar subconjuntos de k elementos nos quais
no haja nmeros consecutivos?
3. Considere as letras da palavra PERMUTA. Quantos anagramas
de 4 letras podem ser formados, onde:
a) no h restries quanto ao nmero de consoantes ou vogais?
b) o anagrama comea e termina por vogal?
c) a letra R aparece?
d) a letra T aparece e o anagrama termina por vogal?
4. Calcular a soma de todos os nmeros de 5 algarismos distintos
formados com os algarismos 1, 3, 5, 7 e 9.
5. Quantos nmeros podem ser formados pela multiplicao de al-
guns ou de todos os nmeros 2, 2, 3, 3, 3, 5, 5, 6, 8, 9, 9?
6. Entre todos os nmeros de sete dgitos, diga quantos possuem
exatamente trs dgitos 9 e os quatro dgitos restantes todos
diferentes?
7. De quantas maneiras podemos distribuir 22 livros diferentes en-
tre 5 alunos se 2 deles recebem 5 livros cada e os outros 3 recebem
4 livros cada?

8. Quantos so os nmeros naturais de sete dgitos nos quais o


dgito 4 gura exatamente 3 vezes e o dgito 8 gura exatamente
2 vezes?
5.9 Exerccios Propostos 199

9. De quantas maneiras uma comisso de 4 pessoas pode ser for-


mada, de um grupo de 6 homens e 6 mulheres, se a mesma
composta de um nmero maior de homens do que de mulheres?

10. O comprimento de uma palavra a quantidade de caracteres que


ela possui. Encontre a quantidade de palavras de comprimento
5 que podemos formar fazendo uso de 10 caracteres distintos, de
forma que no existam trs caracteres consecutivos idnticos em
cada palavra.

11. Quantos nmeros inteiros existem entre 1 e 10.000 que no so


divisveis por 3, 5 e 7?

12. Quantas so as permutaes da palavra PROPOR nas quais no


existem letras consecutivas iguais?

13. De quantos modos 6 casais podem sentar-se ao redor de uma


mesa circular de tal forma que marido e mulher no quem jun-
tos?

14. Quantas so as permutaes das letras da palavra BRASIL em


que o B ocupa o primeiro lugar, ou o R ocupa o segundo lugar,
ou o L o sexto lugar?

15. De quantas formas podemos representar o nmero 15 como soma


de vrios nmeros naturais?

16. Quantos quadrados perfeitos existem entre 40.000 e 640.000 que


so mltiplos simultaneamente de 3, 4 e 5?

17. Oito amigos vo ao cinema assistir a um lme que custa um real.


Quatro deles possuem uma nota de um real e quatro possuem
200 5 Contagem

uma nota de dois reais. Sabendo-se que o caixa do cinema no


possui nenhum dinheiro, como eles podem organizar uma la
para pagar o lme permitindo o troco pelo caixa?

18. Se considerarmos todas as conguraes do tabuleiro com duas


torres que no se atacam, como no Exemplo 5.2, sem distinguir
as torres, quantas conguraes obteremos?

19. Continuando o problema anterior, generalize-o para 3, 4, 5, . . .


torres que no se atacam, encontrando tambm o nmero mxi-
mo de torres que podem ser colocadas no tabuleiro de modo que
duas delas no se ataquem.

20. Tente fazer o problema anterior para cavalos de xadrez.

21. Mostre que em toda sequncia de n2 + 1 inteiros distintos possui


uma subsequncia crescente de n + 1 elementos ou uma sub-
sequncia decrescente de n + 1 elementos.

22. Encontre o nmero de zeros que termina o nmero 2010!.

23. O jogo do 7 consiste em lanar dois dados e somar o nmero


obtido nas suas faces. Caso a soma seja 7, o jogador A ganha o
dois reais do jogador B . Caso a soma no seja 7, o jogador B
ganha um real de A. Pergunta-se: quem leva vantagem?

24. A funo de Euler associa a cada nmero natural n o valor


(n) igual ao nmero de inteiros positivos menores ou iguais a
n relativamente primos com n. Ou seja,

(n) = {1 m n; (m, n) = 1} .
5.9 Exerccios Propostos 201

Usando os princpios estudados, mostre que se n se decompe


em fatores primos como n = p1 1 p2 2 . . . pk k , ento
    
1 1 1
(n) = n 1 1 ... 1 .
p1 p2 pk

O leitor pode achar mais informaes sobre a funo de Euler


nos livros [11] ou ainda [10].
202 5 Contagem
6
Induo Matemtica

Se as pessoas no a ham a Matemti a simples s por que ainda

no per eberam o quanto a vida ompli ada.

John von Neumann

Imagine uma la com innitos domins, um atrs do outro. Supo-


nha que eles estejam de tal modo distribudos que, uma vez que um
domin caia, o seu sucessor na la tambm cai. O que acontece quando
derrubamos o primeiro domin?
Apesar da simplicidade da pergunta acima ela traz em sua essncia
toda a ideia usada no mtodo da induo nita . Muitas descobertas
em Matemtica so feitas baseadas na realizao de testes que nos
fornecem evidncias empricas. Tais evidncias so estudadas para
efetivamente vericarmos se os resultados que elas insinuam so ver-
dadeiros. O mtodo da induo nita constitui uma ferramenta muito
til na hora de desvendar a veracidade de resultados provenientes deste
tipo de estudo. Esse mtodo uma das grandes armas do matemtico
moderno e tem utilidade na soluo de vrios problemas, como iremos
ver ao longo deste captulo.

203
204 6 Induo Matemtica

6.1 Formulao Matemtica

No incio do sculo XX, o matemtico Giuseppe Peano (1858-1932)


estabeleceu os axiomas necessrios que nos permitem hoje descrever
com preciso o conjunto dos nmeros naturais. O ltimo dos seus
axiomas diz o seguinte: seja A um subconjunto de N (A N). Se 1
A e se, alm disso, A contm todos os sucessores dos seus elementos,
ento A = N.
Este axioma conhecido como axioma de induo e serve como
base do mtodo de demonstrao por induo, o qual de grande
utilidade para estabelecer provas rigorosas em Matemtica.
O princpio da boa ordenao dos naturais, enunciado no Cap-
tulo 3, e o axioma de induo no so independentes e sem nenhuma
conexo. De fato, eles so equivalentes, ou seja, se consideramos o
princpio da boa ordenao como sendo um postulado podemos dedu-
zir o axioma de induo e, reciprocamente, se consideramos o axioma
de induo como sendo um postulado podemos deduzir o princpio da
boa ordenao.
No resto do captulo, p(n) representa uma armao em relao ao
natural n, podendo esta ser verdadeira ou falsa.

Teorema 6.1 (Princpio da Induo Finita) . Considere n0 um in-


teiro no negativo. Suponhamos que, para cada inteiro n n0 , seja
dada uma proposio p(n). Suponha que se pode vericar as seguintes
propriedades:

(a) p(n0 ) verdadeira;

(b) se p(n) verdadeira ento p(n + 1) tambm verdadeira, para


todo n n0 .
6.1 Formulao Matemtica 205

Ento, p(n) verdadeira para qualquer n n0 .

A armao (a) chamada de base da induo e a (b) de passo


indutivo. O fato de que p(n) verdadeira no item (b) chamado de
hiptese da induo.
Demonstrao. Denamos o conjunto

V = {m inteiros no negativos ; m n0 e p(m) verdadeira} .

Notemos que V no vazio, pois a condio (a) nos assegura que


n0 V . A prova do teorema equivalente a mostrarmos que

V = {n0 , n0 + 1, n0 + 2, n0 + 3, },

ou equivalentemente, a provarmos que o conjunto

F = {m inteiros no negativos ; m n0 e p(m) falsa}

vazio. Suponhamos que F no vazio. Pelo principio da boa orde-


nao existe um menor elemento m0 F , onde p(m0 ) falso. Obser-
vemos que,

m0 n0 + 1. De fato, m0 n0 , porm a possibilidade m0 = n0


contradiz a condio (a);

m0 1 V . Com efeito, p(m0 1) verdadeira pois, caso


contrrio, m0 1 F e, alm disso, m0 1 < m0 , contradizendo
isto a minimalidade de m0 .

Finalmente, como p(m0 1) verdadeira, segue da condio (b) que


p(m0 ) tambm verdadeira, o que impossvel pela denio de m0 .
Portanto, o conjunto F vazio, concluindo-se assim a prova.
206 6 Induo Matemtica

Para um pouco mais sobre a relao entre os princpios de induo


e da boa ordenao, recomendamos o Apndice A da referncia [11].
Observao 6.2. Uma grande vantagem do princpio da induo -
nita poder provar que uma quantidade innita de armaes so
verdadeiras, simplesmente vericando que uma quantidade nita des-
tas armaes so verdadeiras. Deixaremos clara a utilidade deste
mtodo resolvendo alguns problemas na prxima seo.

6.2 Aplicaes

Dentro da grande gama de problemas que podem ser abordados apli-


cando o mtodo de induo podemos distinguir trs importantes gru-
pos:
demonstrao de identidades;

demonstrao de desigualdades;

demonstrao de problemas de divisibilidade.

A seguir damos vrios exemplos de como aplicar o mtodo em


problemas referentes a cada um destes grupos.

6.2.1 Demonstrando Identidades

Comeamos com os seguintes problemas clssicos:

(P1) Determinar uma frmula para a soma dos n primeiros nmeros


pares, isto ,

sp (n) := 2 + 4 + 6 + + 2n.
6.2 Aplicaes 207

(P2) Determinar uma frmula para a soma dos n primeiros nmeros


mpares, isto ,

si (n) := 1 + 3 + 5 + + 2n 1.

Para induzir ambas as frmulas, primeiro fazemos os clculos para


vrios valores de n, os quais apresentamos na seguinte tabela.

n 1 2 3 4 5
sp (n) 2 = 1 2 6=23 12 = 3 4 20 = 4 5 30 = 5 6
si (n) 1 = 12 4 = 22 9 = 32 16 = 42 25 = 52

Os resultados na tabela sugerem que sp (n) = n(n + 1) e que


si (n) = n2 . Entretanto, isto no constitui por si s uma prova ri-
gorosa destas frmulas, pois para poder garantir a veracidade delas
utilizando a tabela teramos que vericar cada valor de n natural,
sendo isto impossvel. Provaremos agora que, de fato, as frmulas
induzidas so vlidas usando o mtodo de induo nita.

Exemplo 6.3. Demonstre que para qualquer n N vlida a igual-


dade:
2 + + 2n = n(n + 1).

Soluo. Denamos a proposio

p(n) : 2 + + 2n = n(n + 1)

e observemos que a mesma vale para n = 1 (base da induo); de fato

p(1) : 2 = 1(1 + 1).

Agora partimos para a prova do passo indutivo:


208 6 Induo Matemtica

Hiptese: suponhamos que p(k) verdadeira para um certo k >


1, k N.

Tese: devemos mostrar que p(k + 1) tambm verdadeira.

Com efeito, como

2 + + 2k = k(k + 1),
somando 2(k + 1) a ambos os lados desta igualdade, temos que

2 + + 2k + 2(k + 1) = k(k + 1) + 2(k + 1)


= (k + 2)(k + 1).

Esta ltima igualdade arma que p(k + 1) tambm verdadeira. O


Princpio de Induo nos garante que p(n) verdadeira para qualquer
n N.

Exemplo 6.4. Demonstre que para qualquer n N vlida a igual-


dade:
1 + 3 + 5 + + 2n 1 = n2 .

Soluo. Aqui denimos a proposio:

p(n) : 1 + 3 + 5 + + 2n 1 = n2

e notamos que a mesma vlida se tomarmos, por exemplo, n = 1.


De fato,
p(1) : 1 = 2 1 1.
Agora s resta provar o passo indutivo:
Hiptese: suponhamos que p(k) seja verdadeira para um certo
k > 1, k N.
6.2 Aplicaes 209

Tese: devemos mostrar que p(k + 1) tambm verdadeira.

Com efeito, como

1 + 3 + 5 + + 2k 1 = k 2 ,

somando 2k + 1 a ambos os lados desta igualdade, temos que

1 + 3 + 5 + + 2k 1 + 2k + 1 = k 2 + 2k + 1
= (k + 1)2 .

O princpio de induo nos garante que p(n) verdadeira para


qualquer n N.

Uma consequncia imediata do Exemplo 6.3 a frmula para a


soma dos n primeiros nmeros naturais, dada por
n(n + 1)
sn = 1 + 2 + 3 + + n = . (6.1)
2
Com efeito, como

2 + 4 + + 2n = n(n + 1),

ento dividindo por 2 ambos os membros da igualdade acima, obtemos


a equao (6.1).
Continuando com o mesmo raciocnio, natural nos perguntarmos
se possvel obter uma frmula para a soma dos n primeiros quadrados
perfeitos, ou seja, determinar qn onde:

qn = 12 + 22 + 32 + + n2 .

Para induzir a frmula, consideramos os valores de sn e qn numa tabela:


210 6 Induo Matemtica

n 1 2 3 4 5 6
sn 1 3 6 10 15 21
qn 1 5 14 30 55 91
Aparentemente no existe nenhuma relao entre sn e qn . Mas, se
considerarmos o quociente qn /sn , vejamos o que acontece:

n 1 2 3 4 5 6
qn /sn 3/3 5/3 7/3 9/3 11/3 13/3

Isso nos sugere que vale a relao


qn 2n + 1
= ,
sn 3
logo nosso candidato para valor de qn
sn (2n + 1) n(n + 1)(2n + 1)
qn = = .
3 6
Convidamos o leitor a provar a veracidade da equao acima utilizando o
Mtodo da Induo no Exerccio 1 no nal do captulo.

6.2.2 Demonstrando Desigualdades

Apresentamos agora alguns exemplos de como usar induo para provar


desigualdades.

Exemplo 6.5. Prove que 3n1 < 2n2 para todo n N.


Soluo. Denotamos por p(n) a propriedade: 3n1 < 2n . claro que p(1)
2

vlida, pois 1 < 2. Agora supondo que P (n) verdadeira temos que
2 2
3n = 3n1 3 < 2n 22n+1 = 2(n+1) ,

logo p(n + 1) tambm vale. Observamos que na desigualdade acima usamos


o fato de que 3 < 22n+1 para qualquer n N.
6.2 Aplicaes 211

Exemplo 6.6. Mostre que para todo nmero n N, n > 3, vale que 2n < n!
Demonstrao. Para n = 4 a desigualdade vericada, pois 24 = 16 < 4! =
24. Vamos assumir como hiptese de induo que a desigualdade vlida
para n 4. Ento, precisamos mostrar que a mesma vale tambm para
n + 1. De fato, por hiptese de induo:

2n < n! (6.2)

Como 2 < n + 1, podemos multiplicar o lado esquerdo da desigualdade


em (6.2) por 2 e o lado direito por n+1, sem alterar o sinal de desigualdade.
Logo, temos que:

2n .2 = 2n+1 < n!(n + 1) = (n + 1)!,

concluindo-se a demonstrao.

Exemplo 6.7. Prove que, para todo n N,


s r q

2+ 2+ 2 + + 2 < 2.
| {z }
nradicais

Demonstrao. Claramente a desigualdade vale para n = 1, pois 2 < 2.
Suponhamos que para certo n N a desigualdade acontece, ento
s r q

2+ 2 + 2 + + 2 < 2.
| {z }
nradicais

Logo, adicionando 2 em ambos os lados desta desigualdade tem-se


s r q

2+ 2+ 2 + 2 + + 2 < 2 + 2.
| {z }
nradicais
212 6 Induo Matemtica

Tomando raiz quadrada em ambos os lados desta ltima desigualdade ob-


temos v
u s r
u q
t
2 + 2 + 2 + 2 + + 2 < 2,
| {z }
n+1radicais

como desejvamos.

6.2.3 Induo e Problemas de Divisibilidade

Agora damos alguns exemplos de problemas de divisibilidade que podem


ser mostrados utilizando o mtodo da induo:

Exemplo 6.8. Mostre que para qualquer n N, n3 + 2n sempre divisvel


por 3.

Soluo. Para n = 1 a armao vlida, pois 13 +21 = 3, que obviamente


divisvel por 3.
Assumamos como hiptese indutiva que a armao vale para algum
k N, isto ,
Hiptese: k3 + 2k divisvel por 3.

Devemos mostrar que a armao tambm verdadeira para k + 1, ou


seja, temos que provar que

Tese: (k + 1)3 + 2(k + 1) divisvel por 3.

Para provar isto ltimo, usamos o fato de que

(k + 1)3 + 2(k + 1) = (k 3 + 3k 2 + 3k + 1) + (2k + 2);


6.2 Aplicaes 213

agrupando adequadamente,

(k + 1)3 + 2(k + 1) = (k 3 + 2k) + (3k 2 + 3k + 3)


= (k 3 + 2k) + 3(k 2 + k + 1)
| {z } | {z }
mltiplo de 3 mltiplo de 3

= mltiplo de 3,

concluindo assim a prova.


Exemplo 6.9. Mostre que a soma dos cubos de trs nmeros naturais con-
secutivos divisvel por 9.
Soluo. Denamos a seguinte proposio:
p(n) : n3 + (n + 1)3 + (n + 2)3 um mltiplo de nove.

Notemos que P (1) vlida, pois


13 + 23 + 33 = 1 + 8 + 27 = 36 = 9 4.

Precisamos provar agora o passo indutivo, isto ,


Hiptese: P (k) verdadeira para algum k N.

Tese: P (k + 1) tambm verdadeira.


Para provar isto, observamos que
(k + 1)3 + (k + 2)3 + (k + 3)3 = (k + 1)3 + (k + 2)3 + (k 3 + 9k 2 + 27k + 27).

Ordenando adequadamente, temos que o lado direito da ltima igualdade


se escreve como
k 3 + (k + 1)3 + (k + 2)3 + (9k 2 + 27k + 27)
= k 3 + (k + 1)3 + (k + 2)3 + 9(k 2 + 3k + 3)
| {z } | {z }
mltiplo de 9 mltiplo de 9

= mltiplo de 9,

completando assim nossa demonstrao.


214 6 Induo Matemtica

Muitas vezes, para conseguir mostrar que a hiptese p(n + 1) verda-


deira, precisamos supor que p(k) verdadeira para todo n0 k n. Isto
a base do princpio forte da induo nita que enunciamos a seguir:

Teorema 6.10 (Princpio Forte da Induo Finita). Considere n0 um in-


teiro no negativo. Suponhamos que, para cada inteiro n n0 seja dada
uma proposio p(n) e que valem as propriedades
(a) p(n0 ) verdadeira;

(b) se para cada inteiro no negativo k, com n0 k n, temos que p(k)


verdadeira, ento p(n + 1) tambm verdadeira.
Ento, a proposio p(n) verdadeira para qualquer n n0 .

Utilizando o princpio forte da induo, vamos dar uma prova diferente


do teorema fundamental da aritmtica da apresentada no Captulo 3.

Exemplo 6.11 (Teorema Fundamental da Aritmtica). Todo nmero na-


tural N maior que 1 pode ser escrito como um produto

N = p1 p2 p3 pm , (6.3)

onde m 1 um nmero natural e os pi , 1 i m so nmeros primos.


Alm disso, a fatorao em (6.3) nica se exigirmos que p1 p2
pm .

Soluo. Para cada n N, n 2, denamos a proposio

p(n) : n escrito de modo nico como um produto de nmeros primos.

Notemos que p(2) verdadeira, pois 2 um nmero primo.


Agora enunciemos o passo indutivo:

Hiptese indutiva: p(k) verdade para cada inteiro k tal que 2 k


n.
6.3 Induo na Geometria 215

Tese: p(n + 1) verdade. Em outras palavras, temos que mostrar que


n + 1 escrito de modo nico como um produto de nmeros primos.

Faremos a prova dividindo em dois casos:


(a) Se n + 1 um nmero primo, ento p(n + 1) verdade e isto acaba
nossa demonstrao.
(b) Se n + 1 no um nmero primo, ento existem , N com 2
n e 2 n tais que n + 1 = .
Nossa hiptese indutiva vlida para e . Isto signica que se
escreve de modo nico como um produto de nmeros primos e que
se escreve de modo nico um produto de nmeros primos. Portanto,
n + 1 = se escreve como um produto de nmeros primos.
Agora mostraremos que n + 1 se escreve de modo nico como produto
de primos. Assuma que
p1 p2 . . . pk = q1 q2 . . . qm = n + 1, (6.4)
com p1 p2 pk e q1 q2 qm todos primos. Vamos
mostrar que necessariamente k = m e pi = qi .
De fato, como p1 primo, ele divide algum qi . Logo, como qi primo,
p1 = qi q1 . Analogamente, existe um j tal que q1 = pj p1 . Logo,
p1 = q1 . Cancelando p1 em ambos os lados da equao (6.4), temos
que (n + 1)/p1 = p2 . . . pk = q2 . . . qm n. Logo, por hiptese de
induo, k = m e p2 = q2 , . . . , pm = qm , encerrando a demonstrao.

6.3 Induo na Geometria

Tratamos aqui alguns exemplos que mostram a utilidade do mtodo de


induo na resoluo de problemas geomtricos. Vamos comear estudando
216 6 Induo Matemtica

duas propriedades importantes dos polgonos. A primeira delas trata da


soma dos ngulos internos de um polgono convexo de n lados (n-gono).
Um polgono convexo um polgono tal que qualquer segmento de reta
que liga dois de seus pontos est contido no interior dele. No caso de
polgonos, isto equivalente ao fato de que todo segmento que liga dois
vrtices ou uma aresta ou est contido no interior do polgono.

Exemplo 6.12. Mostre que a soma dos ngulos internos de um polgono


convexo de n lados (n 3) igual a (n 2) radianos.

Soluo. No caso de n = 3 a propriedade acima muito bem conhecida.


Desde Tales de Mileto e Euclides se conhecia que a soma dos ngulos internos
de um tringulo radianos. Faamos mais um caso, tomando n = 4. Neste
caso, podemos dividir um quadriltero em dois tringulos, como mostra a
Figura 6.1 (a). Assim, a soma dos ngulos internos de um quadriltero
2 radianos.

A4 A4
A3

A5 A3

A1 A2 A1 A2
(a) (b)

Figura 6.1: Dividindo polgonos

Para elucidar o processo de induo e no deixar dvidas sobre o que


iremos fazer, vamos considerar mais um polgono, o pentgono (n = 5).
Neste caso, para mostrar que a soma dos seus ngulos internos (5 2) =
3 radianos, iremos dividir o pentgono A1 A2 A3 A4 A5 em um quadriltero
A1 A2 A3 A4 e um tringulo A1 A4 A5 , como mostra a Figura 6.1 (b). Assim,
6.3 Induo na Geometria 217

a soma dos ngulos internos do pentgono A1 A2 A3 A4 A5 igual soma dos


ngulos internos do tringulo A1 A4 A5 (igual a ) mais a soma dos ngulos
internos do quadriltero A1 A2 A3 A4 (igual a 2 ), ou seja, igual a 3 .
Finalmente, vamos assumir como hiptese de induo que para um certo
n 3 mostramos que a soma dos ngulos internos do n-gono dada pela
expresso (n 2) . Precisamos mostrar que a soma dos ngulos internos
de um n + 1-gono [(n + 1) 2] = (n 1) . De fato, podemos repetir
o processo anterior. Vamos denominar de A1 , A2 , . . . , An , An+1 os vrtices
consecutivos do (n + 1)-gono. Podemos dividi-lo no n-gono A1 A2 . . . An e
no tringulo A1 An+1 An . Logo, a soma dos ngulos internos do (n+1)-gono
(n 2) + = (n 1) .
Exemplo 6.13. Mostre que o nmero de diagonais de um polgono convexo
n(n3)
de n-lados igual a 2 .
Soluo. Observe que para n = 3 temos que existem 0 = 3.(3 3)/2 dia-
gonais num tringulo. Para n = 4, temos 2 = 4(4 3)/2 diagonais num
quadriltero convexo (veja a Figura 6.2).
Vamos agora assumir como hiptese de induo que se n um n-
gono convexo ento o seu nmero de diagonais n(n 3)/2 e vamos pro-
var que a frmula vale para um (n + 1)-gono convexo. De fato, denote
por A1 , A2 , . . . , An , An+1 os vrtices consecutivos do n + 1-gono. Pode-
mos decomp-lo como a unio do n-gono A1 , A2 , . . . , An e do tringulo
A1 , An , An+1 . Neste caso, para contarmos as diagonais do (n + 1)-gono
devemos considerar os seguintes casos:
Diagonais do n-gono A1 , A2 , . . . , An ; por hiptese de induo, o n-
mero dessas diagonais n(n 3)/2.
n 2 diagonais que partem do vrtice An+1 mais a diagonal A1 An .
Assim, o nmero total de diagonais do (n + 1)-gono
n(n 3) n2 3n + 2n 2 n2 n 2 (n + 1)(n 2)
+ (n 2) + 1 = = = ,
2 2 2 2
218 6 Induo Matemtica

como queramos demonstrar.

A4 A4
A3

A5 A3

A1 A2 A1 A2
(a) (b)

Figura 6.2: Diagonais de polgonos

Exemplo 6.14. Mostre que podemos cobrir os n2 pontos no reticulado a


seguir traando 2n 2 segmentos de reta sem tirar o lpis do papel.


| {z }
nnpontos

Figura 6.3: O problema de bar n n


6.3 Induo na Geometria 219

Soluo. O caso n = 3 j foi enunciado no Problema 1.12 do Captulo 1. A


gura a seguir mostra a soluo, onde o caminho realizado com as 4 linhas
o seguinte: A B C D B .


D B

Figura 6.4: Soluo do problema de bar 3 3

Daremos a prova do problema acima por induo. Para isso, veja que
podemos resolver o caso n = 4 continuando a soluo do caso n = 3. Como
paramos num dos vrtices do quadrado 33, acrescentamos mais uma linha
e uma coluna para obter um reticulado 4 4. Assim, conseguimos cobrir os
16 pontos utilizando 4 + 2 = 6 linhas, sem tirar o lpis do papel e cobrindo
dois lados do quadrado, como mostram as linhas descontnuas na Figura
6.5.

C
A


D B

Figura 6.5: Completando o reticulado


220 6 Induo Matemtica

Finalmente, vamos assumir como hiptese de induo que podemos co-


brir n 2 um reticulado n n com 2n 2 linhas, sendo que a ltima delas
cobre um dos lados do reticulado. Acrescentando 2n+1 pontos como mostra
a Figura 6.5, obtemos um reticulado (n + 1) (n + 1) que pode ser coberto
com 2n 2 + 2 = 2(n + 1) 2 pontos, como queramos demonstrar.

6.4 Miscelnea

Nesta seo discutiremos alguns exemplos interessantes de como podemos


aplicar o mtodo da induo aos mais variados tipos de problemas. O
primeiro deles uma generalizao do Problema 1.8.

Exemplo 6.15 (A Moeda Falsa). Um rei muito rico possui 3n moedas de


ouro. Porm, uma destas moedas falsa e seu peso menor que o peso das
demais. Com uma balana de 2 pratos e sem nenhum peso, mostre que
possvel encontrar a moeda falsa com apenas n pesagens.

Soluo. Para resolver este problema, vamos utilizar o Mtodo da Induo.


De fato, se n = 1, procederemos da seguinte forma: pegamos duas moedas
quaisquer e colocamos na balana, deixando uma do lado de fora. Caso a
balana se equilibre, a moeda que est do lado de fora necessariamente a
que tem menor peso. Caso a balana se desequilibre, a que tem menor peso
est na balana, no prato mais alto. O caso n = 2 foi feito no Problema
1.8.
Vamos agora assumir como hiptese de induo que dadas 3n moedas,
podemos achar a moeda mais leve com n pesagens. Vamos mostrar que
para 3n+1 moedas, suciente n + 1 pesagens. De fato, dividiremos as
3n+1 moedas em 3 grupos, A, B e C com 3n moedas cada. Colocamos na
balana os grupos A e B . Caso os dois grupos se equilibrem, a moeda mais
leve est no grupo C . Caso o grupo A esteja mais leve, a moeda mais leve se
encontra no grupo A. De qualquer modo, com uma pesagem conseguimos
6.4 Miscelnea 221

determinar em qual grupo de 3n elementos a moeda mais leve se encontra.


Por hiptese de induo, precisamos de mais n pesagens para encontrar a
moeda mais leve, totalizando n + 1 pesagens. Desaamos o leitor a mostrar
que no possvel realizar tal tarefa com menos de n pesagens.

Exemplo 6.16. Mostre que utilizando um balde com 5 litros de capacidade


e outro com 7 litros, possvel separar qualquer quantidade superior ou igual
a 24 litros.

Soluo. Novamente, faremos a prova utilizando o Mtodo da Induo.


Neste caso, comearemos o processo de induo a partir de 24. De fato,
podemos separar 24 litros utilizando duas vezes o balde de 7 e duas vezes o
balde de 5 litros. Note que o problema acima equivale a mostrar que

Todo nmero maior ou igual a 24 pode ser escrito da forma


7x + 5y , onde x e y so nmeros inteiros maiores ou iguais a
zero.

Neste caso, escrevemos 24 como 24 = 2 7 + 2 5. Por hiptese de


induo, vamos supor que conseguimos escrever um nmero n 24 como
n = 7x + 5y , com x e y nmeros inteiros maiores ou iguais a zero. Devemos
mostrar que n + 1 se escreve deste modo tambm. Para isso, vamos dividir
a anlise em dois casos:
Caso 1: y3
Logo, x 2 pois se isso no ocorresse, teramos 7x + 5y 22 < 24, o
que impossvel. Assim, podemos escrever:

n + 1 = 7x + 5y + 1 = 7(x 2) + 5(y + 3),

pois x 2 0.
Caso 2: y4
222 6 Induo Matemtica

Neste caso, y 4 0. Logo, podemos escrever:

n + 1 = 7x + 5y + 1 = 7(x + 3) + 5(y 4),

nalizando a nossa prova por induo.

6.4.1 Cuidados ao Usar o Princpio da Induo

Observao 6.17. Quando aplicamos o princpio da induo devemos to-


mar certos cuidados. A seguir damos um exemplo de como o mtodo pode
ser aplicado de forma errada. Vamos mostrar a seguinte armao:

Armao: Num conjunto qualquer de n bolas, todas as bolas


possuem a mesma cor.

Observe que nossa proposio claramente falsa. Mas, mesmo assim, vamos
dar uma prova por induo.
Para n = 1, nossa proposio verdadeira pois em qualquer conjunto
com uma bola, todas as bolas tm a mesma cor, pois s existe uma bola. As-
suma por hiptese de induo que a proposio verdadeira para n e prove-
mos que a proposio verdadeira para n+1. Ora, seja A = {b1 , . . . , bn , bn+1 }
o conjunto com n + 1 bolas referido. Considere os subconjuntos de B e C
de A com n elementos, construdos como:

B = {b1 , b2 , . . . , bn } e C = {b2 , . . . , bn+1 }

Observe que ambos os conjuntos tm n elementos. Assim, as bolas


b1 , b2 , . . . , bn do conjunto B tm a mesma cor. Do mesmo modo, as bo-
las do conjunto C tm a mesma cor. Em particular, a bola bn tem a mesma
cor da bola bn+1 . Assim, todas as bolas tm a mesma cor. Ache o erro no
argumento! Se voc no conseguir, leia a nota de rodap. 1
1 Uma dica da soluo encontra-se no nal do captulo.
6.5 Induo e Recorrncias 223

6.5 Induo e Recorrncias

Vamos comear esta seo discutindo um problema muito conhecido e inte-


ressante.

Exemplo 6.18 (As Torres de Hani2 ). Diz uma antiga lenda que na origem
dos tempos, em um templo de Hani, foram colocados 64 discos perfurados
de ouro puro e de dimetros diferentes ao redor de uma de trs hastes de
diamante. Muitos sacerdotes moviam os discos, respeitando as seguintes
regras: eles comeam empilhados em ordem crescente de acordo com seu
tamanho (ver Figura 6.6). Os discos podem ser deslocados de uma coluna
para qualquer outra, sendo que nunca pode ser colocado um disco maior em
cima de um menor e a cada segundo os sacerdotes movem um disco.
Quando os sacerdotes transportassem todos os discos de uma coluna para
outra, o mundo se acabaria. Suponha que eles comearam esse processo no
ano 2000 e que a lenda verdadeira, quanto tempo ainda resta para a Terra?

Figura 6.6: Torre de Hani

Para responder esse problema, consideraremos o problema geral de des-


cobrir quantos movimentos so necessrios para mover n anis de uma haste
para outra. Argumentaremos do seguinte modo: observe que podemos mover
os discos para outra haste se n = 1 ou 2. Com efeito, se temos somente um
anel basta mover este para qualquer outra haste com um nico movimento.
2 Este jogo foi inventado, em 1882, pelo matemtico Francs douard Lucas.
224 6 Induo Matemtica

Se temos 2 anis ento movemos o menor deles para a segunda haste, o


maior para a terceira haste e, nalmente, o menor para a terceira haste,
realizando um total de 3 movimentos. Para calcular o caso geral, vamos
empregar um mtodo chamado de mtodo recursivo: o nmero ak+1 de mo-
vimentos necessrios para mover k + 1 anis ser expresso como uma funo
de ak .
De fato, se temos k + 1 anis na primeira haste e sabemos mover k anis
de uma haste para outra utilizando ak movimentos, ento podemos mover
todos os k + 1 anis para a segunda haste usando 2ak + 1 movimentos.
De fato, movemos todos eles, exceto o maior, para a terceira haste usando
ak movimentos. A seguir, colocamos o maior na segunda haste usando 1
movimento. Imediatamente, deslocamos todos os anis da terceira haste
para a segunda haste usando mais ak movimentos. Logo, movemos todos os
k + 1 anis utilizando 2ak + 1 movimentos. Em resumo:

ak+1 = 2ak + 1, (6.5)

onde ak o nmero de movimentos necessrios para mover k discos de uma


haste para outra. Vamos agora usar induo para provar que ak = 2k 1.
Uma vez constatada a veracidade da armao para k = 1, 2, para
calcular ak , por hiptese de induo, vamos assumir que ak = 2k 1. Temos
pela equao (6.5):

ak+1 = 2ak + 1 = 2(2k 1) 1 = 2k+1 1.

como queramos demonstrar.


Vamos aproveitar o Exemplo 6.18 para discutir algumas equaes que
aparecem em muitas situaes em Matemtica: as equaes de recorrncia.
Em geral, uma equao de recorrncia uma equao envolvendo uma
certa quantidade de termos de sequncia xn . Para ilustrar isso, observe
a equao (6.5). Aqui, estaremos interessados em um tipo particular de
equao de recorrncia, as equaes de recorrncia lineares.
6.5 Induo e Recorrncias 225

Denio 6.19. Uma equao de recorrncia linear de grau k uma ex-


presso da forma:

xn+1 =rk1 xn + rk2 xn1 + + r0 xnk+1


(6.6)
x1 = a1 , x2 = a2 , . . . , xk = ak ,

onde r0 , r1 , . . . , rk1 so nmeros reais e r0 6= 0.

Por exemplo, so equaes de recorrncia lineares as seguintes equaes


2
2xn 3xn+1 = 0 e 3xn + xn+1 = 5xn+2
3
e no so equaes de recorrncia lineares as equaes
2
2(xn )3 5xn+1 = 0 e 3xn + xn+1 = 5xn+2 + 3.
3
Exemplo 6.20 (Sequncia de Fibonacci). Um exemplo muito interessante
de equao de recorrncia a sequncia conhecida por sequncia de Fibo-
nacci, devido ao matemtico italiano Leonardo di Pisa (1170-1250). Esta
sequncia adquiriu muita fama devido a suas conexes com reas das mais
variadas na cultura humana. Ela aparece em problemas de Biologia, Ar-
quitetura, Engenharia, Fsica, Qumica e muitos outras reas da cincia e
arte.
Denimos a sequncia de Fibonacci como sendo a sequncia Fn que
satisfaz a seguinte equao de recorrncia:

F1 = 1;
F2 = 1;
Fn = Fn1 + Fn2 , se n 3.

Agora vamos utilizar induo para mostrar algumas de suas proprieda-


des.
226 6 Induo Matemtica

Exemplo 6.21. Considere Fn a sequncia de Fibonacci. Mostre que


 n
7
Fn < .
4
n
Soluo. Denamos a proposio p(n) := Fn < 47 . Para n = 1 temos
que F1 = 1 < 74 , de modo que p(1) verdadeira. Suponhamos que

p(1), p(2), . . . , p(n), n 2,



7 n+1
sejam todas verdadeiras. Mostraremos que Fn+1 < 4 . Com efeito,

7 n
n1
Fn+1 = Fn + Fn1 < 4 + 47
n1 n1
< 74 74 + 74
 n1
< 1 + 74 74 .
 
7 2

7 2

7 n1
Como 1 + 7
4 < 4 , segue-se que Fn+1 < 4 4 . Portanto,

7 n+1
Fn+1 < 4 .

Exemplo 6.22. Dada a seguinte relao de recorrncia

a0 = 8;
a1 = 10;
an = 4an1 3an2 , n 2.

Mostre que an = 7 + 3n , para todo n Z+ .

Soluo. Denamos a proposio P (n) : an = 7 + 3n . P (0) verdadeira,


pois P (0) = 7 + 30 = 7 + 1 = 8. Suponhamos que P (k) verdadeiro para
cada inteiro k tal que 1 k n. Vamos mostrar que P (k) verdade para
6.5 Induo e Recorrncias 227

k = n + 1. Com efeito,
an+1 = 4an 3an1
= 4(7 + 3n ) 3(7 + 3n1 )
= 7 + 4 3n 3 3n1

= 7 + 3n1 4 3 3

= 7 + 3n1 9 = 7 + 3n1 32
= 7 + 3n+1 .

Vamos agora discutir o caso geral da equao de recorrncia linear (6.6).


Para isso, vamos fazer algumas observaes preliminares que deixaremos a
cargo do leitor:
se an e bn so solues da equao (6.6), ento an + bn tambm
soluo;
se an soluo da equao (6.6) e um nmero real, ento an
tambm soluo.
Com isto em mente, vamos descrever agora como obter todas as solues
xn da equao (6.6) em funo de n. Observe que dados os termos iniciais
a1 , a2 , . . . , ak a sequencia xn ca inteiramente determinada pela equao de
recorrncia. O interessante aqui determinar o termo xn+1 sem que seja
preciso o clculo dos termos xn , xn1 , . . . , xnk+1 .
Vamos primeiro procurar o que se chama de soluo particular da equa-
o (6.6). Particular porque ela assume uma forma caracterstica e porque
no assumiremos que as condies x1 = a1 , . . . , xk = ak valham.
Vamos procurar solues do tipo xn = n , onde um nmero real
positivo. Neste caso, temos que:
n+1 = xn+1 =rk1 xn + rk2 xn1 + + r0 xnk+1
= rk1 n + rk2 n1 + + r0 nk+1 .
228 6 Induo Matemtica

Passando os termos do lado direito da igualdade e colocando em evi-


dncia o termo nk+1 temos:


nk+1 k rk1 k1 rk2 k2 r0 = 0. (6.7)

Assim, como k 6= 0, pois > 0, temos que

k rk1 k1 rk2 k2 r0 = 0. (6.8)

O polinmio

p() = k rk1 k1 rk2 k2 r0

recebe o nome especial de polinmio caracterstico da equao de recorrn-


cia (6.6). Acabamos de mostrar que qualquer raiz do polinmio caracters-
tico gera uma soluo particular da equao (6.6).
Vamos assumir que a equao (6.8) possui k razes diferentes, digamos
1 > 2 > > k . Ento vale o seguinte teorema:

Teorema 6.23. Se escolhermos nmeros reais c1 , c2 , . . . , ck , ento

xn = c1 n1 + c2 n2 + + ck nk (6.9)

uma soluo da equao de recorrncia, onde os termos iniciais ai para


i = 1, 2, . . . , k so:

ai = c1 i1 + c2 i2 + + ck ik .

Demonstrao. Para mostrar o teorema, como x1 = a1 , . . . xk = ak pela


denio dos ai 's, basta mostrar que xn uma soluo.
Ora, o produto de uma soluo por um nmero real tambm uma
soluo. Assim, como ni uma soluo para i = 1, 2, . . . , k e ci um
6.6 Exerccios 229

nmero real, temos que ci ni soluo para i = 1, 2, . . . , k. Como j vimos


acima, a soma de solues tambm uma soluo. Logo,

xn = c1 n1 + c2 n2 + + ck nk

uma soluo.

Neste ponto, voltamos a equao (6.6). Desde o princpio, dados os


nmeros ai buscvamos a soluo xn tal que x1 = a1 , . . . , xk = ak . A
Equao (6.9) nos d uma variedade de solues, onde podemos escolher
os nmeros ci como bem entendermos. Usando equaes lineares, podemos
mostrar que sempre possvel escolher os nmeros ci de modo que x1 =
a1 , . . . , xk = ak . Isso encerra nossa busca. Para complementar esta seo,
recomendamos a leitura do Captulo 3 de [4].

6.6 Exerccios

1. Se qn denota a soma qn = 12 + 22 + + n2 , prove que para todo


nN
n(n + 1)(2n + 1)
qn = .
6
2. Use o princpio da induo para provar as seguintes armaes:

(a) 3n+1 + 2n+2 divisvel por 7 para todo n N;


(b) a soma dos cubos de trs nmeros naturais consecutivos divi-
svel por 9;
(c) 7 + 77 + 777 + + 777
| {z. . . 7} = 7(10n+1 9n 10)/81;
nvezes

(d) (n + 1)(n + 2) . . . (n + n) = 2n 1 3 5 (2n 1).

3. Use o princpio da induo para provar as seguintes desigualdades:


230 6 Induo Matemtica

(a) 2n1 (an + bn ) > (a + b)n , n N, com a, b R, a + b > 0 e


a 6= b;
1 1 1 1
(b) + + + + > n, para todo n N;
1 2 3 n
1 1 1 1 13
(c) + + + + > , para todo n N.
n+1 n+2 n+3 2n 24
4. Mostre a seguinte identidade trigonomtrica
(n + 1) cos nx n cos(n + 1)x 1
cos x + 2 cos 2x + + n cos nx = .
4 sin2 x2

5. Um torneio de xadrez tem n jogadores. Cada jogador joga uma nica


partida com cada um dos outros jogadores. Calcule o nmero total
de partidas realizadas no torneio.

6. Demonstre que para qualquer n N vlida a igualdade


 2
3 3 3 n(n + 1)
3
1 + 2 + 3 + + n = .
2

7. Demonstre que para qualquer n N valida desigualdade


 
1 n
an = 1 + < 3.
n

8. Prove que, para todo n N e a > 0,


s r q
1 + 4a + 1
a+ a + a + + a < .
2
| {z }
nradicais

9. Mostre que para qualquer nmero natural n 0, 11n+2 + 122n+1


sempre divisvel por 133.

10. Mostre que para todo n Z+ temos que 32n+1 + 2n+2 um mltiplo
de 7.
6.6 Exerccios 231

11. Mostre que para todo n Z+ temos que 32n+2 + 26n+1 um mltiplo
de 11.

12. Considere Fn a sequncia de Fibonacci . Mostre que


!n !n
1 1+ 5 1 1 5
Fn = .
5 2 5 2

13. Mostre as seguintes propriedades a respeito da sequncia de Fibonacci


Fn :
n
X n
X
(a) Fi = Fn+2 1; (b) F2i1 = F2n ;
i=1 i=1
Xn
(c) F2i = F2n+1 1; (d) Fn1 Fn+1 Fn2 = (1)n .
i=1

14. De quantas formas diferentes podemos cobrir um tabuleiro de 2


n com peas de domins que cobrem exatamente duas celas do ta-
buleiro?

15. Calcular o nmero de regies em que o plano dividido por n retas


distintas em cada uma das seguintes situaes:

(a) as n retas so concorrentes;


(b) no existem duas retas paralelas nem trs retas concorrentes.3

16. Dizemos que uma gura enquadrvel com rgua e compasso, se a


partir dela possvel, utilizando apenas rgua e compasso, construir
um quadrado de mesma rea. Prove que:
3 At
onde sabemos, este problema conhecido como a pizza de Steiner, o qual
foi resolvido, em 1826, pelo notvel gemetra Jacob Steiner (1796-1863).
232 6 Induo Matemtica

(a) um tringulo sempre enquadrvel;


(b) um polgono qualquer enquadrvel.

Sugesto para o item (b): Utilize induo dividindo a gura em tri-


ngulos.

17. D uma resposta situao Observao 6.17.


Sugesto: Observe a validade do argumento quando o conjunto A tem
2 elementos. Veja que B e C no se intersectam. Ou seja, o passo
indutivo falha de n = 1 para n = 2.
Referncias Bibliogrcas
[1] AIGNER, M. e ZIEGLER, G. (2002). As Provas esto no
Livro. Edgard Blcher.

[2] GARCIA, A. e LEQUAIN, I. (2003). Elementos de lgebra.


Projeto Euclides, IMPA.

[3] LIMA, E. L.; CARVALHO, P. C. P.; WAGNER, E. e MOR-


GADO, A.C. (2004). A Matemtica do Ensino Mdio. Volume
1. Sociedade Brasileira de Matemtica.

[4] LIMA, E.L.; CARVALHO, P. C. P.; WAGNER, E. e MOR-


GADO, A.C. (2004). A Matemtica do Ensino Mdio. Volume
2. Sociedade Brasileira de Matemtica.

[5] LIMA,E.L.; CARVALHO,P. C. P.; WAGNER,E. e MOR-


GADO,A.C. (2004). A Matemtica do Ensino Mdio. Volume
3. Sociedade Brasileira de Matemtica.

[6] LIMA, E.L.; CARVALHO, P. C. P.; WAGNER,E. e MOR-


GADO, A.C. (2001). Temas e Problemas. Sociedade Brasileira
de Matemtica.

[7] LIMA, E.L. (2001). lgebra Linear. Sociedade Brasileira de


Matemtica.

285
286 REFERNCIAS BIBLIOGRFICAS

[8] MORAIS FILHO, D. C. (2007). Um Convite Matemtica.


EDUFCG.

[9] MORGADO, A.; CARVALHO, J.; CARVALHO, P.; FER-


NANDEZ, P. (1991). Anlise Combinatria e Probabilidade .
Sociedade Brasileira de Matemtica.

[10] RIBENBOIM, P. (2001). Nmeros Primos: Mistrios e Re-


cordes. Sociedade Brasileira de Matemtica.

[11] SANTOS, J. P. O. (1993) Introduo Teoria dos Nmeros.


IMPA.

[12] SANTOS, J. P. O.; MELLO, M. P. e MURARI, I. T. C.


(2006). Introduo Anlise Combinatria. Editora Unicamp.

[13] SOARES, M. G. (2005). Clculo em uma Varivel Complexa.


Sociedade Brasileira de Matemtica.
Mestrado Profissional
em Matemtica em Rede Nacional

Iniciao Matemtica

Autores:

Krerley Oliveira Adn J. Corcho

Unidade IV:

Captulos VII e VIII


7
Desigualdades

Existem duas formas de fazer tima Matemti a. A primeira ser

mais esperto que todo mundo. A segunda ser mais estpido que

todo mundo  mas persistente.

Raoul Bott

Neste captulo estudaremos algumas desigualdades clssicas que

so usadas frequentemente na resoluo de problemas matemticos,

sendo estas aplicadas em contextos que variam desde o nvel mais

simples at o mais complexo.

Uma vez que uma inequao em uma ou mais variveis resolvida,

o resultado d lugar a uma desigualdade que vlida para um certo

conjunto de valores. Alguns exemplos simples de desigualdades so os

seguintes:

(a) x |x|, para qualquer 1 < x < 1;

(b) x2 < x, se x < 1;

(c) (x y)2 0, para quaisquer x e y reais;

x y
(d)
y
+ x
2, para quaisquer x, y > 0.

233
234 7 Desigualdades

7.1 Desigualdade Triangular

A desigualdade triangular arma o seguinte

Teorema 7.1 (Desigualdade Triangular) . Dado um tringulo ABC o


comprimento de um dos lados sempre inferior soma dos compri-
mentos dos outros dois lados, ou seja,

AB < AC + CB, AC < AB + BC e BC < BA + AC.

A B

Figura 7.1: Desigualdade Triangular

Em outras palavras, a desigualdade triangular a formulao ma-

temtica da ideia intuitiva de que o caminho reto mais curto entre

os pontos A e B.

Em analogia com a geometria plana temos uma verso da desigual-

dade triangular para nmeros reais, que provamos a seguir.

Proposio 7.2. Sejam a e b nmeros reais quaisquer, ento


|a + b| |a| + |b|.

Demonstrao. Se a + b 0, ento |a + b| = a + b |a| + |b|. Caso

contrrio, se a + b < 0, ento |a + b| = a b |a| + |b|.


7.1 Desigualdade Triangular 235

Corolrio 7.3. As seguintes desigualdades valem


|a b| |a| + |b| (7.1)

|a b| |a| |b|, (7.2)



|a b| |a| |b| (7.3)

Demonstrao. Para a primeira, escrevemos |a b| = |a + (b)|


|a| + | b| = |a| + |b|. A segunda desigualdade decorre de |a| =

|b + (a b)| |b| + |a b|. A ltima desigualdade consequncia da

segunda, trocando os papis de a e b.

C
O
A
P

Figura 7.2: Problema da central de energia

Exemplo 7.4. Quatro cidades rurais, A, B , C e D, esto situadas


geogracamente formando um quadriltero convexo. Deseja-se cons-
truir uma central de distribuio de energia para as quatro cidades de
modo que a soma total das distncias da central a cada uma das quatro
cidades seja a mnima possvel. Onde dever ser construda a central?

Soluo. Mostraremos que a central de energia dever ser colocada

no ponto O de interseco das diagonais do polgono ABCD. Com


236 7 Desigualdades

efeito, considerando um ponto P, diferente de O, (veja Figura 7.2) a

desigualdade triangular nos garante que

OA + OC = AC < P A + P C

OB + OD = BP < P B + P D,

de onde se segue que

OA + OC + OB + OD < P A + P C + P B + P D,

como espervamos.

Exemplo 7.5. Duas torres de alturas h1 e h2 , respectivamente, esto


separadas a uma distncia d. As torres so amarradas por uma corda
AP B que vai do topo A da primeira torre para um ponto P no cho,
entre as torres, e ento at o topo B da segunda torre, como na Figura
7.3. Qual a posio do ponto P que nos d o comprimento mnimo da
corda a ser utilizada?

A
B

Figura 7.3: Problema das Torres


7.1 Desigualdade Triangular 237

Soluo. Imaginemos que a superfcie do cho um espelho e que re-

etimos o ponto atravs deste, obtendo assim o ponto B0 como mostra

a Figura 7.4.

A
B

C P D

P

Figura 7.4: Soluo geomtrica do problema das torres

Consideremos o segmento AB 0 que intercepta o cho no ponto P


e para nossa surpresa vericaremos que este o ponto que nos d o

comprimento mnimo das cordas. Com efeito, suponhamos que existe

outro P0 situado entre as torres que nos d um comprimento menor

para a corda, ento da Figura 7.4 fcil ver que os tringulos BP D


0 0
e B PD so congruentes, assim como os tringulos BP D e B0P 0D
tambm so congruentes. Logo, as seguintes igualdades seguem dire-

tamente das congruncias:

BP = B 0 P e BP 0 = B 0 P 0 .

Agora, usando a desigualdade triangular no tringulo AB 0 P 0 e as igual-


dades acima, temos que

AP 0 + P 0 B = AP 0 + P 0 B 0 AB 0 = AP + P B 0 = AP + P B,
238 7 Desigualdades

chegando assim concluso de que AP + P B nos oferece o compri-

mento mnimo desejado.

Agora calcularemos a que distncia est P da base D. Lembremos

que AC = h1 , BD = h2 e CD = d e observamos que

h2 h1
tang(]BP D) = = .
PD d PD
dh2
Da tem-se PD = .
h1 + h2

7.2 Desigualdade das Mdias

Denio 7.6. Sejam a1 , a2 , . . . , an1 e an nmeros reais positivos.


As quantidades
n
mh (a1 , a2 , . . . , an ) = , (7.4)
1/a1 + 1/a2 + + 1/an


mg (a1 , a2 , . . . , an ) = n
a1 a2 an , (7.5)

a1 + a2 + + an
ma (a1 , a2 , . . . , an ) = , (7.6)
n

r
a21 + a22 + + a2n
mq (a1 , a2 , . . . , an ) = (7.7)
n
so chamadas, respectivamente, de mdia harmnica, mdia geom-
trica, mdia aritmtica e mdia quadrtica dos nmeros ai , i = 1, 2, . . . , n.
A seguir provaremos alguns resultados que estabelecem relaes de

desigualdades entre as mdias denidas acima.


7.2 Desigualdade das Mdias 239

Proposio 7.7 (Desigualdade das Mdias Aritmtica e Quadrtica) .


Dados a1 , a2 , . . . , an nmeros reais positivos tem-se
r
a1 + a2 + + an a21 + a22 + + a2n
,
n n
ou seja, ma (a1 , a2 , . . . , an ) mq (a1 , a2 , . . . , an ). Alm disso, a igual-
dade vale se, e somente se, a1 = a2 = = an .
Demonstrao. Usando a igualdade

X n
X X
2
(ai aj ) = (n 1) a2i 2 ai aj (7.8)
1i<jn i=1 1i<jn

conclumos que,

X n
X
2 ai aj (n 1) a2i , (7.9)
1i<jn i=1

dado que o termo da esquerda em (7.8) no negativo. Somando em


n
a2i obtemos
P
ambos os membros de (7.9) a quantidade
i=1

n
X 2 n
X
ai n a2i ,
i=1 i=1

donde, dividindo por n2 e tomando a raiz quadrada, segue-se a desi-

gualdade desejada. Por ltimo, observamos que a igualdade em (7.9)

(ai aj )2 = 0,
P
atingida se, e somente se, o que verdade se,
1i<jn
e somente se, a1 = a2 = = an .

Proposio 7.8 (Desigualdade das mdias Geomtrica e Aritmtica) .


Dados a1 , a2 , . . . , an nmeros reais positivos tem-se
a1 + a2 + + an
n
a1 a2 an ,
n
240 7 Desigualdades

ou seja, mg (a1 , a2 , . . . , an ) ma (a1 , a2 , . . . , an ). Alm disso, a igual-


dade vale se, e somente se, a1 = a2 = = an .

Demonstrao. A prova desta desigualdade mais tcnica e exige um

pouco mais de esforo. Dividiremos a mesma em dois passos.

Passo 1. A desigualdade vale para n = 2m .


Procederemos por induo. Para n = 2 a desigualdade vale. De

fato,

( a1 a2 )2 = a1 + a2 2 a1 a2 0.
a +a
Assim, a1 + a2 2 a1 a2 e conseqentemente 1 2 a1 a2 .
2
Agora provamos que se a desigualdade vale para n = k , ento

tambm vale para n = 2k . Com efeito,

a1 + + a2k a1 ++ak
k
+ ak+1 ++a
k
2k

=
2k 2
(1) k a1 ak + k ak+1 a2k

2
(2) q
k
a1 ak k ak+1 a2k

= 2k a1 a2k ,

onde em (1) e (2) usamos a validade da desigualdade em para n=k


e para n = 2, respectivamente. Logo, como j provamos a validade
para n = 2, claro que vale tambm para n = 4, 8, . . . , 2m , . . . , como
espervamos.

Passo 2. Dado m inteiro positivo, ento a desigualdade vale para


m
todo n<2 .

Para vericar isto, denimos o nmero


L= n
a1 an ,
7.2 Desigualdade das Mdias 241

e como a desigualdade vale para n = 2m , temos ento que

a1 + + an + L
| + {z
+ L}
2m n vezes 2m
p
a1 an L2m n
2m
2m
= Ln L2m n = L.
Portanto,
a1 + + an + (2m n)L
L,
2m
logo

a1 + + an 2m L (2m n)L = nL,


obtendo assim que


a1 + + an nL = n n a1 an ,

o que nos d a desigualdade desejada.

Como para qualquer inteiro positivo n sempre existe um inteiro


m
positivo m tal que n<2 , a desigualdade ca provada para todo n.
A prova de que a igualdade s ocorre quando a1 = a2 = = an
pode tambm ser feita por induo e deixamos a cargo do leitor.

Proposio 7.9 (Desigualdade das Mdias Harmnica e Geomtrica) .


Dados a1 , a2 , . . . , an nmeros reais positivos tem-se
n
n a1 a2 an ,
1/a1 + 1/a2 + + 1/an
ou seja, mh (a1 , a2 , . . . , an ) mg (a1 , a2 , . . . , an ). Alm disso, a igual-
dade vale se, e somente se, a1 = a2 = = an .
Demonstrao. Usando a Proposio 7.8 com os nmeros ai substitu-

dos por 1/ai (i = 1, 2 . . . , n) vale que


n n
Y 1 1/n 1X 1
= mg (1/a1 , . . . , 1/an ) ma (1/a1 , . . . , 1/an ) = .
a
i=1 i
n i=1 ai
242 7 Desigualdades

Invertendo esta ltima desigualdade, obtemos ento

mh (a1 , a2 , . . . , an ) mg (a1 , a2 , . . . , an ),

concluindo-se assim a prova. Notemos que as igualdades s ocorrem

se 1/a1 = 1/a2 = = 1/an equivalem as igualdades a1 = a2 = =


an .

O prximo resultado resume as relaes provadas, nas proposies

7.7, 7.8 e 7.9, para as mdias mh , mg , ma e mq .

Teorema 7.10 (Desigualdade das Mdias) . Para toda coleo de n-


meros reais positivos a1 , a2 , . . . , an1 e an se vericam as seguintes
desigualdades:

min(a1 , . . . , an ) mh (a1 , a2 , . . . , an )
mg (a1 , a2 , . . . , an )
(7.10)
ma (a1 , a2 , . . . , an )
mq (a1 , a2 , . . . , an ) max(a1 , . . . , an ).

Alm disso, em cada caso a igualdade ocorre se, e somente se, a1 =


a2 = = an .

Exemplo 7.11. Num tringulo retngulo a altura relativa hipote-


nusa sempre menor ou igual que a metade da hipotenusa. Alm
disso, a igualdade s ocorre quando o tringulo retngulo issceles
(ou seja, seus catetos so iguais).

Soluo. Usando a Figura 7.5, temos que a hipotenusa c dada por

c = x+y e usando o teorema das alturas para um tringulo retngulo


7.2 Desigualdade das Mdias 243


temos que h2 = xy , logo h= xy. A desigualdade entre as mdias

geomtrica e aritmtica nos d que

x+y c
h= xy = ,
2 2
como queramos. Alm disso, a altura a metade da hipotenusa se, e

a b
h
x y
c

Figura 7.5: Interpretao geomtrica da desigualdade das mdias geo-


mtrica e aritmtica

somente se, a igualdade entre as mdias ocorre, ou seja, quando x = y.


Ento, os catetos a e b do tringulo so iguais, sendo este issceles.

Exemplo 7.12 (Desigualdade Isoperimtrica para Tringulos) . O pe-


rmetro de um tringulo de lados a, b e c a soma p = a + b + c.
Entre todos os tringulos com permetro xado p o de maior rea o
tringulo equiltero.
Soluo. Usando a Frmula de Hern temos que a rea de um trin-

gulo com permetro p dada pela expresso


q
p p
A= (
2 2
a)( p2 b)( p2 c),

onde a, b e c so os lados do tringulo.

Usando agora a desigualdade mg ma temos que,


s  3
p p p
p a+ b+ c p2
A 2
2 2
3
2 = .
12 3
244 7 Desigualdades

2
p
Logo a maior rea possvel , a qual atingida quando
12 3
p p p
2
a= 2
b= 2
c a = b = c,
ou seja, quando o tringulo equiltero. Notemos que neste caso,
2

p a2 3
12 3
= 4
.

Exemplo 7.13 (Desigualdade Isoperimtrica para Paraleleppedos) .


Entre todos os paraleleppedos com rea lateral xada A o de maior
volume o cubo (ou seja, o paraleleppedo com todos seus lados iguais).

Figura 7.6: A rea lateral de um paraleleppedo de lados a, b e c dada


por AL = 2(ab + bc + ac).

Soluo. Denotando por a, b e c as medidas das arestas do paralele-

ppedo sabemos que a soma das reas de todas as faces do paralele-

ppedo, ou seja,

AL = 2(ab + ac + bc).
Sendo V o volume do paraleleppedo e usando a desigualdade entre as

mdias aritmtica e geomtrica temos que


 3  3
2 ab + ac + bc AL
V = ab ac bc = . (7.11)
3 6
q
AL 3

Assim, o maior volume possvel V = , obtido quando ab =
6
ac = bc, consequentemente a = b = c.
7.3 Desigualdade de Cauchy-Schwarz 245

7.3 Desigualdade de Cauchy-Schwarz

Teorema 7.14 (Desigualdade de Cauchy-Schwarz) . Dados a1 , . . . ,


an e b1 , . . . , bn nmeros reais tem-se
q q
|a1 b1 + + an bn | x1 + + an b21 + + b2n
2 2 (7.12)

Alm disso, a igualdade s ocorre se existir um nmero real , tal que


a1 = b1 , . . . , an = bn ou b1 = a1 , . . . , bn = an .

Demonstrao. Usando a identidade de Lagrange:

n
X n
X n
X 2 X
a2i b2i = ai b i + (ai bj aj bi )2
i=1 i=1 i=1 1i<jn

temos que
n
X 2 n
X n
X
ai b i a2i b2i ,
i=1 i=1 i=1

de onde se obtm diretamente a desigualdade de Cauchy-Schwarz.

Alm disso, a igualdade ocorre se, e somente se,

X
(ai bj aj bi )2 = 0 ai bj aj bi = 0, 1 i < j n,
1i<jn

o que verdade se, e somente se, existe tal que ai = bi ou bi = ai ,


com i = 1, 2, . . . , n.

Exemplo 7.15. Entre todos os tringulos retngulos de catetos a e b


e hipotenusa c xada, o que tem maior soma dos catetos s = a + b
o tringulo issceles.
Soluo. Usando a desigualdade de Cauchy-Schwarz temos que


a+b=a1+b1 a2 + b2 12 + 12 = c 2
246 7 Desigualdades

e este mximo atingido quando a = 1 e b = 1 ou 1 = a e

1 = b. Em qualquer caso devemos ter a = b.

Exemplo 7.16 (Desigualdade de Minkowski) . Dados ai , bi com 1


i n, nmeros reais, tem-se
v v v
u n u n u n
u X u X uX
2
t (ai + bi )2 t ai + t b2i .
i=1 i=1 i=1

Soluo. Partimos da seguinte igualdade:

n
X n
X n
X n
X
(ai + bi )2 = a2i + b2i + 2 ai b i . (7.13)
i=1 i=1 i=1 i=1

Aplicando a desigualdade de Cauchy-Schwarz no lado direito de (7.13)

temos que

v v
n
X n
X n
X
u n u n
uX uX
(ai + bi )2 a2i + 2
b + 2t
i
2
at b2 i i
i=1 i=1 i=1 i=1 i=1
v v 2 (7.14)
u n u n
uX uX
= t a2 + t b2 .
i i
i=1 i=1

Tomando raiz quadrada em ambos os membros de (7.14) obtemos a

desigualdade de Minkowski.

7.4 Desigualdade de Jensen

A Desigualdade de Jensen est estreitamente relacionada com o con-

ceito de convexidade, o qual explicamos a seguir.


7.4 Desigualdade de Jensen 247

Denio 7.17. Uma funo f : [, ] R dita convexa se para


quaisquer a, b [, ] e para todo [0, 1] satisfaz


f a + (1 )b f (a) + (1 )f (b).

y
y = f (x)

(b, f (b))

(a, f (a))

a b x

Figura 7.7: Grco de uma funo convexa

Geometricamente, a denio de convexidade signica que para

cada par de pontos a e b escolhidos no intervalo [, ] o grco da

funo encontra-se abaixo do segmento de reta secante que junta os

pontos (a, f (a)) e (b, f (b)), como mostra a Figura 7.7.

Exemplo 7.18. A funo f (x) = x2 convexa em qualquer intervalo


[, ].

Soluo. Sejam a, b [, ] e suponhamos, sem perda de generalidade,


248 7 Desigualdades

que a < b. Ento, para todo [0, 1] valem as desigualdades:

(a + (1 )b)2 = 2 a2 + (1 )2 b2 + 2(1 )ab


(1)
2 a2 + (1 )2 b2 + (1 )(a2 + b2 ) (7.15)

= a2 [2 + (1 )] + b2 [(1 )2 + (1 )]
= a2 + (1 )b2 ,

a2 +b2
onde na passagem (1) usamos a desigualdade ab 2
.

Exemplo 7.19. A funo f (x) = 1/x convexa em qualquer intervalo


[, ] com positivo.

Soluo. Sendo a, b [, ] com a < b, para todo [0, 1] tem-se

1 = ( + (1 ))2
= 2 + 2(1 ) + (1 )2
(1) a b 
2
+ + (1 ) + (1 )2 (7.16)
b a
a b
= 2 + (1 ) + (1 ) + (1 )2
b a
 (1 )   (1 ) 
= a + +(1 )b +
a b a b
  (1 ) 
= a + (1 )b +
a b
onde na passagem (1) usamos que a/b + b/a 2 para quaisquer n-

meros positivos a e b. De (7.16) segue-se que

1 1 1
+ (1 ) ,
a + (1 )b a b
1
mostrando isto a convexidade da funo
x
.
7.4 Desigualdade de Jensen 249

Observemos que, usando a desigualdade entre as mdias aritmtica

e quadrtica obtemos

2
a1 + a2 + + an a21 + a22 + + a2n

,
n n

em outras palavras

(ma (a1 , a2 , . . . , an ))2 ma (a21 , a22 , . . . , a2n ). (7.17)

Por outro lado, a desigualdade entre as mdias harmnica e aritmtica

nos garantem que

1
ma (1/a1 , 1/a2 . . . , 1/an ). (7.18)
ma (a1 , a2 , . . . , an )

O seguinte resultado garante que as propriedades (7.17) e (7.18), satis-


1
feitas pelas funes convexas x2 e
x
, so vlidas para qualquer funo

convexa.

Teorema 7.20 (Desigualdade de Jensen) . Seja f : [, ] R uma


n
funo convexa e sejam i [0, 1] (i = 1, . . . , n) tais que i = 1.
X

i=0
Ento, para quaisquer ai [, ] (i = 1, . . . , n) vale

f (1 a1 + + n an ) 1 f (a1 ) + + n f (an ). (7.19)

Observao 7.21. Observemos que, quando 1 = 2 = = n =


1/n, a desigualdade de Jensen nos diz que
 a + a + + a  f (a ) + f (a ) + + f (a )
1 2 n 1 2 n
f ,
n n
ou seja, f (ma (a1 , . . . , an )) ma (f (a1 ), . . . , f (an )).
250 7 Desigualdades

Demonstrao. Faremos a prova por induo. Para n=2 a validade

decorre diretamente da denio. Suponhamos que dado n natural

(7.19) vale, ento temos que provar a validade de

n+1  n+1
(7.20)
X X
f j aj j f (aj ).
j=1 j=i
Notemos que
n+1
X n
X  n
X 
j aj = j aj + 1 j an+1
j=1 j=1 j=1
(7.21)
n
X j
= aj + (1 )an+1 ,
j=1

n n
P P j
onde = j . Assim, usando que
= 1 e a hiptese de induo,
j=1 j=i
obtemos
n+1 n
X  X j

f j aj f aj +(1 )f (an+1 )
j=1 j=1

n
X j
f (aj ) + (1 )f (an+1 ) (7.22)
j=1

n+1
X
= j f (aj ),
j=1

como queramos provar.

7.5 Exerccios

1. Provar que em todo tringulo a soma dos comprimentos das

medianas menor que o permetro do tringulo e maior que o

semipermetro deste.
7.5 Exerccios 251

2. Os centros de trs crculos que no se intersectam esto sobre

uma reta. Prove que se um quarto crculo toca de forma tangente

os trs crculos, ento o raio deste maior que pelo menos um

dos raios dos trs crculos dados.


n
X 1 2n
3. Dado n inteiro positivo, provar que .
j=1
j n+1

4. A soma de trs nmeros positivos 6. Provar que a soma de

seus quadrados no menor que 12.

5. Determinar as dimenses do paraleleppedo de menor diagonal

possvel, sabendo que a soma dos comprimentos de todas suas

arestas 12.

6. Encontrar todas as solues positivas do sistema de equaes

no lineares
x 2 + + x 2 = 1
1 10
1 1
2 + + 2 = 100.
x1 x10

7. Demonstrar que, se a1 , a2 , . . . , an so nmeros positivos tais que

a1 a2 an = 1

ento

(1 + a1 )(1 + a2 ) (1 + an ) 2n .

8. Prove que a mdia geomtrica super-aditiva, isto , para n-

meros no negativos bi , 1 i n, tem-se


ai e

v v v
u n u n u n
uY uY uY
n
t ai + t
n
bi t
n
(ai + bi ).
i=1 i=1 i=1
252 7 Desigualdades

Alm disso, estude em que condies ocorre a igualdade.

Sugesto: Use a desigualdade entre as mdias geomtrica e aritmtica.

9. Usar o mtodo de induo para provar a desigualdade de Cauchy-

Schwarz.
n
(ai + bi )2 0.
P
10. Para todo real Use este fato para dar outra
i=1
prova da desigualdade de Cauchy-Schwarz.

11. Use a desigualdade de Cauchy-Schwarz para dar uma prova alter-

nativa da desigualdade entre as mdias aritmtica e quadrtica

(ma mq ).

12. Prove que ( n )


n n
X 1 X X
ai b i a2i + b2i .
i=1
2 i=1 i=1

13. Prove que a4 + b4 + c4 abc(a + b + c).

14. Prove que se a 0, b 0 e c 0, ento

(a + b)(a + c)(b + c) 8abc.

15. Prove a desigualdade de Bernoulli: (1 + x)n > 1 + nx, para

qualquer x positivo e n inteiro positivo.

16. Prove que se a, b, c e d so inteiros positivos, ento:


 
1 1 1 1
(a + b + c + d) + + + 16.
a b c d

17. Prove que se a 0, b 0 c 0, ento


e

(ab + bc + ca) a bc + b ac + c ab.
7.5 Exerccios 253

18. Prove que se x 0, ento 3x3 6x2 + 4 0.


Sugesto: Use a desigualdade entre as mdias aritmtica e geomtrica.


19. Prove que se x 0, ento 2x + 3/8 4 x.

20. Sejam C1 e C2 dois crculos concntricos de raios r1 e r2 , res-

pectivamente, com r1 < r2 . Sobre o crculo C1 se marcam dois

pontos P1 e P2 diametralmente opostos. Deseja-se encontrar o

ponto P sobre o crculo C2 que maximiza a soma

d(P ) = P P1 + P P2 .
254 7 Desigualdades
8
Polinmios

A oisa mais bela que podemos ontemplar o mistrio. Isto a

fonte da verdadeira arte e in ia.

Albert Einstein

8.1 Operaes com Polinmios

A necessidade de estudar equaes polinomiais aparece em problemas

prticos da humanidade desde pocas muito remotas. Indcios arque-

olgicos indicam que os babilnicos j tinha o domnio de tcnicas de

resoluo de algumas equaes do primeiro grau e do segundo grau,

apresentadas em forma de problemas cotidianos. Contudo, o grande

avano terico no estudo das equaes polinomiais s se iniciou com o

Renascimento na Europa. No incio do sculo XVI, Viti introduziu o

uso de letras para representar quantidades desconhecidas.

Na mesma poca, um outro grande desao estava perturbando as

mentes matemticas de toda a Europa, em especial as da Itlia. A

soluo explcita utilizando as operaes elementares (soma, subtra-

o, multiplicao, diviso, radiciao e potenciao) da equao do

255
256 8 Polinmios

terceiro grau no era conhecida e muitos dos melhores matemticos

da poca trabalharam neste problema, destacando-se entre eles Ni-

colo Fontana, o Tartaglia (gago, em italiano). A histria da soluo

desta equao est repleta de intrigas, disputas e acusaes, envol-

vendo Tartaglia e Cardano. Hoje os historiadores atribuem a Tarta-

glia a primazia na descoberta da soluo da equao do terceiro grau

como conhecemos. desta poca tambm a soluo da equao do

quarto grau, atribuda a Ludovico Ferrari.

Entretanto, apesar dos muitos esforos empreendidos na direo de

encontrar a soluo geral da equao do quinto grau, mais de 200 anos

se passaram sem nenhum sucesso. At que em 1824, o matemtico

noruegus Niels Abel mostrou que impossvel resolver as equaes de

grau cinco em sua forma geral. Ou seja, nem todas as equaes de grau

cinco podem ser resolvidas com as operaes elementares. Mais ainda,

em 1830 o matemtico francs Evariste Galois descobriu um mtodo

que determina quando uma equao de grau qualquer resolvel com

as operaes elementares, encerrando um belssimo captulo do estudo

das equaes polinomiais e da Matemtica.

Neste captulo iremos estudar um pouco mais formalmente os poli-

nmios e suas propriedades.

Denio 8.1. Um polinmio na varivel x uma expresso do tipo


p(x) = an xn + an1 xn1 + + a1 x + a0

onde a0 , a1 , . . . , an so nmeros. Se an 6= 0, dizemos que n o grau


do polinmio e a0 , a1 , . . . , an so seus coecientes. O coeciente an
chamado de coeciente lder do polinmio.
Observao 8.2. No se dene o grau do polinmio nulo, que tem
todos os coecientes iguais a zero.
8.1 Operaes com Polinmios 257

Por exemplo,

p(x) = 3x 1 um polinmio de grau 1;

q(x) = 4x3 + 7x + 1 um polinmio de grau 3;

4
t(x) = x um monmio de grau 4;
2

v(x) = x4 + 5x2 + 1 um polinmio de grau 4;
2
u(x) = 7 um polinmio de grau 0.

Uma equao polinomial de graun, ou simplesmente uma equao


de grau n, uma sentena p(x) = 0, onde p(x) um polinmio de

grau n com coecientes reais. Por exemplo, 2x 1 = 0 uma equao


5 3
do primeiro grau, enquanto x + 4x + 5x 1 = 0 uma equao de

grau 5. Note que nem todos os coecientes precisam ser diferentes de

zero.

Para obtermos o valor do polinmio p(x) = an xn + an1 xn1 +


+ a1 x + a0 no nmero real r, devemos substituir x por r para obter
o nmero real

p(r) = an rn + an1 rn1 + + a1 r + a0 .

Por exemplo, o valor do polinmio p(x) = 4x3 7x + 1 em 2 p(2) =


4 23 7 2 + 1 = 19.
Dizemos que um nmero real r uma raiz para a equao

an xn + an1 xn1 + + a1 x + a0 = 0

se o valor de p(x) = an xn + an1 xn1 + + a1 x + a0 em r zero, ou

seja, se r verica

an rn + an1 rn1 + + a1 r + a0 = 0.
258 8 Polinmios

Por exemplo, 5 raiz da equao:

2x 10 = 0.

Uma das vantagens dos polinmios sobre outros objetos matem-

ticos que podemos denir as operaes de soma de polinmios e

multiplicao de polinmios. Com estas operaes, o conjunto dos po-

linmios possui muitas propriedades similares dos nmeros inteiros,

tornando prtico o seu uso.

Vamos denir agora o que signica a soma de dois polinmios.

Para isso, vamos comear somando dois monmios e depois estender

nossa denio para polinmios em geral.

Para somar dois monmios de mesmo grau p(x) = ak xk e q(x) =


bk xk somamos seus coecientes, obtendo o polinmio t(x) = p(x) +
q(x) = (ak +bk )xk . Em geral, para somar o polinmio p(x) = a0 +a1 x+
a2 x2 + + an xn com o polinmio q(x) = b0 + b1 x + + bm xm , onde
n m devemos somar todos os monmios de mesmo grau, obtendo o
polinmio:

t(x) = p(x) + q(x) = c0 + c1 x + + cm xm

onde, c i = ai + b i para 0in e ci = b i para i > n.


Por exemplo, sendo

p(x) = 3x 1,

q(x) = 4x3 + 7x + 1,

t(x) = 2 x4 ,

v(x) = 2 x4 + 5x2 + 1
8.1 Operaes com Polinmios 259

temos que

p(x) + q(x) = 4x3 + (3 + 7)x 1 + 1 = 4x3 + 10x,

v(x) + t(x) = ( 2 2 )x4 + 5x2 + 1 = 5x2 + 1.

A seguir, enumeramos algumas propriedades simples e importantes

da soma de polinmios que decorrem da denio dada e das propri-

edades anlogas vlidas para os nmeros reais.

1. Associatividade . Dados polinmios p(x), q(x) e t(x), vale

(p(x) + q(x)) + t(x) = p(x) + (q(x) + t(x))

2. Elemento neutro . Se 0 denota o polinmio nulo e p(x) um

polinmio qualquer, ento

0 + p(x) = p(x).

3. Elemento simtrico . Se p(x) = a0 + a1 x + + an xn um


n
polinmio, ento o polinmio q(x) = a0 a1 x an x

satisfaz:

p(x) + q(x) = 0.

4. Comutatividade . Se p(x) e q(x) so polinmios, ento

p(x) + q(x) = q(x) + p(x).

Note que os nmeros inteiros possuem propriedades similares para

a operao de soma de nmeros inteiros. Vamos agora denir o produto


de dois polinmios. Para isso, vamos primeiramente denir o produto

de dois monmios, como j zemos no caso de soma de polinmios.


260 8 Polinmios

Se n, m so nmeros naturais, denimos o produto dos monmios

p(x) = an xn e q(x) = bm xm como:

p(x)q(x) = an bm xn+m .

Tendo isto em mente, para efetuarmos o produto do polinmio de

grau n, p(x) = a0 + a1 x + a2 x2 + + an xn pelo polinmio q(x) =


b0 + b1 x + + bm xm de grau m, com n m, devemos:

Completamos a escrita de p(x) e de q(x) at o termo n + m


colocando ak = 0 para k > n e bk = 0 para k > m;

Denimos

t(x) = p(x)q(x) = c0 + c1 x + + cn+m xn+m

onde, ci = a0 bi + a1 bi1 + + ai1 b1 + ai b0 para 0 i n + m.

Apesar de parecer complicada, a denio no to difcil de ser

aplicada. Para tentar visualizar o processo de multiplicao de dois

polinmios vamos pensar que os monmios so seres aliengenas vin-

dos do distante planeta de Algebrum e possuam mos. Quando dois

monmios se encontram, invariavelmente eles apertam as mos e desse

aperto aparece o produto desses monmios.

Assim, para multiplicar os polinmios p(x) e q(x), que so forma-

dos por dois grupos de monmios, devemos escolher o primeiro mon-

mio de p(x) e faz-lo apertar a mo de cada um dos monmios de

q(x), somando os monmios obtidos. Aps isso, tomamos o segundo

monmio de p(x) e fazemos ele apertar a mo de cada um dos mon-

mios deq(x), somando os monmios obtidos aos monmios anteriores.


Repetimos o processo at o ltimo monmio de p(x).
8.1 Operaes com Polinmios 261

Deste modo, se p(x) = x2 + 2x 3 e q(x) = x2 + 5x + 1, para

obter p(x)q(x) fazemos:

p(x)q(x) = x4 + 5x3 + x2 2x3 + 10x2 + 2x + 3x2 15x 3


= x4 + 3x3 + 14x2 13x 3.

Observe que com a denio de multiplicao de polinmios dada

acima, o coeciente c0 igual a a0 b 0 . Do mesmo modo, o coeciente


n+m
do termo x cn+m = an bm . Como p(x) tem grau n (isto , an 6= 0)

e q(x) tem grau m (bm 6= 0), o coeciente cn+m = an bm 6= 0. Logo,


o polinmio p(x)q(x) tem grau n + m. Com isso, demonstramos o

seguinte fato:

Proposio 8.3. Se o polinmio p(x) tem grau n e o polinmio q(x)


tem grau m, ento o polinmio p(x)q(x) tem grau n + m.

Um caso particular interessante quando multiplicamos um nme-

ro c, que podemos considerar como sendo um polinmio de grau zero


q(x) = c, por um polinmio p(x) = a0 + a1 x + + an xn . Neste caso,
ns obtemos o polinmio

cp(x) = ca0 + ca1 x + + can xn .

Do mesmo modo em que podemos vericar as propriedades da

soma de polinmios a partir das propriedades similares dos nmeros

reais, podemos tambm vericar as propriedades abaixo sobre a mul-

tiplicao de polinmios. Deixamos essa vericao como exerccio.

1. Associatividade . Dados polinmios p(x), q(x) e t(x), vale

(p(x)q(x))t(x) = p(x)(q(x)t(x))
262 8 Polinmios

2. Elemento neutro . Se 1 denota o polinmio constante e p(x)

um polinmio qualquer, ento

1p(x) = p(x).

3. Comutatividade . Se p(x) e q(x) so polinmios, ento

p(x)q(x) = q(x)p(x).

4. Distributividade . Se p(x), q(x) e t(x) so polinmios, ento

(p(x) + q(x))t(x) = q(x)t(x) + p(x)t(x).

Note que, assim como nos inteiros, a propriedade de existncia de

elementos inversos para a multiplicao de polinmios no vale. De

fato, podemos vericar que se p(x) um polinmio de grau n maior ou


igual a um, ento no existe um polinmio q(x) tal que p(x)q(x) = 1.

De fato, suponha por absurdo, que exista q(x) um polinmio com grau

m 0 tal que
p(x)q(x) = 1.

Ento, utilizando a Proposio 8.3 temos que o grau de p(x)q(x)

n+m que maior ou igual que um. Como o grau do polinmio

constante 1 zero, temos que a igualdade acima no pode valer, onde

chegamos a um absurdo.

Em resumo, os nicos polinmios que podem ter inversos com res-

peito operao de multiplicao so os polinmios constantes no

nulos. Esta mais uma das semelhanas entre os inteiros e os polin-

mios.
8.2 Algoritmo de Euclides 263

8.2 Algoritmo de Euclides

Diremos que um polinmio a(x) divide o polinmio b(x) se existir q(x)


tal que b(x) = q(x)a(x).
Por exemplo, o polinmio a(x) = x2 + x + 1 divide o polinmio

x3 1 pois

(x 1)(x2 + x + 1) = x3 1.

Devido Proposio 8.3, se o polinmio a(x) divide o polinmio

no nulo b(x), ento o grau de a(x) menor ou igual ao grau de b(x).


Agora, vamos enunciar um fato que vale para os inteiros e que vale

tambm para os polinmios e que ser de grande utilidade. Pedimos

que o leitor releia o algoritmo de Euclides , estudado no Captulo 3.

No conjunto dos polinmios, ainda vale

Teorema 8.4 (Algoritmo de Euclides) . Sejam a(x) e b(x) dois polin-


mios com coecientes reais, b(x) 6= 0. Ento, existem polinmios com
coecientes reais q(x) e r(x), com r(x) = 0 ou grau de r(x) menor
que o grau de b(x) tais que:

a(x) = b(x)q(x) + r(x).

Alm disso, q(x) e r(x) esto determinados de modo nico.

Demonstrao. Vamos mostrar primeiro a unicidade. De fato, assuma

que

a(x) = b(x)q1 (x) + r1 (x) = b(x)q2 (x) + r2 (x),

com r1 e r2 de graus menores que o grau de b. Assim,

b(q1 q2 ) = r2 r1 .
264 8 Polinmios

Consequentemente, q1 = q2 , j que caso contrrio, o polinmio b(q1


q2 ) teria grau pelo menos igual ao grau de b e o polinmio r2 r1 tem
grau menor que o grau de b.
Vamos agora mostrar a existncia. Os passos da prova so idnticos

a prova do algoritmo de Euclides para nmeros inteiros, demonstrado

no Captulo 3. De fato, a ideia reduzir o grau do dividendo at

que ele se torne menor que o do divisor e a diviso se torne imediata.

Note que se a tem grau menor que b, ento tomamos o resto com
sendo r = a e o quociente como sendo q = 0. Suponhamos que

a(x) = an xn + +a1 x+a0 tenha grau n e b(x) = bm xm + +b1 x+b0


tenha grau m e que n > m. Dena

an nm
c1 (x) = a(x) x b(x).
bm
Observe que o grau de c1 no mximo n 1. Se c1 puder se dividido
por b, digamos com c1 (x) = b(x)q(x) + r(x), com grau de r(x) menor
que o grau de b(x), ento

an nm an
a(x) = b(x) x + c1 (x) = b(x)( xnm + q(x)) + r(x).
bm bm
Logo, reduzimos o problema de dividir o polinmio a(x) por b(x) pelo
problema de dividir o polinmio c1 (x) por b(x), com c1 (x) de grau

menor que a(x). Repetimos o processo, utilizando c1 no lugar de a(x),

obtendo o polinmio c2 (x) de grau menor que o de c1 (x). Como a cada

passo reduzimos o grau do dividendo em pelo menos uma unidade, ao

m de no mximo n m passos, obteremos um polinmio com grau


menor que o grau de b(x), que claramente divisvel por b(x). Proce-

dendo como antes, achamos q(x) e r(x) tais que a(x) = b(x)q(x)+r(x)

e r(x) com grau menor que o grau de b(x).


8.2 Algoritmo de Euclides 265

Por exemplo, se a(x) = 10x3 3x + 2 e b(x) = x2 + 1, tomando

q(x) = 10x e r(x) = 13x + 2 temos que

10x3 3x + 2 = (x2 + 1)10x + (13x + 2).

Note que o grau de r(x) = 13x + 2 menor que o grau de b(x) =


x2 + 1 .
Se na expresso do polinmio p(x) decidimos substituir a varivel

x por um nmero real s, estaremos avaliando o polinmio p(x) em s


e denotamos este nmero por p(s).
2
Por exemplo, se p(x) = x + 3x + 1, ento substituindo x por 2,
temos que

p(2) = 22 + 3 2 + 1 = 11

e fazendo x = 3

p(3) = (3)2 + 3 (3) + 1 = 1.

Quandop(s) = 0 dizemos que s anula o polinmio no nulo p(x),


ou ainda, que s uma raiz do polinmio p(x).
3
Por exemplo, para p(x) = x 8, temos que 2 uma raiz de p(x)
3
j que p(2) = 2 8 = 0.

Um fato muito importante que consequncia do algoritmo de

Euclides o seguinte teorema:

Teorema 8.5. Se s uma raiz do polinmio p(x), ento o polinmio


x s divide p(x). Reciprocamente, se x s divide p(x), ento s raiz
de p(x).

Demonstrao. Primeiramente, assuma que x s divida p(x). Neste


caso, existe um polinmio q(x) tal que p(x) = q(x)(x s). Avaliando
266 8 Polinmios

o polinmio p(x) em s, temos que:

p(s) = q(s)(s s) = q(s) 0 = 0.

Logo s uma raiz de p(x).


Para provar que se s uma p(x) ento x s divide p(x),
raiz de

vamos utilizar o algoritmo da diviso, com a(x) = p(x) e b(x) = x s.

Neste caso, temos que existem q(x) e r(x) de modo que r(x) = 0 ou o

grau de r(x) menor que o grau de x s e alm disso vale

p(x) = q(x)(x s) + r(x).

Observe que, com as condies do resto r(x), podemos escrever que


r(x) = c R. Ento, p(x) = q(x)(xs)+c e 0 = p(s) = q(s)0+c = c.
Portanto, r(x) = 0 e p(x) = q(x)(x s), isto , x s divide p(x).

A proposio anterior nos permite determinar o nmero mximo

de razes reais de um polinmio no nulo. De fato, vamos mostrar.

Proposio 8.6. O nmero mximo de razes reais do polinmio no


nulo p(x) = an xn + an1 xn1 + + a1 x + a0 n.

Demonstrao. Digamos que s0 < s1 < s2 < < sk sejam razes

distintas do polinmio p(x). Observe que podemos utilizar a Propo-

sio 8.5 para garantir que existe um polinmio no nulo q1 (x) tal

que

p(x) = q1 (x)(x s0 ).
Assim, pela Proposio 8.3, o grau de q1 (x) deve ser igual a n 1.
Note que p(si ) = q1 (si )(si s0 ). Como para todo i = 1, 2, . . . , k temos
que si > s0 com p(si ) = 0, temos que, necessariamente, q1 (si ) = 0.

Assim, em particular, temos que q1 (s1 ) = 0. Logo, podemos aplicar


8.2 Algoritmo de Euclides 267

a proposio novamente para obter que existe um polinmio no-nulo

q2 (x) tal que

q1 (x) = q2 (x)(x s1 ).

Assim, como o grau de q1 (x) n 1, pela Proposio 8.3, o grau de


q2 (x) deve ser igual a n 2.
Novamente, temos que q1 (si ) = q2 (si )(si s1 ), si > s1 e p(si ) = 0

para todo i = 2, . . . , k . Disto segue que, necessariamente, q2 (si ) = 0,

se i = 2, 3, . . . , k. Assim, temos que q2 (s2 ) = 0.

Logo, podemos repetir esse argumento para obter um polinmio

q3 (x) de grau n 3, de modo que s3 , s4 , . . . , sk q3 (x). Re-


so razes de

petindo o argumento, encontramos uma sequncia q1 (x), q2 (x), q3 (x), . . .

com graus no mximo n 1, n 2, n 3, . . . o que nos leva a con-

cluir que no podemos repetir esse argumento mais que n vezes, j

que os graus dos polinmios q1 (x), q2 (x), q3 (x), . . . esto diminuindo.

Ou seja, no podemos ter mais que n razes para o polinmio p(x), o

que conclui a prova.

Alertamos que, apesar da Proposio 8.6 nos garantir que existem

no mximo n razes reais de um polinmio de grau n no nulo, existem


polinmios que no possuem razes reais. Por exemplo, p(x) = x + 1
2

2
no possui razes rais, j que x 0 para todo nmero real x.

Uma consequncia da Proposio 8.6 a seguinte:

Proposio 8.7. Se dois polinmios p(x) e q(x) de grau n avaliados


em n + 1 nmeros r1 , r2 , . . . , rn+1 coincidem, isto , p(ri ) = q(ri ) para
i = 1, 2, 3, . . . , n + 1, ento p(x) e q(x) so iguais.

Demonstrao. Considere o polinmio t(x) = p(x) q(x). Observe

que se t(x) no-nulo, o grau de t(x) no mximo n, j que p(x)


268 8 Polinmios

e q(x) tm graus iguais a n. Observe ainda que t(ri ) = 0, j que

p(ri ) = q(ri ) e
t(ri ) = p(ri ) q(ri ) = 0.

Logo, t(x) tem grau no mximo n e mais de n razes, contradizendo a

Proposio 8.6.

No Exerccio 23 faremos uma aplicao interessante dessa propo-

sio, propondo que voc prove que dados nmeros reais a1 , a2 , . . . ,


an+1 e r1 , r2 , . . . , rn+1 , ento existe um nico polinmio de grau n tal

que p(ri ) = ai .

8.3 Sempre Existem Razes de um Polinmio?

Pode parecer frustrante o fato de que um polinmio com coecientes

reais pode no possuir razes reais. Por exemplo, quando tentamos

aplicar a frmula de Bhaskara equao x2 + 1 = 0, encontramos

= 4 e, consequentemente, se fosse possvel escrever as solues,

elas se escreveriam como


4
x1 =
2
e
4
x2 =
2
claro que as expresses acima no tm sentido no conjunto dos

nmeros reais, pois no existe nmero cujo quadrado seja 4, ou seja,


no possvel extrair a raiz quadrada de 4. Isso tirou o sono de

vrias geraes de matemticos. Desde Hern de Alexandria h dois

mil anos atrs, os matemticos encontram expresses como a do tipo

acima, envolvendo razes de nmeros negativos.


8.3 Sempre Existem Razes de um Polinmio? 269

A primeira reao da comunidade matemtica foi rejeitar esses

nmeros complexos e simplesmente desconsiderar razes de nmeros

negativos. Porm, j no sculo XVI, Cardano se deu conta de que os

nmeros complexos surgem naturalmente quando desejamos resolver

uma equao do terceiro ou quarto grau, mas relutava quanto ao seu

uso, dizendo que esses nmeros eram to sutis, quanto inteis.

No sculo seguinte, motivado pela sugesto de Albert Girard que

uma equao de grau n possui n razes, Rene Descartes observou que


os nmeros reais eram insucientes para representar todas essas razes

e utilizou o termo imaginrias para as razes que no so reais.



A notao tradicional i = 1 s veio a ser introduzida um sculo
mais tarde, com Leonard Euler, que tambm o pai do termo nmero
complexo. Euler e o matemtico francs Jean D'Alambert zeram apli-

caes dos nmeros complexos a problemas prticos, como projeo

de mapas e hidrodinmica. Euler e Lagrange, grandes matemticos

da histria da humanidade, tentaram mostrar a armao de Girard,

de que uma equao de grau n possui n razes, mas sem sucesso. A

primeira prova correta de tal teorema s apareceu no nal do sculo

XVIII com os trabalhos de Gauss.

8.3.1 Nmeros Complexos e Razes de Polinmios


O conjunto dos nmeros complexos, denotado pela letra C, o con-

junto das expresses

C = {x + iy; x, y R},

onde i satisfaz i2 = 1. Costuma-se denotar i por 1. Destacamos

que i meramente um smbolo que nos ajudar a denir as operaes

de soma e de multiplicao de nmeros complexos. Essas operaes


270 8 Polinmios

tero as mesmas propriedades que as operaes de nmeros reais, como

associatividade, comutatividade, elemento neutro, etc. Por exemplo,

so nmeros complexos 2 3i, 3 + i e 3i.


Vamos denir a soma e multiplicao de nmeros complexos. Da-

dos dois nmeros complexos a + bi e c + di denimos a soma como:

(a + bi) + (c + di) = (a + b) + (c + d)i

e denimos a multiplicao como

(a + bi)(c + di) = (ac bd) + (bc + ad)i

Por exemplo, se tomamos os nmeros 2 3i e 3 + 4i ento

(2 3i) + (3 + 4i) = 5 + i

(2 3i)(3 + 4i) = (2 3 (3 4)) + (3 3 + 2 4)i = 18 i.

Aqui ns estamos considerando 0 + 3i = 3i e 3 + 0 i = 3. Isso

nos permite colocar os nmeros reais dentro do conjunto dos nmeros

complexos, considerando cada nmero real r como sendo um nmero

complexo da forma r + 0 i.
Fica para o leitor a vericao de que valem as propriedades de

associatividade, comutatividade, etc. O elemento neutro da soma o

elemento 0+0i que simplesmente denotaremos por 0. Do mesmo

modo, o elemento neutro da multiplicao 1+0i, que ser denotado


por 1. O leitor curioso pode achar mais informaes sobre nmeros

complexos e solues de equaes algbricas em [5] ou [13].


8.3 Sempre Existem Razes de um Polinmio? 271

Assim, dado um nmero complexo z faz sentido avaliar o polinmio


(de coecientes complexos ou reais) p(x) = an xn + an1 xn1 + +
a1 x + a0 em z, obtendo o nmero complexo

p(z) = an z n + an1 z n1 + + a1 z + a0 .

Por exemplo, se p(x) = x2 + 4, ento 2i e 2i so razes deste

polinmio, j que:

p(2i) = (2i)2 + 4 = 4 + 4 = 0.

p(2i) = (2i)2 + 4 = 4i2 + 4 = 4 + 4 = 0.

Note que p(x) no possui nenhuma raiz real, mas possui duas razes
complexas. Como j mencionamos, a grande vantagem em utilizar os

nmeros complexos em vez dos nmeros reais que, dado um polin-

mio qualquer com coecientes complexos, ele sempre tem uma raiz

complexa. Isso foi o assunto da tese de doutorado do Prncipe da


Matemtica, Johann Carl Friedrich Gauss (1777-1855).

Teorema 8.8 (Teorema Fundamental da lgebra) . Todo polinmio


no constante com coecientes complexos de grau n possui exatamente
n razes complexas, contadas com multiplicidade.

Uma demonstrao do Teorema Fundamental da lgebra foge do

objetivo deste livro. Podem ser dadas vrias demonstraes diferen-

tes desse teorema, utilizando diversas teorias matemticas avanadas.

Uma demonstrao desse teorema pode ser achada em [13].


272 8 Polinmios

8.4 Exerccios

1. Calcule o quociente e o resto da diviso de p(x) por q(x) para

os polinmios p(x) e q(x) dados:

(a) p(x) = 3x3 2x + 1 e q(x) = 7x 1;


(b) p(x) = x5 1 e q(x) = x 1;
(c) p(x) = 3x5 2x3 + 1 e q(x) = x2 + x + 1

2. Encontre os valores de A e B de forma que

x+1 A B
= + .
x x
2 x x1

3. Se os polinmios x2 x+4 e (xa)2 +(x+b) so iguais, encontre


a + b.

4. Quais os valores de a e b que tornam iguais os polinmios


2
P1 (x) = x x 6 e P2 (x) = (x + a)2 b?

5. A diviso de P (x) por x4 + 1 tem quociente x+2 e resto 1.

Encontre o polinmio P (x).

6. Qual o resto da diviso do polinmio x100 por x + 1?

7. Determine o resto da diviso do polinmio p(x) pelo polinmio

g(x) = x, onde p(x) = (x 1)(x 2) . . . (x n) + b .

8. Mostre que xn 1 divisvel por x1 para todo n 1.

9. Faa os seguintes itens:

(a) encontre o quociente da diviso de xn+1 1 por x 1;


8.4 Exerccios 273

(b) utilize a diviso anterior para calcular a soma 1 + x + x2 +


x3 + + xn dos n primeiros termos de uma progresso

geomtrica de razo x.

10. Determine o valor de a para que o polinmio P (x) seja divisvel

por x a, onde P (x) = x3 + (1 a)x2 + (1 + a)x 1.

11. Mostre que o polinmio P (x) = x100 2x50 + 1 divisvel por


2
x 1.

12. Mostre que o resto r(x) da diviso do polinmio p(x) por xs


r(x) = p(s).
Dado o polinmio p(x) = an xn + an1 xn1 + + a1 x + a0
denimos a derivada de p(x) como sendo o polinmio:

p0 (x) = nan xn1 + (n 1)an1 xn2 + + 2a2 x + a1 .

Por exemplo, a derivada do polinmio x5


5x4 e a
o polinmio
3 2 2
derivada do polinmio x +5x +2x1 o polinmio 3x +10x+2.

13. Usando as informaes do Exerccio 12, calcule:

(a) a derivada dos polinmios:

(i) x + 1;
(ii) x4 + 3 ;
(ii) 1 + x + x2 + x3 + + xn .
(b) Sabendo que p(0) = 1, calcule tambm o polinmio p(x)
cuja derivada

(i) x4 .
(ii) x2 + 1.
274 8 Polinmios

(ii) x3 + 2x2 + 3.
(c) Prove que se p(x) e q(x) so polinmios, ento

(i) (p + q)0 (x) = p0 (x) + q 0 (x)


(ii) (pq)0 (x) = p0 (x)q(x) + p(x)q 0 (x)
Sugesto: Faa primeiro para monmios.

Denimos uma raiz mltipla de um polinmio p(x) como sendo


2
uma raiz a tal que (x a) divide p(x). Caso a seja uma raiz
que no raiz mltipla, dizemos que ela raiz simples.

14. Mostre que a raiz mltipla de um polinmio p(x) se, e somente


0
se, a raiz de p(x) e de p (x).

Sugesto: Use o exerccio anterior.

15. Para quais valores de nN tem-se que

(a) 1 + x2 + x4 + . . . + x2n2 divisvel por 1 + x + . . . + xn1 ?


(b) 1 + x3 + x6 + . . . + x3n3 divisvel por 1 + x + . . . + xn1 ?
(c) Generalize.

16. (a) Resolva a equao 20x3 30x2 + 12x 1 = 0, sabendo-se


1
que uma de suas razes.
2
(b) Uma raiz da equao x3 (2a+1)x2 +a(a+2)xa(a+1) = 0
a + 1, ache as outras duas.

17. Ache os possveis valores de aZ para que o polinmio

a2 x4 + 4x3 + 4ax + 7

seja divisvel por x + 1.


8.4 Exerccios 275

Um polinmio com coecientes reais no constante p(x) dito ir-


redutvel se p(x) = a(x)b(x), ento a(x) ou b(x) so polinmios
constantes. Quando p(x) no for irredutvel, diremos simples-

mente que ele redutvel. Os polinmios irredutveis desempe-

nham papel anlogo no conjunto dos polinmios ao dos nmeros

primos em Z.

18. Prove que todo polinmio de grau 1 irredutvel.

19. Prove que se f (x) um polinmio de grau 2 e possui uma raiz


real, ento f (x) redutvel.

20. Mostre que todo polinmio f (x) de grau mpar 3 redutvel.

Um polinmio com coecientes inteiros no constante p(x) dito


irredutvel sobre Q se p(x) = a(x)b(x) com a(x) e b(x) polinmios
com coecientes racionais, ento a(x) ou b(x) so polinmios

constantes.

Um teorema importante que descreve uma condio para um

polinmio ser irredutvel sobre Q o conhecido critrio de Ei-

senstein, que diz:

Teorema 8.9 (Critrio de Eisenstein) . Seja f (x) = a0 + a1 x +


+ an xn um polinmio com coecientes inteiros. Suponha que
exista um primo p tal que:

(a) p - an ;
(b) p | a0 , p | a1 , . . . , p | an1 ;
(c) p2 - a0 .

Ento, f (x) irredutvel sobre Q.


276 8 Polinmios

Para uma prova desse resultado veja o livro [2]. Faa os seguintes

problemas:

21. Mostre que os seguintes polinmios f (x) so irredutveis sobre

Q.
Sugesto: Use o critrio de Eisenstein .

(a) f (x) = x4 + 2x3 + 2x2 + 2x + 2;


(b) f (x) = x6 + 15;
(c) f (x) = x4 + 10x3 + 20x2 + 30x + 22.

22. Determine quais dos polinmios abaixo so irredutveis sobre Q.


Sugesto: Use o critrio de Eisenstein .

(a) x3 x + 1
(b) x3 + 2x + 10
(c) x4 x + 1

O problema a seguir trata do polinmio de interpolao de La-

grange.

23. Demonstre a proposio a seguir:

Polinmio de Interpolao de Lagrange. Sejam ai , bi em


0 0
R, i = 1, 2, . . . , n, com os ai s dois a dois distintos e os bi s nem

todos nulos. Considere os polinmios

(x ai ) (x ai1 )(x ai+1 ) (x an )


pi (x) = bi
(ai a1 ) (ai ai1 )(ai ai+1 ) (ai an )
8.4 Exerccios 277

para i = 1, 2, . . . , n. Ento, o polinmio

n
X
p(x) = pi (x)
i=1

o nico polinmio de grau menor que n, tal que p(ai ) = bi ,


para todos i = 1, 2, . . . , n.

24. Determine o polinmio p(x) de grau 7 tal que

p(1) = p(2) = = p(7) = 8 e p(0) = 1.


278 8 Polinmios
A
Apndice: Funes

Estamos acostumados a expresses cotidianas que retratam uma

relao entre grandezas, como por exemplo, o quanto Joo ganha


funo do que ele trabalha , ou ainda a distncia que percorremos
uma funo da velocidade e do tempo que viajamos . Essas e outras
expresses ilustram a noo de funo como uma relao entre grande-

zas de dois conjuntos dados. Matematicamente, a noo de funo foi

melhor entendida muito recentemente, com os avanos tericos ocorri-

dos no nal do sculo XIX e incio do sculo XX. Entretanto, o seu uso

como instrumento e os estudos para tornar sua denio um objeto

claro so bem antigos e datam pelo menos desde o incio do clculo

diferencial, onde a noo de funo era por vezes entendida como sua

expresso analtica. O entendimento dessa noo foi crucial para o

avano da Matemtica e importante que o estudante de Matemtica

tenha claro seu signicado.

Para iniciar a discusso um pouco mais formalmente da noo

de funo, vamos denir intuitivamente uma funo como um objeto

matemtico composto de trs ingredientes: um conjunto no vazio A,

279
280 A Apndice: Funes

chamado de domnio da funo, um conjunto no vazio B , chamado


de contradomnio da funo e uma correspondncia, que associa a
cada elemento do primeiro conjunto um nico elemento do segundo

conjunto. O trio domnio, contradomnio e correspondncia damos o

nome de funo. Para simplicar o seu uso, foi criada uma notao

que empacota todos os trs ingredientes. Denotamos uma funo por

f : AB
x f (x)
para indicar que A o domnio, B o contradomnio e que se x
um elemento de A ento a ele associaremos o elemento f (x) de B .
importante no confundir uma funo com sua expresso analtica,

quando esta dada. Para caracterizar uma funo, precisamos dar

seus trs ingredientes: domnio, contradomnio e correspondncia, e

no somente a correspondncia y = f (x).

Exemplo A.1. Seja a funo f denida de modo que o seu domnio


o conjunto dos nmeros naturais e o contradomnio o conjunto dos
nmeros naturais, e a correspondncia tal que a cada nmero natural
n associamos o seu quadrado n2 . Observe que podemos denotar isso
compactamente por:
f : NN
n n2
Veja tambm que se dermos simplesmente a expresso analtica
x x2 ou y = x2 para nossa funo, ela no estaria caracterizada,
pois no saberamos qual o domnio e o seu contradomnio.
Em alguns casos onde o domnio e o contradomnio esto xados e

claros para o interlocutor, podemos nos referir a uma funo simples-

mente invocando sua correspondncia y = f (x).


281

Exemplo A.2. Considere o domnio como sendo o conjunto P for-


mado pelas pessoas do Brasil e o segundo conjunto como sendo o con-
junto L das letras do alfabeto. A correspondncia ser a seguinte: a
cada pessoa do Brasil, associaremos a primeira letra do seu nome.
Assim, uma pessoa chamada Mrio, ser associada letra M. Em
notao de funo:
f : P L
x f (x)
onde f (x) a primeira letra do nome de x.

Exemplo A.3. Considere o domnio S como sendo o conjunto dos


pontos de uma sala de aula e o contradomnio como sendo os nmeros
reais. A cada ponto x da sala de aula associamos sua temperatura t(x)
em um dado momento, medida por um termmetro instalado na sala.
Observe que t : S R assim denida uma funo, pois cada ponto
possui uma nica temperatura bem denida no instante xado, que
um nmero real. Por outro lado, se trocarmos os papis do domnio
e contradomnio e a cada nmero real associamos o ponto da sala que
tem aquela temperatura, no teremos uma funo, pois pode haver
mais de um ponto com a dada temperatura ou ainda uma temperatura
que no atingida por nenhum ponto da sala.

Exemplo A.4. Vamos agora dar outro exemplo em que no temos


uma funo, isto , cuja a nossa aparente correspondncia no de
fato uma correspondncia, pois no associa a cada elemento x do do-
mnio um nico elemento f (x) do contradomnio. Para tanto, xe o
domnio como sendo o conjunto dos nmeros reais no intervalo [0, 1]
e como contradomnio o conjunto denido pelas sequncias de ele-
mentos no conjunto {0, 1, 2, . . . , 9}. Ou seja,
282 A Apndice: Funes


= (a1 , a2 , a3 , . . . ); ai {0, 1, 2, 3, . . . , 9} .
Cada elemento x [0, 1] possui uma expanso decimal x = 0, x1 x2 x3 . . . .
Dena f : [0, 1] colocando f (x) = (x1 , x2 , x3 , . . . ).
A princpio, parece que f denida desse modo uma funo. Po-
rm, olhando de perto vemos que o nmero 0, 1 possui mais de uma
representao na base decimal, pois 0, 1 = 0, 09999 . . . . Portanto, f
no est bem denida, isto , f no associa a cada elemento de [0, 1]
um nico elemento de .
Denio A.5. Dada uma funo f denida por
f : AB
x f (x)
o conjunto imagem de f o subconjunto f (A) do contradomnio B
formado pelos pontos y do contradomnio tais que existe algum ponto
x no domnio A tal que y = f (x). Ou seja
f (A) = {y B; existe x A tal que y = f (x)}.
imagem de um ponto x A o ponto f (x). Denimos tambm
A

a restrio de f a um subconjunto A de seu domnio a nova funo


0

denida considerando-se o domnio como sendo o conjunto A0 e os

demais elementos os mesmos. Denotamos essa nova funo por f |A0


ou ainda
f |A0 : A0 B
x f (x)
No Exemplo A.2 poderamos trocar o domnio por um de seus

subconjuntos no vazios. Por exemplo, poderamos considerar o sub-

conjunto A de P formado pelas pessoas do Brasil que nasceram em

Alagoas.
283

Denio via Relaes


Um modo mais formal de denir funo usar a noo de relao

entre dois conjuntos A e B. Uma relao entreA e B simplesmente


um subconjunto R do produto cartesiano A B . Uma funo uma
relao R entre A e B que satisfaz duas condies:

R unvoca: dados x1 , x2 A e yB tais que (x1 , y) R e

(x2 , y) R ento x1 = x2 ;

R total: dado x A existe y B tal que (x, y) R. de modo


que dado x A, existe um nico y B tal que (x, y) R.

Funes Injetoras, Sobrejetoras e Bijetoras


Denio A.6. Uma funo f : A B dita injetora (ou injetiva )
se a seguinte propriedade vale:

Dados x, y A tais que f (x) = f (y), ento x = y.

Outro modo equivalente de formular tal propriedade usando sua


forma contrarrecproca:

Se x, y A so tais que x 6= y, ento f (x) 6= f (y).

Exemplo A.7. Por exemplo, a funo f : R R dada por f (x) = x2


no injetora, pois f (1) = (1)2 = 12 = f (1).
Por outro lado, se g : [0, +) R dada por g(x) = x2 , ento g
injetora, pois dados dois nmeros no negativos a e b tais que g(a) =
g(b), isto , a2 = b2 , ento a2 b2 = 0, de onde (a b)(a + b) = 0,
restando as possibilidades a = b ou a = b. Como a e b so positivos,
temos que a = b.
284 A Apndice: Funes

Denio A.8. Uma funo f : A B dita sobrejetora (ou so-

brejetiva ) se a seguinte propriedade vale:

Dado y B existe x A, tal que f (x) = y.

Exemplo A.9. Por exemplo, a funo f : R R dada por f (x) =


x2 do exemplo anterior no sobrejetora, pois no existe nenhum
nmero real x tal que f (x) = 1, por exemplo. Por outro lado, se
considerarmos g(x) : R [0, +) dada por g(x) = x2 , ento g
sobrejetora, pois dado qualquer nmero no negativo b, podemos tomar

a como sendo a = b de modo que g(a) = a2 = b.

Denio A.10. Uma funo dita bijetora (ou ainda bijetiva ) se


ela injetora e sobrejetora.

Por exemplo, a funof : R R dada por f (x) = x3 uma funo


bijetora, pois injetora e sobrejetora, j que dado y R, existe um
3
nico x R tal que y = x .

Quando f : A B bijetora, ento dado qualquer elemento y

B , existe um elemento x A tal que f (x) = y (pois f sobrejetora) e


esse elemento nico (pois f injetora). Em outros termos, podemos

denir uma nova funo: g : B A associando a cada elemento y B

o nico elemento x em A tal que f (x) = y . Em outras palavras,

g(y) = x, se e somente se, f (x) = y.

g chamada de funo inversa de f .


Quando existe uma bijeo f entre dois conjuntos A e B, dizemos

que A e B tm a mesma quantidade de elementos ou cardinalidade.


Para mais informaes sobre funes, recomendamos a leitura de [3].
Referncias Bibliogrcas
[1] AIGNER, M. e ZIEGLER, G. (2002). As Provas esto
no Livro. Edgard Blcher.

[2] GARCIA, A. e LEQUAIN, I. (2003). Elementos de l-


gebra. Projeto Euclides, IMPA.

[3] LIMA, E. L.; CARVALHO, P. C. P.; WAGNER, E. e

MORGADO, A.C. (2004). A Matemtica do Ensino M-


dio. Volume 1. Sociedade Brasileira de Matemtica.

[4] LIMA, E.L.; CARVALHO, P. C. P.; WAGNER, E. e

MORGADO, A.C. (2004). A Matemtica do Ensino M-


dio. Volume 2. Sociedade Brasileira de Matemtica.

[5] LIMA,E.L.; CARVALHO,P. C. P.; WAGNER,E. e

MORGADO,A.C. (2004). A Matemtica do Ensino M-


dio. Volume 3. Sociedade Brasileira de Matemtica.

[6] LIMA, E.L.; CARVALHO, P. C. P.; WAGNER,E. e

MORGADO, A.C. (2001). Temas e Problemas. Socie-

dade Brasileira de Matemtica.

[7] LIMA, E.L. (2001). lgebra Linear. Sociedade Brasileira


de Matemtica.

285
286 REFERNCIAS BIBLIOGRFICAS

[8] MORAIS FILHO, D. C. (2007). Um Convite Matem-


tica. EDUFCG.

[9] MORGADO, A.; CARVALHO, J.; CARVALHO, P.;

FERNANDEZ, P. (1991). Anlise Combinatria e Pro-


babilidade . Sociedade Brasileira de Matemtica.

[10] RIBENBOIM, P. (2001). Nmeros Primos: Mistrios e


Recordes. Sociedade Brasileira de Matemtica.

[11] SANTOS, J. P. O. (1993) Introduo Teoria dos N-


meros. IMPA.

[12] SANTOS, J. P. O.; MELLO, M. P. e MURARI, I. T.

C. (2006). Introduo Anlise Combinatria. Editora

Unicamp.

[13] SOARES, M. G. (2005). Clculo em uma Varivel Com-


plexa. Sociedade Brasileira de Matemtica.
Resoluc
ao de Problemas
Lista 01

Relembramos algumas dicas discutidas no livro-texto para ajudar na res-


oluc
ao de um problema em Matematica.

(D1) Ler bem o enunciado do problema e utilizar todas as informacoes disponveis.

(D2) Resolver casos particulares ou casos mais simples de problemas similares,


para adquirir familiaridade com o problema.

(D3) Mudar a representac


ao do problema, transformando-o em um problema
equivalente.

(D4) Usar a imaginac


ao, pesquisando caminhos alternativos.

Alem dessas dicas, vamos acrescentar mais algumas outras:

(D5) Reuna todos os fatos que voce conhece e que estejam relacionados ao
problema; Em seguida, selecione alguns deles que voce ache que possam
servir para resolver o problema.
(D6) Se o problema permitir, ache um invariante. Isto e, uma quantidade ou
propriedade que nao muda e que possa ser usada para resolver seu prob-
lema.

(D7) Se for possvel e conveniente, comece de tras-para-frente, iniciando na


situac
ao final onde voce deseja chegar e, a partir de jogadas validas, tente
chegar na situacao inicial.
(D8) Antes de avancar demais, cheque o trabalho ja desenvolvido, verificando
se voce cometeu algum erro. Isso lhe poupara tempo.

Claramente, nenhuma dessas dicas faria sentido se lida isoladamente e de-


sconectada de um problema especfico. Alem disso, nenhuma delas e universal,
permitindo-lhe tornar-se um grande resolvedor de problemas. Na verdade, re-
solver problemas e algo que so se aprende fazendo. Porem, adotar estrategias
como as que listamos acima pode ajuda-lo a ganhar tempo. A seguir, vamos
listar alguns problemas para que voce tente empregar as estrategias descritas
acima e, eventualmente, outras que voce venha a criar. Boa sorte!
Buscando um Invariante
Em algumas situac
oes, a busca de uma quantidade ou de uma propriedade que
n
ao muda quando um processo ocorre, pode levar `a solucao do problema. Essa
quantidade e chamada de invariante. Isso e particularmente verdadeiro quando
consideramos problemas quem involvem impossibilidades, como os que vamos
descrever abaixo. Aplique a dica (D6) aos problemas 5, 6, 7 e 8.

1. Seis pessoas formando um crculo seguram pequenos quadros em suas


maos, nos quais estao escritos n
umeros. A cada rodada, escolhe-se uma
das pessoas e adiciona-se uma unidade ao numero escrito no quadro dessa
pessoa, bem como uma unidade aos n umeros nos quadros de seus vizinhos.

(a) Se os n
umeros iniciais forem 1,0,1,0,0,0 e possvel, apos repetir esse
procedimento um certo n umero de vezes, fazer com que todos os
quadros tenham os mesmos n umeros?
(b) Se os numeros iniciais forem 5,2,0,3,5,6 mostre que e possvel, apos
repetir esse procedimento um certo n umero de vezes, fazer com que
todos os quadros tenham os mesmos n umeros.
(c) Se os n
umeros iniciais forem 5,2,0,3,5,6, qual o menor n
umero de
jogadas de modo que todos os quadros tenham os mesmos n umeros?

2. Invente e resolva um problema, usando como inspiracao o problema ante-


apropriado para que
rior. Decida o grau de dificuldade da resolucao. E
ano? Discuta com seus colegas do PROFMAT.
3. Num tabuleiro de xadrez 8 8 e permitido escolher um quadrado 2 2
qualquer e trocar as cores de uma das linhas ou colunas deste quadrado.
possvel que se chegue a uma situacao na qual todos os quadrados do
E
tabuleiro 8 8 sejam brancos, exceto um?

4. Invente e resolva um problema, usando como inspiracao o problema ante-


apropriado para que
rior. Decida o grau de dificuldade da resolucao. E
ano? Discuta com seus colegas do PROFMAT.
5. Os numeros 1, 2, 3, . . . , 99 sao escritos no quadro-negro e e permitido re-
alizar a seguinte operacao: apagar dois deles e substitu-los pela diferenca
do maior com o menor. Fazemos esta operacao sucessivamente ate restar
apenas um u ltimo n umero no quadro. Pode o u ltimo n
umero que restou
ser o zero?
Resolvendo de Tr
as-para-frente
Andar para tras pode ser uma ferramenta importante na solucao de um
problema. A tecnica consiste em supor que chegamos ao objetivo que
pretendemos (posicao vitoriosa, por exemplo, ou uma configuracao espe-
cial) e, fazendo movimentos para tras, tentamos chegar na situacao inicial.
Resolva o seguinte problema usando a dica (D7):
6. Joao e Maria brincam com um monte de 30 palitos. E permitido a cada
um deles retirar no seu turno 1, 2 ou 3 palitos. Ganha quem retirar o
u
ltimo palito. Sabendo que Joao comeca e que os dois aprenderam a jogar
com o mestre sabetudo, quem ganha o jogo? E retirando-se 1,2,3 ou 4
palitos, quem ganharia?
O jogo anterior se inclui numa classe chamada de jogos progressivamente
finitos. Esta classe e constituida de jogos que necessariamente acabam
apos um n
umero de jogadas. Outro exemplo deste tipo de jogo e o seguinte:
7. Suponha agora que na pilha existam 107 palitos e que a cada rodada, Joao
e Maria se alternam, escolhendo um primo p, n 0 e inteiro nao-negativo
e retirando pn palitos. Ganha quem retirar o u
ltimo palito. Pergunta-se:
se Joao comeca o jogo, quem ganha?
8. Agora, Joao e Maria dispoem de dois montes com 30 palitos cada. Em
cada turno, o jogador escolhe somente um dos montes e retira quantos
palitos quiser, inclusive o monte inteiro. Ganha quem retirar o u
ltimo
palito. Sabendo que Joao comeca, quem ganha o jogo?
9. Joao e Maria se alternam desenhando diagonais de um 2012-agono. Perde
quem desenhar uma diagonal cruzando alguma outra ja desenhada. Qual
e a estrategia vitoriosa para esse jogo?

Usando equac
oes
10. Demonstre que:

(a) n4 + 4 nao e primo se n > 1;


(b) Generalize, mostrando que n4 + 4n nao e primo, para todo n > 1.
11. Nove copias de certas notas custam menos de R$ 10,00 e dez copias das
mesmas notas (com o mesmo preco) custam mais de R$ 11,00. Quanto
custa uma copia das notas?

12. As paginas de um livro sao numeradas de 1 ate n. Ao somarmos estes


numeros, por engano um deles e somado duas vezes, obtendo-se o resultado
incorreto: 2.012. Qual e o n
umero da pagina que foi somado duas vezes?
13. Analise variac
oes deste problema. Por exemplo, permita que se some dois
n
umeros consecutivos duas vezes, ou ainda, o mesmo n umero tres vezes.
Mude o valor da soma. Para cada mudanca que voce fizer, discuta com
seus colegas do PROFMAT a serie onde um desafio desse poderia ser
proposto.

14. Mostre que entre os retangulos com um mesmo permetro, o de maior area
e um quadrado.

15. Mostre que para quaisquer a, b, c reais vale

a2 + b2 + c2 ab + ac + bc.

16. Usando cada dgito 1,2,3,4,5,6,7,8, 9 somente uma vez, decida se e possvel
escrever n
umeros de modo que sua soma seja 100.

17. Invente e resolva um problema, usando como inspiracao o problema ante-


apropriado para que
rior. Decida o grau de dificuldade da resolucao. E
ano? Discuta com seus colegas do PROFMAT.

18. Joao esta na beira de um rio com dois baldes de 9 e 4 litros, sem marcacoes.
Ele deseja medir exatamente 6 litros de agua para poder levar para Maria
fazer uma deliciosa sopa para sua numerosa famlia. Mostre como Joao
devera proceder para obter os 6 litros de agua.

19. Analise variac


oes deste problema. Por exemplo, permita que a quantidade
a ser separada seja diferente de 6, ou ainda, que os baldes tenham outra
capacidade. Por exemplo, discuta o que ocorre quando os baldes tem
MDC diferente de 1. Decida o grau de dificuldade da resolucao de cada
problema que voce criar. E apropriado para que ano? Discuta com seus
colegas do PROFMAT.

20. Faca mentalmente as seguintes multiplicacoes:

(a) 27 37
(b) 21 23

21. Invente e resolva um problema, usando como inspiracao o problema ante-


apropriado para que
rior. Decida o grau de dificuldade da resolucao. E
ano? Discuta com seus colegas do PROFMAT.
Sugest
oes de Leitura
Revista Eureka!
Na revista Eureka! e possvel encontrar varios problemas interessantes e
desafiadores, alem de artigos que tratam de assuntos que nao sao rotineiros
na sala de aula. A revista esta disponvel no site www.obm.org.br, onde
podem ser encontradas as provas (muitas resolvidas) da Olimpada Brasileira
de Matematica.

Colec
ao de Iniciac
ao Cientfica J
unior da OBMEP
A colec
ao e destinada aos alunos premiados na Olimpada Brasileira de
Matem atica das Escolas P
ublicas. Pode ser baixada em http://www.obmep.org.br,
onde e possvel tambem obter um banco de problemas e solucoes.

Tio Petros http://tiopetrus.blogia.com/


Um blog interessante, com varios recursos que podem ser aproveitados.
Cabe ao leitor uma leitura crtica.
Alem da colec ao Olimpadas de Matem atica e da colecao Professor de
Matem atica, que podem ser adquiridos no site www.sbm.org.br, recomendo
a leitura dos seguintes livros:

How to solve it - G. Polya, Princeton Science Library. Princeton University


Press, 1945.
Excelente livro sobre didatica da Matematica e a arte de resolver proble-
mas. O autor apresenta seu ponto de vista sobre as tecnicas de resolucao
de problemas, com alguns exemplos discutidos. A discussao envolve prob-
lemas de todos os nveis (Fundamental e Medio). Sem traducao (que eu
conheca) para o portugues.

Mathematical Circles: Russian Experience (Mathematical World, Vol. 7)


Dimitri Fomin, Sergey Genkin, Ilia V. Itenberg.
Excelente livro, com varios problemas interessantes para ampliar a visao
do estudante. Esta dividido em duas partes, sendo a primeira dedicada
aos primeiros anos do Fundamental II. Os problemas sao daqueles que
consideramos de raciocnio, privilegiando a criatividade e ideias novas dos
estudantes. Pode ser usado como livro para um clube ou grupo especial
de treinamento de Matematica na sua escola.
Problem Solving Strategies - A. Engel. Problem Books in Mathematics,
Springer.
Uma coletanea de exerccios, em geral do nvel de olimpadas de matematica.
Est
a separado por temas (geometria, inducao etc) e com dicas e resolucoes
para os exerccios propostos. Pode ser usado numa turma avancada de
preparacao para olimpadas ou exames difceis.
Resoluc
ao de Problemas
Lista 01 com dicas e discuss
ao

Faca mentalmente as seguintes multiplicacoes:

1. 27 37
2. 21 23

Invente e resolva um problema, usando como inspiracao o problema anterior.


apropriado para que ano? Discuta
Decida o grau de dificuldade da resolucao. E
com seus colegas do PROFMAT.
Dica 1. Para o item a), note que 3 37 = 111. Para o item b), lembre-se que
212 = 441.
Discuss ao 1. A ideia desse problema e simplesmente ilustrar que conhecimento
previo relacionado ao que procuramos pode ajudar na resolucao de um problema.
Creio que e o tipo problema que pode ser usado em qualquer serie, especialmente
as menores do Fundamental II. Discuta com seus colegas a respeito. Comecar
do zero um problema pode tornar mais difcil encontrar a solucao do problema.
Uma avaliac ao criteriosa de fatos conhecidos ou de problemas semelhantes pode
nos ajudar a encontrar a solucao mais rapidamente.

Buscando um Invariante
Em algumas situac
oes, a busca de uma quantidade ou de uma propriedade que
n
ao muda quando um processo ocorre, pode levar `a solucao do problema. Essa
quantidade e chamada de invariante. Isso e particularmente verdadeiro quando
consideramos problemas quem involvem impossibilidades, como os que vamos
descrever abaixo. Aplique a dica (D6) aos problemas 5, 6, 7 e 8.

1. Seis pessoas formando um crculo seguram pequenos quadros em suas


maos, nos quais estao escritos n
umeros. A cada rodada, escolhe-se uma
das pessoas e adiciona-se uma unidade ao numero escrito no quadro dessa
pessoa, bem como uma unidade aos n umeros nos quadros de seus vizinhos.

(a) Se os n
umeros iniciais forem 1,0,1,0,0,0 e possvel, apos repetir esse
procedimento um certo n umero de vezes, fazer com que todos os
quadros tenham os mesmos n umeros?
(b) Se os numeros iniciais forem 6,3,0,0,3,6 mostre que e possvel, apos
repetir esse procedimento um certo n umero de vezes, fazer com que
todos os quadros tenham os mesmos n umeros.
(c) Se os n
umeros iniciais forem 6,3,0,0,3,6, qual o menor n
umero de
jogadas de modo que todos os quadros tenham os mesmos n umeros?
Dica 2. Observe o resto na divisao por 3 da soma dos n
umeros nos quadros
e um invariante.
Discussao 2. A nocao-chave aqui e o que vem a ser um invariante. E im-
portante frisar que algo n
ao muda quando realizamos uma jogada. Neste
caso, para o primeiro item, o invariante em questao e o resto da divisao
da soma total por 3. Como a soma da configuracao inicial deixa resto 2
quando dividida por 3, temos que e impossvel chegar na situacao onde to-
dos os n
umeros dos quadros sao iguais comecando com 1,0,1,0,0,0, ja que
se todos os numeros sao iguais, o resto da soma e zero quando dividida
por 3.
Para o segundo item, basta fazer uma construcao. Em geral, o aluno
comeca com o processo tentativa-e-erro, ate pegar o jeito e inferir que jo-
importante guiar o aluno
gadas sao interessantes para atingir o objetivo. E
neste processo, estimulando a inferencia de propriedades gerais. Isso per-
mite, por exemplo, ter ideias sobre o terceiro item. Por exemplo, ele pode
perceber que os n umeros so aumentam a cada jogada. Isso permite inferir
que o numero mnimo de jogadas deve ser maior igual a (36-18)/3=6. Isso
se deve ao fato que a primeira configuracao que talvez seja possvel que
igual todos os numeros e 6,6,6,6,6,6 (soma 36) e que a configuracao inicial
6,3,0,0,3,6 tem soma 18. Assim, como a cada jogada acrescentamos 3, o
mnimo e maior ou igual a 6. De fato, 6 e o mnimo pois uma sequencia
que resolve o problema e

6, 3, 0, 0, 3, 6 6, 4, 1, 1, 3, 6 6, 4, 2, 2, 4, 6

6, 5, 3, 3, 4, 6 6, 5, 4, 4, 5, 6 6, 6, 5, 5, 5, 6 6, 6, 6, 6, 6, 6.

2. Invente e resolva um problema, usando como inspiracao o problema ante-


apropriado para que
rior. Decida o grau de dificuldade da resolucao. E
ano? Discuta com seus colegas do PROFMAT.

3. Num tabuleiro de xadrez 8 8 e permitido escolher um quadrado 2 2


qualquer e trocar as cores de uma das linhas ou colunas deste quadrado.
possvel que se chegue a uma situacao na qual todos os quadrados do
E
tabuleiro 8 8 sejam brancos, exceto um?

Dica 3. Olhe a paridade da diferenca entre o n


umero de quadrados bran-
cos e quadrados pretos.

Discuss ao 3. A paridade da diferenca entre o n


umero de quadrados bran-
cos e quadrados pretos nao muda a cada operacao permitida. Como no
incio temos 32 quadrados brancos e 32 quadrados pretos, a cada passo
teremos sempre um n umero PAR como resultado da diferenca entre os
quadrados brancos e os quadrados pretos, mostrando que e impossvel
atingir a configurac
ao pedida.
4. Invente e resolva um problema, usando como inspiracao o problema ante-
apropriado para que
rior. Decida o grau de dificuldade da resolucao. E
ano? Discuta com seus colegas do PROFMAT.

5. Os numeros 1, 2, 3, . . . , 99 sao escritos no quadro-negro e e permitido rea-


lizar a seguinte operacao: apagar dois deles e substitu-los pela diferenca
do maior com o menor. Fazemos esta operacao sucessivamente ate restar
apenas um u ltimo n umero no quadro. Pode o u ltimo n
umero que restou
ser o zero?

Resolvendo de Tr
as-para-frente
Andar para tras pode ser uma ferramenta importante na solucao de um
problema. A tecnica consiste em supor que chegamos ao objetivo que
pretendemos (posicao vitoriosa, por exemplo, ou uma configuracao espe-
cial) e, fazendo movimentos para tras, tentamos chegar na situacao inicial.
Resolva o seguinte problema usando a dica (D7):
6. Joao e Maria brincam com um monte de 30 palitos. E permitido a cada
um deles retirar no seu turno 1, 2 ou 3 palitos. Ganha quem retirar o
u
ltimo palito. Sabendo que Joao comeca e que os dois aprenderam a jogar
com o mestre sabetudo, quem ganha o jogo? E retirando-se 1,2,3 ou 4
palitos, quem ganharia?

Dica 4. Para o jogo retirando-se 1,2 ou 3 palitos, tente resolver o problema


fazendo uso do mnimo de itens da lista abaixo.

(a) Reduza o n
umero de palitos no monte.
(b) Tente com 7 e depois passe para 10 palitos.
(c) Analise o que acontece quando seu adversario esta na posicao 4.
(d) Analise o que acontece quando seu adversario esta na posicao 8.
(e) Analise o que acontece quando seu adversario esta em uma posicao
m
ultiplo de 4.

Discuss ao 4. A estrategia vencedora e deixar seu adversario numa posicao


multiplo de 4. Verifique isso tracando uma arvore de possveis caminhos a
partir de uma dada posicao, onde a posicao 0 significa vitoria. Isso pode
ser feito construindo-se um grafo orientado, isto e, um conjunto de pontos
(posic
oes) e setas indicando as jogadas permitidas.

O jogo anterior se inclui numa classe chamada de jogos progressivamente


finitos. Esta classe e constituida de jogos que necessariamente acabam
apos um n
umero de jogadas. Outro exemplo deste tipo de jogo e o seguinte:
7. Suponha agora que na pilha existam 107 palitos e que a cada rodada, Joao
e Maria se alternam, escolhendo um primo p, n 0 e inteiro nao-negativo
e retirando pn palitos. Ganha quem retirar o u
ltimo palito. Pergunta-se:
se Joao comeca o jogo, quem ganha?
Dica 5. Observe que o primeiro n umero que nao e potencia de primo e o
6. Ou seja, se Joao deixa Maria com 6 palitos na jogada dela, Joao ganha.
Discuss ao 5. O conjunto de posicoes vitoriosas consiste nos m ultiplos de
6. De fato, se Joao deixa Maria com um m ultiplo de 6 palitos na jogada
dela, depois de Maria retirar uma potencia de primo, Joao pode retornar
a uma posic
ao que e multiplo de 6 retirando 1,2,3,4 ou 5 palitos. Assim,
no final Maria estara na posicao 6 e Joao ganhara o jogo.

8. Agora, Joao e Maria dispoem de dois montes com 30 palitos cada. Em


cada turno, o jogador escolhe somente um dos montes e retira quantos
palitos quiser, inclusive o monte inteiro. Ganha quem retirar o u
ltimo
palito. Sabendo que Joao comeca, quem ganha o jogo?

Dica 6. Olhe na expressao dos montes quando escritos na base 2.


Discuss ao 6. A estrategia vencedora desse jogo e deixar seu oponente
numa situac ao de modo que quando somamos os dgitos na base dois
dos montes relativos a mesma potencia, o resultado e par. Ou seja, os
dgitos sao iguais. Por exemplo, se os montes tem 15 e 21 palitos, primeiro
escrevemos eles na base dois:

15 = (1111)2 e 21 = (10101)2 .

Em seguida, escrevemos a soma

10101
+1111

11212

Chamaremos de posicao vencedora, aquela que obtiver como resultado das


somas das colunas apenas os dgitos 2 e 0. As demais sao as perdedoras.
Por exemplo, a posic ao acima (um monte com 15 e outro com 21) e posicao
perdedora. Para vencer o jogo, basta o jogador transformar este resultado
(11212) numa posic ao vencedora, retirando palitos. Por exemplo, pode-
mos retirar 6 palitos do monte com 21 para deixa-lo com 15 tambem.
Assim, a soma coluna a coluna dos dgitos sera (2222). Para mostrar que
a estrategia e a vitoriosa, voce deve verificar que
A partir de uma posicao vencedora, sempre podemos ir para uma
posic
ao perdedora.
A partir de uma posicao perdedora, nunca podemos ir para uma
posic
ao vencedora
Como o zero e uma posicao vencedora, essa e a estrategia vitoriosa do
jogo.

9. Joao e Maria se alternam desenhando diagonais de um 2012-agono. Perde


quem desenhar uma diagonal cruzando alguma outra ja desenhada. Qual
e a estrategia vitoriosa para esse jogo?
Dica 7. Observe que o que acontece quando um jogador traca uma di-
agonal principal, isto e, aquela que divide o polgono em dois polgonos
com a mesma quantidade de vertices.
Discuss ao 7. Como temos uma quantidade par de vertices, e possvel
tracar alguma diagonal principal. Digamos que Joao tenha tracado uma
diagonal principal, dividindo o 2012-agono em dois 1006-agonos, P1 e P2 .
A seguir, observe que as diagonais que nao cruzam a diagonal principal
corresponde exatamente `as diagonais de um dos dois polgonos obtidos.
Assim, para cada diagonal de Maria que nao cruza a diagonal principal,
Joao podera tracar uma diagonal correspondente no outro polgono. Ao
final, Maria tera que cruzar a diagonal principal tracada.

Usando equac
oes do primeiro e segundo graus
10. Nove copias de certas notas custam menos de R$ 10,00 e dez copias das
mesmas notas (com o mesmo preco) custam mais de R$ 11,00. Quanto
custa uma copia das notas?
11. As paginas de um livro sao numeradas de 1 ate n. Ao somarmos estes
numeros, por engano um deles e somado duas vezes, obtendo-se o resultado
incorreto: 2.012. Qual e o n
umero da pagina que foi somado duas vezes?
Discussao 8. Primeiramente, relembre que a soma dos n primeiros n
umeros
e S = n(n + 1)/2. Observe tambem a soma efetuada incorretamente esta
entre S + 1 e S + n. Assim, sabemos que S + 1 2012 S + n, ou seja,
n2 + n + 2 4024 n2 + 3n

Testando os valores de n2 + n + 2 temos que n = 62, ja que 61 e pouco


e 63 e demais e a func
ao e crescente. Portanto, o valor desejado e obtido
fazendo 2012 S = 58.

12. Analise variac


oes deste problema. Por exemplo, permita que se some dois
n
umeros consecutivos duas vezes, ou ainda, o mesmo n umero tres vezes.
Mude o valor da soma. Para cada mudanca que voce fizer, discuta com
seus colegas do PROFMAT a serie onde um desafio desse poderia ser
proposto.
13. Determine os valores de a para os quais a funcao quadratica ax2 ax + 12
e sempre positiva.
14. Mostre que entre os retangulos com um mesmo permetro, o de maior area
e um quadrado.
15. Mostre que para quaisquer a, b, c reais vale

a2 + b2 + c2 ab + ac + bc.

16. Usando cada dgito 1,2,3,4,5,6,7,8, 9 somente uma vez, decida se e possvel
escrever n
umeros de modo que sua soma seja 100.
Discuss ao 9. A resposta deste problema e negativa, ou seja, e impossvel
realizar o que se pede. De fato, primeiramente observe que todos os
numeros construdos tem que ser menores que 100. Portanto, cada n umero
tem no maximo dois dgitos. Digamos que os dgitos que forem usados na
casa das unidades dos n umeros construdos formem um conjunto com soma
t. Naturalmente, o conjunto dos n umeros que foram usados nos algaris-
mos das dezenas dos n umeros construdos formara um conjunto com soma
45 t, ja que 1 + 2 + 3 + . . . + 9 = 45. Portanto, teremos que a soma sera

(45 t)10 + t = 100,

e essa equac
ao do primeiro grau nao tem solucoes inteiras.

17. Invente e resolva um problema, usando como inspiracao o problema ante-


apropriado para que
rior. Decida o grau de dificuldade da resolucao. E
ano? Discuta com seus colegas do PROFMAT.
18. Joao esta na beira de um rio com dois baldes de 9 e 4 litros, sem marcacoes.
Ele deseja medir exatamente 6 litros de agua para poder levar para Maria
fazer uma deliciosa sopa para sua numerosa famlia. Mostre como Joao
devera proceder para obter os 6 litros de agua.
19. Analise variac
oes deste problema. Por exemplo, permita que a quantidade
a ser separada seja diferente de 6, ou ainda, que os baldes tenham outra
capacidade. Por exemplo, discuta o que ocorre quando os baldes tem
MDC diferente de 1. Decida o grau de dificuldade da resolucao de cada
problema que voce criar. E apropriado para que ano? Discuta com seus
colegas do PROFMAT.
Resoluc
ao de Problemas
Lista 02

Divisibilidade
1. Prove que se n e mpar
(a) n2 1 e divisvel por 8;
(b) n3 n e divisvel por 24;
(c) n2 + (n + 2)2 + (n + 4)2 + 1 e divisvel por 12.
2. Tres n
umeros primos p, q e r, maiores que 3, formam uma progressao
aritmetica, ou seja, q = p + d e r = p + 2d. Prove que d e divisvel por 6.1
3. Mostre que 311 3 e divisvel por 112 .
4. Um robo possui dois botoes, permitindo que a cada momento ele suba a
degraus ou desca b degraus de uma escada com infinitos degraus. Sabendo
que o robo esta no incio da escada, pergunta-se:

(a) Se a = 12 e b = 3, e possvel que o robo visite todos os degraus apos


uma sucessao desses movimentos?
(b) Mostre que se a e b sao primos entre si, o robo consegue visitar todos
os degraus.

5. Encontre o resto que deixa


(a) 2001 2002 2003 2004 + 20052 quando e dividido por 7;
(b) 2100 quando e dividido por 3;
28
(c) 1237156 + 34 quando e dividido por 111.
6. Encontrar o u
ltimo dgito dos n
umeros
(a) 19892005 ;
(b) 777777 + 250 ;
(c) 1 + 22 + 32 + + 20052 .
7. Verifique que
111
| {z. . . 1} = 222
| {z. . . 2} +(333
| {z. . . 3})2 .
2012 uns 1006 dois 1006 tr
es
1 Foi demonstrado em 2004 pelos Matem aticos B. Green e T. Tao que existem progress
oes
aritmeticas de tamanho arbitrariamente grande formadas somente por primos. A prova pode
ser encontrada em www.arxiv.org.
8. Demonstre que o n
umero 1 000
| {z. . . 00} 1 e composto.
2012 zeros

9. Considere o polinomio p(n) = am nm +am1 nm1 + +a0 de grau m 1


com coeficientes inteiros e n N. Prove que p(n) e um n
umero composto
para infinitos valores de n.2

10. Prove que se (x0 , y0 ) e uma solucao da equacao diofantina linear ax by =


1, ent
ao a area do triangulo cujos vertices sao (0, 0), (b, a) e (x0 , y0 ) e
1/2.
11. Ao entrar numa sala, Joao se depara com 100 interruptores e 100 lampadas,
numerados de 1 a 100. O interruptor n acende somente a lampada n, para
cada valor de n = 1, 2, . . . , 100. De incio, todas as 100 lampadas estao
apagadas. Joao aperta todos os interruptores m ultiplos de 2. A seguir,
aperta todos os multiplos de 3, e assim sucessivamente, ate o u nico inter-
ruptor multiplo de 100. Ao final deste procedimento, pergunta-se:

(a) Quais lampadas estao apagadas?


(b) Quantas lampadas acenderam exatamente 4 vezes?
(c) Qual e o n
umero da lampada que mais acendeu?

ao: Use o fato de que existe a N tal que = |p(a)| > 1 e mostre que divide a
2 Sugest

p(k + a), para todo k Z.


Princpio das Casas dos Pombos
12. Seja C um conjunto formado por cinco pontos de coordenadas inteiras no
plano. Prove que o ponto medio de algum dos segmentos com extremos
em C tem tambem coordenadas inteiras.
13. O conjunto dos dgitos 1, 2, ..., 9 e dividido em tres grupos. Prove que o
produto dos n
umeros de algum dos grupos deve ser maior que 71.
14. Prove que se N e mpar entao para qualquer bijecao

p : IN IN

do conjunto IN = {1, 2, . . . , N } o produto P (p) = (1p(1))(2p(2)) (N


p(N )) e necessariamente par.3
15. Dado um conjunto de 25 pontos no plano tais que entre quaisquer 3 deles
existe um par com distancia menor que 1. Prove que existe um crculo de
raio 1 que contem pelo menos 13 dos 25 pontos dados.

16. Marquemos todos os centros dos 64 quadradinhos de um tabuleiro de


xadrez de 8 8. E
possvel cortar o tabuleiro com 13 linhas retas que nao
passem pelos pontos marcados e de forma tal que cada pedaco de recorte
do tabuleiro tenha no maximo um ponto marcado?

17. Prove que existem duas potencias de 3 cuja diferenca e divisvel por 2012.
18. Mostre que entre nove numeros que nao possuem divisores primos maiores
que cinco, existem dois cujo produto e um quadrado.
19. Um disco fechado de raio um contem sete pontos, cujas distancias entre
quaisquer dois deles e maior ou igual a um. Prove que o centro do disco e
um destes pontos.
20. Seja a um n umero irracional. Prove que existem infinitos n
umeros racio-
nais r = p/q tais que |a r| < 1/q 2 .
21. Suponha que cada ponto do reticulado4 plano e pintado de vermelho ou azul.
Mostre que existe algum ret
angulo com vertices no reticulado e todos da mesma
cor.

22. Um certo livreiro vende pelo menos um livro por dia. Sabendo que o livreiro
vendeu 463 livros durante 305 dias consecutivos, mostre que em algum perodo
de dias consecutivos o livreiro vendeu exatamente 144 livros.

3 Dica: O produto de varios fatores


e par se, e somente se, um dos fatores
e par.
4 Reticulado refere-se ao conjunto dos pontos do plano com coordenadas inteiras
Resoluc
ao de Problemas
Lista 02

Solu
coes dos Exerccios de Divisibilidade
1. Prove que se n e mpar
(a) n2 1 e divisvel por 8;

Demonstrac
ao. Como n e mpar, temos que n = 2q + 1 para algum
q Z. Assim,

n2 1 = (2q + 1)2 1 = 4q 2 + 4q + 1 1 = 4q 2 + 4q = 4q(q + 1).

Como q e q + 1 sao consecutivos, um deles tem que ser par. Logo,


n2 1 e divisvel por 8.

(b) n3 n e divisvel por 24;

Demonstrac
ao. Usaremos o fato que
se a, b s
ao primos entre si, entao c e divisvel por ab se, e somente
se, c e divisvel por a e por b.
Assim, para mostrar que n3 n e m ultiplo de 24, basta mostrar que
n3 n e m
ultiplo de 3 e de 8, uma vez que 3 e 8 sao primos entre si.
Observe que n3 n = n(n2 1). Como n e mpar, segue do exerccio
anterior que n3 n e divisvel por 8. A divisibilidade por 3 decorre
do lema dos restos. De fato, se n 0 mod 3, n3 n sera m ultiplo
de 3, claramente. Se n 1 mod 3, temos que n3 n sera m ultiplo
de 3, j
a que
13 1 0 mod 3.
Se n 2 mod 3, temos que n3 n sera m
ultiplo de 3, ja que

23 2 0 mod 3,

completando assim a prova.

(c) n2 + (n + 2)2 + (n + 4)2 + 1 e divisvel por 12.


Demonstrac ao. Novamente, usaremos o fato que se a, b sao primos entre
si, entao c e divisvel por ab se, e somente se, c e divisvel por a e por
b. Assim, para mostrar que a expressao e m ultiplo de 12, basta mostrar
que 3 e de 4. Vamos primeiro `a divisibilidade por 4. Observe que como
n e mpar, basta verificar os casos quando n 1 mod 4 e n 3 mod 4.
Quando n 1 mod 4 temos que

n2 + (n + 2)2 + (n + 4)2 + 1 12 + (1 + 2)2 + (1 + 4)2 + 1 36 0 mod 4.

Do mesmo modo, quando n 3 mod 4 temos que

n2 + (n + 2)2 + (n + 4)2 + 1 32 + (3 + 2)2 + (3 + 4)2 + 1 84 0 mod 4.

Para verificar a divisibilidade por 3, repetimos o argumento usando os


ao por 3. Observe que quando n 0 mod 3 temos que
restos da divis

n2 + (n + 2)2 + (n + 4)2 + 1 02 + (0 + 2)2 + (0 + 4)2 + 1 21 0 mod 3.

Procedendo de modo inteiramente analogo para os casos n 1 mod 3 e


n 2 mod 3, completamos a prova.

2. Tres n
umeros primos p, q e r, maiores que 3, formam uma progressao
aritmetica, ou seja, q = p + d e r = p + 2d. Prove que d e divisvel por 6.1

Demonstracao. Temos que d e par, ja que d = q p, com p e q primos


maiores que dois, logo mpares. Basta mostrar que d e divisvel por 3.
Ora, supondo o contrario, temos que d 1 mod 3 ou n 2 mod 3.
Analisando o resto de q e r modulo 3 nas tabelas a seguir (em funcao dos
restos de d nas colunas e p nas linhas).
Para q temos

mod 3 d1 d2
p1 2 0
p2 0 1.

Para r temos:

mod 3 d1 d2
p1 0 1
p2 1 0.
1 Foi demonstrado em 2004 pelos Matem aticos B. Green e T. Tao que existem progress
oes
aritmeticas de tamanho arbitrariamente grande formadas somente por primos. A prova pode
ser encontrada em www.arxiv.org.
Logo, se d n ao e divisvel por 3, ou q ou r serao divisveis por 3, o que e
impossvel, j
a que sao ambos n umeros primos.

3. Mostre que 311 3 e divisvel por 112 .


4. Um rob o possui dois botoes, permitindo que a cada momento ele suba a
degraus ou desca b degraus de uma escada com infinitos degraus. Sabendo
que o robo est
a no incio da escada, pergunta-se:

(a) Se a = 12 e b = 3, e possvel que o robo visite todos os degraus apos


uma sucessao desses movimentos?

Demonstracao. Se apertamos o botao de subir x vezes e o de descer


y vezes, o robo ira subir ou descer 12x 3b degraus. Note que esse
n
umero e sempre m ultiplo de 3, nao sendo possvel para o robo com
uma sequencia desses movimentos atingir um degrau com n umero
que n
ao e divisvel por 3.

(b) Mostre que se a e b sao primos entre si, o robo consegue visitar todos
os degraus.

Demonstrac ao. Como a e b sao primos entre si, pelo Teorema de


Bezout-Bachet, e possvel encontrar x e y de modo que ax by = 1.
Assim, o robo pode sair do nvel inicial e ir para o degrau 1 apertando-
se x vezes o bot
ao de subir e em seguida apertando-se y vezes o botao
de descer. Repetindo esse procedimento, o robo pode atingir qualquer
degrau.

5. Encontre o resto que deixa


(a) 2001 2002 2003 2004 + 20052 quando e dividido por 7;

Demonstracao. Vamos usar o lema dos restos. 2001 quando dividido


por 7 deixa resto 6; Assim, 2002 e divisvel por 7 e 2005 deixa resto
3 quando dividido por 7. Logo, 20052 deixa resto 2, ja que 32 deixa
resto 2 quando dividido por 7.

(b) 2100 quando e dividido por 3;

Demonstracao. 22 resto 1 quando dividido por 3. Logo, 2100 = (22 )5 0


deixa resto 1 quando dividido por 3.
28
(c) 1237156 + 34 quando e dividido por 111.
6. Encontrar o u
ltimo dgito dos n
umeros
(a) 19892005 ;
(b) 777777 + 250 ;
(c) 1 + 22 + 32 + + 20052 .
7. Verifique que
111
| {z. . . 1} = |222{z
. . . 2} +(333
| {z. . . 3})2 .
2012 uns 1006 dois 1006 tr
es

Demonstrac
ao. Observe que

102012 1
111 . . . 1} =
| {z 9
2012 uns

e que

101006 1 101006 1 2
222 . . . 2} = 2 e (333 . . . 3})2 = 9 .
| {z 9 | {z 9
1006 dois 1006 tr
es

Portanto, basta verificar que

102012 1 101006 1 101006 1 2


=2 +9 ,
9 9 9
De fato, cancelando 9 em ambos os lados

102012 1 = 2 (101006 1) + (101006 1)2 ,

como queramos demonstrar.

8. Demonstre que o n
umero 1 000
| {z. . . 00} 1 e composto.
2012 zeros

Demonstracao. Para termos uma ideia da prova deste fato, vamos provar
que 1001 e composto. Ora, temos que

1001 = 11 91.

Do mesmo modo, 100001 = 11 9091 e que 10000001 = 11 909091.


Assim, depois dessa analise inicial, fica bem claro que nosso candidato
para divisor do n
umero pedido e o 11; Podemos verificar sem maiores
dificuldades que

. . . 00} 1 = 11 9090
1 |000 {z | {z . . . 90} 91.
2012 zeros 1005 vezes
9. Considere o polinomio p(n) = am nm +am1 nm1 + +a0 de grau m 1
com coeficientes inteiros e n N. Prove que p(n) e um n
umero composto
para infinitos valores de n.2
10. Prove que se (x0 , y0 ) e uma solucao da equacao diofantina linear ax by =
1, ent
ao a
area do tri angulo cujos vertices sao (0, 0), (b, a) e (x0 , y0 ) e
1/2.

11. Ao entrar numa sala, Joao se depara com 100 interruptores e 100 lampadas,
numerados de 1 a 100. O interruptor n acende somente a lampada n, para
cada valor de n = 1, 2, . . . , 100. De incio, todas as 100 lampadas estao
apagadas. Jo ao aperta todos os interruptores m ultiplos de 2. A seguir,
aperta todos os multiplos de 3, e assim sucessivamente, ate o u nico inter-
ruptor multiplo de 100. Ao final deste procedimento, pergunta-se:

(a) Quais l
ampadas estao apagadas?
k
Demonstracao. Dado um n umero natural n = p 1 0
1 pk , com pi s
0
primos e i s 0, sabemos que o n
umero de divisores de n e

d(n) = ki=1 (i + 1).

Para responder essa pergunta, basta analisar quantos divisores cada


numero de 1 a 100 possui. Na verdade, basta verificar a paridade
da quantidade de divisores de cada n umero, ja que se d(n) for par,
a lampada n ficara acesa. Da expressao acima, e claro que d(n) e
mpar se, e somente se, i e par, para cada valor de i. Assim, d(n)
e mpar se, e somente se, n e um quadrado. Logo, as lampadas que
ficar
ao apagadas sao aquelas que tem numero que e um quadrado
perfeito.

(b) Quantas l
ampadas acenderam exatamente 4 vezes?

Demonstrac
ao. Basta contar quantas lampadas tem 8 ou 9 diviso-
res. Fazendo a contagem usando a expressao em produto de primos,
obtemos a lista: 24, 30, 36, 40, 42, 54, 56, 66, 70, 78, 88 e 100.

(c) Qual e o n
umero da lampada que mais acendeu?

Demonstracao. Essa lampada corresponde aos n umeros que tem mais


divisores. O problema esta mal-formulado, ja que temos cinco n
umeros
com exatamente 12 divisores: 60, 72, 84, 90 e 96.

2 Sugest
ao: Use o fato de que existe a N tal que = |p(a)| > 1 e mostre que divide a
p(k + a), para todo k Z.
Resoluc
ao de Problemas
Lista 03

1. De quantas maneiras podemos escolher tres n umeros distintos do conjunto


I50 = {1, 2, 3, . . . , 49, 50} de modo que sua soma seja

a) um m
ultiplo de 3?
b) um n
umero par?
2. Considere o conjunto In = {1, 2, 3, . . . , n 1, n}. Diga de quantos modos e
possvel formar subconjuntos de k elementos nos quais nao haja n umeros
consecutivos?

3. Considere as letras da palavra PERMUTA. Quantos anagramas de 4 letras


podem ser formados, onde:
a) n
ao h
a restric
oes quanto ao n
umero de consoantes ou vogais?
b) o anagrama comeca e termina por vogal?
c) a letra R aparece?
d) a letra T aparece e o anagrama termina por vogal?
4. Calcular a soma de todos os n umeros de 5 algarismos distintos formados
com os algarismos 1, 3, 5, 7 e 9.

5. Quantos numeros podem ser formados pela multiplicacao de alguns ou de


todos os n
umeros 2, 2, 3, 3, 3, 5, 5, 6, 8, 9, 9?
6. Quantos s
ao os n
umeros naturais de sete dgitos nos quais o dgito 4 figura
exatamente 3 vezes e o dgito 8 figura exatamente 2 vezes?

7. De quantas maneiras uma comissao de 4 pessoas pode ser formada, de um


grupo de 6 homens e 6 mulheres, se a mesma e composta de um n umero
maior de homens do que de mulheres?
8. O comprimento de uma palavra e a quantidade de caracteres que ela pos-
sui. Encontre a quantidade de palavras de comprimento 5 que podemos
formar fazendo uso de 10 caracteres distintos, de forma que nao existam
tres caracteres consecutivos identicos em cada palavra.
9. De quantos modos 6 casais podem sentar-se ao redor de uma mesa circular
de tal forma que marido e mulher nao fiquem juntos?
10. Quantas s
ao as permutacoes das letras da palavra PROFMAT em que o
P ocupa o primeiro lugar, ou o T ocupa o segundo lugar, ou o A o sexto
lugar?
11. De quantas formas podemos representar o n
umero 15 como soma de varios
n
umeros naturais?
12. Quantos quadrados perfeitos existem entre 40.000 e 640.000 que sao m
ultiplos
simultaneamente de 3, 4 e 5?
13. Oito amigos vao ao cinema assistir a um filme que custa um real. Quatro
deles possuem uma nota de um real e quatro possuem uma nota de dois
reais. Sabendo-se que o caixa do cinema nao possui nenhum dinheiro, de
quantas formas eles podem organizar uma fila para pagar o filme permi-
tindo o troco pelo caixa?
14. Encontre o n
umero de zeros que termina o n
umero 2010!.

15. A func
ao de Euler associa a cada n
umero natural n o valor (n) igual ao
n
umero de inteiros positivos menores ou iguais a n relativamente primos
com n. Ou seja,

(n) = {1 m n; (m, n) = 1} .

Usando os princpios estudados, mostre que se n se decompoe em fatores


k
primos como n = p 1 2
1 p2 . . . pk , ent
ao
    
1 1 1
(n) = n 1 1 ... 1 .
p1 p2 pk

Você também pode gostar